ASIPP Controlled Substance Management Questions Flashcards

1
Q
  1. A 25-year-old male receiving hydrocodone and diazepam
    presents with disorientation. He states that he had
    nausea, vomiting, abdominal pain and diarrhea since he
    took “too many pain pills”. During this time he becomes
    extremely lethargic, with slow respirations. No other
    history is available. Your immediate action, in addition to
    O2 administration is to administer:
    A. Naloxone
    B. Diphenoxylate
    C. N-acetyl-L-cysteine
    D. Prochlorperazine
    E. Flumazenil
A
  1. Answer: A
    Explanation:
    CNS depression is hallmark of overdosage with opioids
    or benzodiazepines.
    Respiratory depression is seen with opioids.
    Nausea, vomiting, abdominal pain, and diarrhea are
    early signs of the severe liver toxicity caused
    by high levels of acetaminophen. Other symptoms of
    acetaminophen toxicity include dizziness,
    excitement, and disorientation.
    A. Naloxone is antagonist of opioids. Symptoms indicate
    opioid overdose.
    B. Diphenoxylate is an antidiarrheal agent
    C. N-acetyl-L-cysteine is the appropriate treatment for
    acetaminophen overdose.
    D. Prochlorperazine is to treat the nausea and vomiting
    caused by radiation therapy, cancer chemotherapy, surgery,
    and other conditions
    E. Flumazenil is a competitive benzodiazepine receptor
    antagonist.
    Source: Stern - 2004
How well did you know this?
1
Not at all
2
3
4
5
Perfectly
2
Q

2475.Once an opioid treatment is selected, titration upwards
should continue until:
A. A ceiling is reached.
B. Addiction occurs
C. Tolerance occurs
D. A balance between analgesia and side effects is reached.
E. Respiratory depression occurs

A
  1. Answer: D
    Explanation:
    There is no ceiling for opioids (other than the limitations
    of agonist/antagonists or APAP). The goal is to prevent
    addiction. Tolerance is less likely with long acting opioids.
    Respiratory depression is unlikely with stable doses of
    opioids. The goal is a balance between pain relief and
    intolerable side effects.
    Source: Trescot AM, Board Review 2004
How well did you know this?
1
Not at all
2
3
4
5
Perfectly
3
Q

2476.

A
  1. Answer: D
    Explanation:
    A. Naltrexone is an antagonist therapy for heroin
    addiction
    B. Physostigmine is used to treat glaucoma
    C. Pralidoxime is used together with another medicine
    called atropine to treat poisoning caused by organic
    phosphorus pesticides
    D. Flumazenil is a competitive antagonist of
    benzodiazepines at the GABA receptor.
    Repeated administration is necessary because of its
    short half-life relative to that of most
    benzodiazepines.
    E. Naloxone is an opioid antagonist.
How well did you know this?
1
Not at all
2
3
4
5
Perfectly
4
Q

2477.A patient on methadone 90 mg daily, stable, with good
relief now presents with a kidney stone. For the present
problem :
A. Continue at 90 mg of methadone daily
B. Stop methadone
C. Continue 90 mg of methadone, but add higher than
normal doses of hydrocodone
D. Continue 90 mg of methadone and add lower doses of
hydrocodone
E. Continue 90 mg of methadone and add usual doses of
hydrocodone

A
  1. Answer: E

Source: Laxmaiah Manchikanti, MD

How well did you know this?
1
Not at all
2
3
4
5
Perfectly
5
Q

2478.A 29-year-old male uses secobarbital to satisfy his
addiction to barbiturates. During the past week, he is
imprisoned and is not able to obtain the drug. He is
brought to the prison medical ward because of the onset
of severe anxiety, increased sensitivity to light, dizziness,
and generalized tremors. On physical examination, he is
hyperrefl exic. Which of the following agents should he be
given to diminish his withdrawal symptoms?
A. Buspirone
B. Chloral hydrate
C. Chlorpromazine
D. Diazepam
E. Trazodone

A
  1. Answer: D
    Explanation:
    Reference: Hardman, p 564.
    A. The anxiolytic effects of buspirone take several days to
    develop, obviating its use for acute severe anxiety.
    B. Chloral hydrate a sedative, is used in the short-term
    treatment of insomnia .
    C. Chlorpromazine is used to treat psychotic disorders
    and symptoms such as hallucinations, delusions, and
    hostility
    D. A long-acting benzodiazepine, such as diazepam, is
    effective in blocking the secobarbital withdrawal
    symptoms.
    E. Trazodone is an anti-depressant
    Source: Stern - 2004
How well did you know this?
1
Not at all
2
3
4
5
Perfectly
6
Q

2479.Which of the following statements regarding controlled
substances prescriptions is FALSE?
A. A controlled substances prescription must be dated as
of and signed on the day it is issued.
B. A controlled substances prescription must contain the full name and address of the patient.
C. A physician who has given his or her staff at least
eight hours of training on the federal laws and
regulations concerning controlled substances prescriptions
has no liability for a controlled substance
prescription completed by a staff member that does not
comply with the federal requirements.
D. A controlled substances prescription must contain the
name of the drug, the strength of the drug, the dosage
form of the drug, quantity of drugs prescribed, and directions
for use.
E. A prescription for a Schedule III, IV, or V drug given
for the purpose of detoxifi cation or maintenance treatment
must include the unique identifi cation number
issued by the Administrator of DEA in addition to the
physician’s DEA number.

A
  1. Answer: C
    Explanation:
    Answer (c) is wrong because, under 21 CFR 1306.05, the
    physician is responsible if the prescription does not
    conform to applicable laws and regulations.
    Reference: 21 CFR 1306.05.
    Source: Erin Brisbay McMahon, JD, Sep 2005
How well did you know this?
1
Not at all
2
3
4
5
Perfectly
7
Q

2480.A 16-year old patient has terminal cancer and has failed
all treatment. Pain is worsening and he requires higher
doses of opioid analgesics for pain relief. He inquires as
to whether a research program may or may not help. One
of the side effects with the new treatment is worsening of
peripheral neuropathy. At this point, he refuses further
treatment. His parents want you to talk to him and enroll
him in the experimental protocol. Which of the following
is your next course of action?
A. Inform the patient that he can not refuse treatment
B. Begin treatment if the parents provide written consent
C. Respect the patient’s wishes and cancel plans for treatment
D. Avoid further escalation in opioid doses.
E. Discuss the issues with the patient

A
  1. Answer: E
    Explanation:
    The next course of action is to explore the issues with the
    patient.
How well did you know this?
1
Not at all
2
3
4
5
Perfectly
8
Q

2481.Addiction is defi ned as:
A. Physical dependence and the need to increase the drug to
obtain the same effect.
B. A patient who needs the drug and has good control over
personal behavior.
C. Psychological dependence on the use of controlled substances
for their psychic effects and is characterized by
compulsive use.
D. Slow but progressive deterioration of health in light of
drug use.
E. Loss of effectiveness of the drug to control pain.

A
  1. Answer: C
    Explanation:
    Addiction is a chaotic disturbance in physical and
    psychological control factors that involve impulse control,
    and often evolves to a patient utilizing the drug to their
    detriment, inducing physical harm, and personal disregard
    of danger. There is a signifi cant loss of personal control,
    and the patient seeks the drug, sometimes at all costs.
    Addiction does not necessarily mean a non-functional
    individual. We see some levels of addiction, even with our
    very straightforward pain control patients, i.e. tobacco use.
    Functional alcoholics perform in some segments of
    society, and it is not uncommon to be introduced to a
    patient with an iatrogenic addiction to a controlled
    substance such as benzodiazepine (Xanax?), opioid,
    Oxycontin?. It is also not unusual to hear that some of
    these patients are very highly respected members of
    society, kind and caring, and have had personality changes
    that are not appreciated by the individual. This is where
    professional and family intervention is necessary.
    Source: Hans C. Hansen, MD
How well did you know this?
1
Not at all
2
3
4
5
Perfectly
9
Q

2482.According to the 2003 National Survey on Drug Use and
Health (NSDUH), 6.3 million persons age 12 or older
used prescription medications for non-medical reasons.
Which of the following is the most prevalent agent for
non-medical reasons?
A. Pain reliever
B. Tranquilizers
C. Stimulant
D. Sedatives
E. NSAIDs

A
  1. Answer: A

Source: Hans C. Hansen, MD

How well did you know this?
1
Not at all
2
3
4
5
Perfectly
10
Q

2483.Fluoxetine (Prozac®) is classifi ed as:
A. As an MAO inhibitor (MAOI)
B. As a tricyclic nonselective amine reuptake inhibitor
C. As a heterocyclic nonselective amine reuptake inhibitor
D. As a selective serotonin reuptake inhibitor
E. As an alpha2-adrenergic receptor inhibitor

A
  1. Answer: D
    Explanation:
    Fluoxetine is a highly selective serotonin reuptake
    inhibitor (SSRI) acting on the 5-HT transporter. It forms
    an active metabolite that is effective for several days.
    Selective serotonin reuptake inhibitors are inhibitors of
    cytochrome P450 isoenzymes, which is the basis of
    potential drug interactions
How well did you know this?
1
Not at all
2
3
4
5
Perfectly
11
Q
  1. Tolerance is:
    A. A need to increase drug dosage to obtain the same effect.
    B. A rapid immunity to opioids, secondary to cross-reactive
    antibodies.
    C. The concept of understanding that a drug is necessary,
    except in the community.
    D. A patient’s ability to take the drug.
    E. The physician’s willingness and acceptance to prescribe
    the drug.
A
  1. Answer: A
    Explanation:
    Tolerance is the concept of the need to increase dosage of
    drug to produce the same level of analgesic capacity that
    previously existed. Tolerance may occur at a constant
    dose, and tolerance should not be considered within the
    defi ned purview of addiction.
    Source: Hans C. Hansen, MD
How well did you know this?
1
Not at all
2
3
4
5
Perfectly
12
Q

2485.All of the following are true statements regarding
ketamine EXCEPT:
A. Ketamine is a dissociative anesthetic
B. Dissociative anesthesia induced by ketamine emphasizes
that the anesthetized patient is “disconnected”- from
his or her environment
C. Ketamine is one of the most commonly abused drugs
D. Ketamine has been placed in Schedule I of the Federal
Controlled Substances Act.
E. Ketamine induces coma in a dose-dependent manner

A
  1. Answer: D
    Explanation:
    A. Ketamine is a dissociative anesthetic
    B. Dissociative anesthesia induced by ketamine
    emphasizes that the anesthetized patient is “disconnected”-
    from his or her environment
    C. Persons who abuse ketamine may use a variety of
    routes of administration, and general anesthesia obviously
    is not the object of their use. It is the low-dose mental
    state that ketamine induces is considered as reinforcing by
    substance abusers.
    D. Ketamine is a Schedule III drug
    E. Ketamine induces coma in a dose-dependent manner.
    A minimum of 0.5 mg/kg intravenous is necessary to
    induce coma for approximately 1.5 minutes.
    A dose of 1 mg/kg induces coma for approximately
    5.8 minutes, whereas a dose of 2 mg/kg
    induces coma for approximately 10 minutes.
How well did you know this?
1
Not at all
2
3
4
5
Perfectly
13
Q

2486.The W.H.O. cancer pain relief ladder is:
A. Poorly validated with anecdotal reports of effectiveness.
B. Emphasizes non-opioid treatment through the fi rst three
steps.
C. Is inappropriate to utilize on a terminally ill patient.
D. Is a simple and effective tool to afford relief with a high
level of confi dence, in an overwhelming majority of
patients.
E. Suggests alternative therapy management in the early
stages.

A
  1. Answer: D
    Explanation:
    The W.H.O. ladder emphasizes Step One: Non-Opioid,
    Step Two: Opioid for Mild to Moderate Pain, and Step
    Three: Opioids for Moderate to Severe Pain. The ladder is
    just that. It increases potency and adjunctive medication
    to treat pain, and is highly effective, well-validated,
    suggesting 90% of cancer patients receiving relief. 75% of
    terminally ill patients also report relief.
    Source: Hans C. Hansen, MD
How well did you know this?
1
Not at all
2
3
4
5
Perfectly
14
Q

2487.Among the following neuroleptics, the most likely
neuroleptic associated with skeletal muscle rigidity,
tremor at rest, fl at facies, uncontrollable restlessness, and
spastic torticollis is
A. Clozapine
B. Haloperidol
C. Olanzapine
D. Sertindole
E. Ziprasidone

A
  1. Answer: B
    Explanation:
    Reference: Katzung, p 482.
    Haloperidol, a butyrophenone is by far the most likely
    antipsychotic to produce extrapyramidal toxicities.
    Other agents, such as piperazine (an aromatic
    phenothiazine), thiothixene ( a thioxanthene), and
    pimozide ( a diphenylbutyropiperidine) are comparatively
    less likely to produce extrapyramidal toxicity than
    haloperidol.
    The antagonism of dopamine in the nigrostriatal system
    might explain the Parkinson-like effects.
    Both haloperidol and pimozide act mainly on D2
    receptors, whereas thioridazine and piperazine act on
    alpha-adrenergic receptors, and have a less potent but
    defi nite effect on D2 receptors.
    Source: Stern - 2004
How well did you know this?
1
Not at all
2
3
4
5
Perfectly
15
Q
2488.Among the following neuroleptics, the agent most likely
associated with constipation, urinary retention, blurred
vision, and dry mouth is:
A. Chlorpromazine (Thorazine®)
B. Clozapine (Clozaril®)
C. Olanzapine (Zyprexa®)
D. Sertindole (Serdolect®)
E. Haloperidol (Haldol®)
A
  1. Answer: A
    Explanation:
    Reference: Katzung, pp 471, 473, 482.
    The phenothiazines as a class are the most potent
    anticholinergics of the neuroleptics.
    Tolerance to their anticholinergic effects occur in most
    patients.
    Cholinomimetic agents may be used to overcome
    symptoms that persist.
    Source: Stern - 2004
How well did you know this?
1
Not at all
2
3
4
5
Perfectly
16
Q

2489.Drug interactions common to cyclobenzaprine
(Flexeril®) include all of the following except :
A. MAOI agents
B. Barbiturates
C. Tertiary tricyclic antidepressants
D. Zolpidem (Ambien®)
E. Alcohol

A
  1. Answer: D

Source: Hansen HC, Board Review 2004

How well did you know this?
1
Not at all
2
3
4
5
Perfectly
17
Q
  1. Gabapentin (Neurontin®) exerts its analgesic affect by:
    A. Inhibition of reuptake Serotonin and Norepinephrine
    B. Central modulation of the dorsal lateral funiculus at the
    dorsal horn intermediary.
    C. GABA-A affi nity and activation.
    D. The analgesic effect is unknown.
    E. NMDA modulation.
A
  1. Answer: D

Source: Hansen HC, Board Review 2004

How well did you know this?
1
Not at all
2
3
4
5
Perfectly
18
Q

2491.A 36-year-old male with a bipolar disorder is treated
with lithium. Among the following adverse effects, the
side effect attributed to lithium treatment is:
A. Browning of the vision
B. Hypothyroidism
C. Agranulocytosis
D. Neuroleptic malignant syndrome
E. Pseudodepression

A
  1. Answer: B
    Explanation:
    Reference: Katzung, pp 493-494.
    A decrease in thyroid function occurs in most patients on
    lithium. This effect is usually reversible or not
    progressive, but a few patients develop symptoms of
    hypothyroidism.
    A serum thyroid-stimulating hormone (TSH)
    concentration is recommended every 6 to 12 months.
    “Browning” of vision, clinically described as pigmentary
    retinopathy, occurs with thioridazine. This is due to retinal
    deposition of the drug.
    Although neurologic adverse effects (e.g., tremor,
    choreoathetosis, motor hyperactivity, ataxia, dysarthria,
    and aphasia) can occur with lithium, it does not cause the
    neuroleptic malignant syndrome associated with antipsychotic
    agents. Pseudodepression sometimes occurs in patients on antipsychotics.
    This may be related to drug-induced akinesia.
    Source: Stern - 2004
How well did you know this?
1
Not at all
2
3
4
5
Perfectly
19
Q
2492.N-acetyl benzoquinoneimine is the hepatotoxic
metabolite of which drug?
A. Sulindac
B. Acetaminophen
C. Isoniazid
D. Indomethacin
E. Procainamide
A
  1. Answer: B
    Explanation:
    Reference: Hardman, pp 632-633.
    Hepatic necrosis can occur with overdosage of
    acetaminophen. The hepatic toxicity is the result of the
    biotransformation of acetaminophen to Nacetylbenzoquinoneimine,
    which reacts with hepatic
    proteins and glutathione. This metabolite depletes
    glutathione, stores and produces necrosis. The
    administration of N-acetyl-L-cysteine restores hepatic
    concentrations of glutathione and reduces the potential
    hepatotoxicity. Sulindac is biotransformed to sulindac
    sulfi de, the active form of the drug. Both sulindac and its
    metabolites are excreted in the urine and in the feces.
    Indomethacin undergoes a demethylation reaction and an
    N-deacylation reaction. The parent compound and its
    metabolites are mainly excerted in the urine. Procainamide
    is converted to an active metabolite by an acetylation
    reaction. The product that is formed is Nacetylprocainamide
    (NAPA). In addition, procainamide is
    hydrolyzed by amidases. An N-acetylation reaction occurs
    also in the biotransformation of isoniazid. In the liver, the
    enzyme N-acetyltransferase converts isoniazid to
    acetylisoniazid.
    Source: Stern - 2004
How well did you know this?
1
Not at all
2
3
4
5
Perfectly
20
Q

2493.The mechanism of action of most benzodiazepines is
by :
A. Activation of GABA receptors
B. Antagonism of glycine receptors in the spinal cord
C. Blockade of the action of glutamic acid
D. Increased GABA-mediated chloride ion conductance
E. Inhibition of GABA aminotransferase

A
  1. Answer: D
How well did you know this?
1
Not at all
2
3
4
5
Perfectly
21
Q
2494. A drug that is used in the treatment of parkinsonism and
will also attenuate reversible extrapyramidal side effects
of neuroleptic is
A. Amantadine (Symmetrel®)
B. Levodopa (Dopar®)
C. Pergolide (Permax®)
D. Selegiline (Eldepryl®)
E. Trihexyphenidyl (Artane®)
A
  1. Answer: E
How well did you know this?
1
Not at all
2
3
4
5
Perfectly
22
Q
2495.Meprobamate is the active metabolite of which skeletal
muscle relaxant?
A. Carisoprodol (soma®)
B. Cyclobenzaprine (Flexeril®)
C. Methocarbamol (Robaxin®)
D. Valdecoxib (Bextra®)
E. Baclofen®
A
  1. Answer: A

Source: Jackson KC. Board Review 2003

How well did you know this?
1
Not at all
2
3
4
5
Perfectly
23
Q

2496.The following statements are true regarding opioidinduced
constipation.
A. Treat constipation
B. To obtain a surgical consult to rule out complications
C. To evaluate for drug abuse
D. Start on transdermal fentanyl
E. Start on methadone maintenance program

A
  1. Answer: A
    Explanation:
    A. Constipation is the most frequent side effect of opioid
    therapy.
    Tolerance does not develop to this side effect.
    Therefore, as the dose of opioid increases, so does
    the potential for constipation.
    Frank bowel obstruction, biliary spasm, and ileus have
    occurred with opioid use.
    It is crucial to place patients on an active bowel regimen
    that includes laxatives, stool softeners,
    adequate fl uids and exercise, and cathartics as needed to
    prevent the severe constipation that
    can occur with opioid use.
    B. Surgical complications are unlikely.
    C. Constipation is not a symptom of drug abuse.
    D. Transdermal fentanyl may be an option if morphine
    titration fails. Constipation is similar.
    E. Methadone maintenance is not indicated
    Source: Manchikanti L, Board Review 2005
How well did you know this?
1
Not at all
2
3
4
5
Perfectly
24
Q

2497.What document refl ects the practitioner’s explanation
and the patient’s understanding of the risks, benefi ts,
alternative treatments, and special issues concerning the
use of controlled substances?
A. Narcotic contract
B. History and Physical Evaluation form
C. Pain scale evaluation and update report
D. Informed consent form
E. The approval letter from the patient’s health care benefi t
plan

A
  1. Answer: D
    Explanation:
    Reference: The Federation of State Medical Boards’ Model
    Policy for the Use of Controlled Substances for the
    Treatment of Pain, (May 2004); Bolen, J Pain Medicine
    News (Informed Consent).
    Explanation:
    A. This is not the best answer. A narcotic contract
    (typically called a Controlled Substances Treatment
    Agreement) usually contains boundaries for use with high
    risk patients. Most often, boundary language includes the
    use of urine screens, one physician and one pharmacy for
    obtaining controlled substances, a specifi c term for
    periodic review, and a discussion of the consequences
    should the patient fail to abide by the agreement.
    B. This is not the best answer. A History and Physical
    Evaluation form is not used to explain the risks and
    benefi ts of using controlled substances to treat pain.
    Rather, the H&P form is designed to gather information
    about the patient’s medical history and treatment past, so
    the provider can decide on a course of treatment. Once the
    provider and the patient agree upon a treatment plan, the
    provider should engage in informed consent with the
    patient.
    C. This is not the best answer. A pain scale and periodic
    evaluation form are used to follow the patient after
    treatment begins.
    D. This is the best answer. An Informed Consent form is
    different from a Narcotic Contract or Treatment
    Agreement, because it helps the practitioner establish the
    proper interaction between him/her and the patient
    concerning the risks, benefi ts, treatment alternatives, and
    special issues regarding the use of controlled substances to
    treat pain. When a practitioner uses an Informed Consent,
    he/she is minimizing legal exposure for negligence
    associated with the use of controlled substances to treat
    pain. Of course, it is up to the practitioner to follow
    accepted current clinical care standards, which include a
    proper informed consent process.
    E. This is not the best answer. Rarely, if ever, does a letter
    from the patient’s health care benefi t plan contain
    language relating to informed consent.
    Source: Jennifer Bolen, JD, Sep 2005
How well did you know this?
1
Not at all
2
3
4
5
Perfectly
25
Q
2498.Which one of the following effects is unlikely to occur
during treatment with amitriptyline?
A. Alpha adrenoceptor blockade
B. Elevation of the seizure threshold
C. Mydriasis
D. Sedation
E. Urinary retention
A
  1. Answer: B
    Explanation:
    Tricyclics modify peripheral sympathetic effects in two
    ways; through blockade of norepinephrine reuptake at
    neuroeffector junctions and through alpha adrenoceptor
    blockade. Sedation and atropine-like side effects are
    common with tricyclics, especially amitriptyline. In
    contrast to sedative-hypnotics, tricyclics lower the
    threshold to seizures.
    Source: Katzung & Trevor’s Pharmacology, Examination
    and Board Review, 6th Ed., McGraw Hill, New York, 1998
How well did you know this?
1
Not at all
2
3
4
5
Perfectly
26
Q

2499.Regarding the clinical use of antidepressant drugs, which
one of the following statements is false.
A. Patients should be advised not to abruptly discontinue
antidepressant medications.
B. In selecting an appropriate drug for treatment of depression,
the past history of patient response to specifi c
drugs is a valuable guide
C. In the treatment of major depressive disorders, sertraline
is usually more effective than fl uoxetine
D. MAO inhibitors are sometimes effective in depressions
with attendant anxiety, phobic features, and hypochondriasis
E. Weight loss often occurs in patients taking SSRIs

  1. In severe tricyclic antidepressant overdose, it
    would NOT be of value to
    A. Administer lidocaine (to control cardiac arrhythmias)
    B. Institute hemodialysis (to hasten drug elimination)
    C. Administer bicarbonate and potassium chloride (to correct
    acidosis and hypokalemia)
    D. Provide intravenous diazepam (to control seizures)
    E. Maintain the rhythm of the heart by electrical pacing
A
  1. Answer: E
    Explanation:
    There is no evidence that any SSRI is more effective than
    another in its antidepressant effi cacy. While an individual
    patient may respond more favorably to a specifi c drug,
    several controlled studies have shown equivalent effective
    ness of these agents. However, SSRIs may be more effective
    than tricyclic antidepressants in some patients.
    Source: Katzung & Trevor’s Pharmacology, Examination
    and Board Review, 6th Ed., McGraw Hill, New York, 1998
  2. Answer: B
    Explanation:
    Tricyclic antidepressant overdose is a medical emergency.
    The “three Cs”- coma, convulsions, and cardiac problems are
    the most common causes of death. Widening of the
    QRS complex on the ECG is a major diagnostic feature of
    cardia toxicity. Arrhythmias resulting from cardiac
    conductivity (eg, lidocaine). There is no evidence that
    hemodialysis (or hemoperfusion) increases the rate of
    elimination of tricyclic antidepressants, presumably
    because of their large volume of distribution and their
    binding to tissue components.
    Source: Katzung & Trevor’s Pharmacology, Examination
    and Board Review, 6th Ed., McGraw Hill, New York, 1998
How well did you know this?
1
Not at all
2
3
4
5
Perfectly
27
Q

2501.A weak acid drug (A), with a pKa = 6, is given orally.
Assuming that the pH of the stomach equals 3 and the pH
of the blood equals 7, which of the following statements
is true?
A. At equilibrium, there is roughly 1000 times more dissociated
drug than undissociated drug in the stomach
B. At equilibrium, the ratio of dissociated to undissociated
drug in the blood is approximately 10
C. At equilibrium, 10 times more undissociated drug than
dissociated drug is in the blood
D. Drug concentrations on the blood side of the stomach
barrier will never reach the concentration of drug in
the stomach
E. The drug will be more rapidly excreted if the urine is
made acidic

A
  1. Answer: B
    Explanation:
    According to the Henderson-Hasselbalch equation, the pK
    equals the pH when the log of the ratio is ionized
    (dissociated) and protonated (undissociated) forms is 0
    (i.e., their concentrations are equal and have a ratio of 1).
    When the pH of a solution (blood) is 7 and the pK of the
    acid is 6, at equilibrium, the log of the ratio of
    concentrations of ionized form to protonated form is 1
    (i.e., there is 10 times more ionized than protonated acid
    in the blood). When the pH of a solution (stomach) is 3
    and the pK of the acid is 6, the log of the ratio of the
    concentrations of the ionized to protonated forms is –3
    (i.e., the concentration of the ionized form is 1/1000 that
    of the protonated form, meaning that there is 1000 times
    more protonated than ionized acid). Drug will accumulate
    in the compartment in which it is more highly charged
    (ion trapping) – in this case, the blood. Acidifi cation of
    the urine will increase the protonation of an acid and
    increase reabsorption, thereby slowing renal excretion.
How well did you know this?
1
Not at all
2
3
4
5
Perfectly
28
Q
  1. Which of the following is NOT a true statement with
    respect to a Schedule III or IV drug?
    A. Schedule III and IV prescriptions may not be fi lled or
    refi lled more than six months after the date the original
    prescription was issued.
    B. Schedule III and IV prescriptions may not be refi lled
    more than 10 times after the date of the prescription
    unless renewed by the prescribing practitioner.
    C. No Schedule III or IV drug may be dispensed without
    a written or oral prescription, except when dispensed
    directly by a physician or other practitioner to an ultimate
    user.
    D. A pharmacist may dispense a Schedule III or IV drug
    based on a fax to the pharmacy of a written, signed
    prescription transmitted by the practitioner or one of
    his/her staff.
    E. A pharmacist may dispense a Schedule III or IV drug
    based on an oral prescription received from the
    prescribing practitioner, if the oral authorization is
    promptly reduced to writing by the pharmacist.
A
  1. Answer: B
    Explanation:
    Schedule III and IV prescriptions may not be refi lled more
    than 5 times after the date of the original prescription
    unless renewed by the prescribing practitioner.
    Reference: 21 USC 829 (b) - (c).
    Source: Erin Brisbay McMahon, JD, Sep 2005
How well did you know this?
1
Not at all
2
3
4
5
Perfectly
29
Q

2503.Identify the LEAST important advice in defending drug
charges:
A. Comply with all federal and state laws and regulations
governing prescribing – have and follow a compliance/
risk management program
B. Keep current with and comply with DEA Policy Statements
C. Comply with Kentucky Board of Medical Licensure Policies,
Guidelines, and Newsletters, especially Guidelines
for Use of Controlled Substances in Pain Treatment
D. Keep up with your documentation
E. Check if the patients are paying for visits.

A
  1. Answer: E

Source: Erin Brisbay McMahon, JD, Sep 2005

How well did you know this?
1
Not at all
2
3
4
5
Perfectly
30
Q
2504. All of the following are chronic pain related psychiatric
disorders, EXCEPT:
A. Anxiety
B. Depression
C. Sleep disorders
D. Post traumatic stress disorder
E. Increased sexual function
A
  1. Answer: E

Source: Renee R. Lamm, MD, Sep 2005

How well did you know this?
1
Not at all
2
3
4
5
Perfectly
31
Q
  1. Which of the following statements concerning lithium
    is true?
    A. Lithium is used to control agitation associated with
    schizophrenia
    B. Retention of lithium is enhanced by a high-sodium diet
    C. Early signs of lithium toxicity may include tremors
    D. The onset of lithium action occurs within 24 hours
    E. All of the above
A
  1. Answer: C
    Explanation:
    Severe tremors, along with confusion, drowsiness,
    vomiting, ataxia and dizziness, are an early sign of lithium
    toxicity. Retention of lithium may be enhanced by a lowsodium
    diet because sodium competes with lithium for
    reuptake in the kidney. The onset of lithium action may
    take a week or more; the drug is used to normalize mood
    in patients with mania or bipolar disorder.
How well did you know this?
1
Not at all
2
3
4
5
Perfectly
32
Q
  1. If a patient brings unused controlled substances back to
    you at your offi ce, you should do which of the following?
    A. Dispose of the controlled substances after the patient
    leaves your offi ce and write down what you did in the
    medical record.
    B. Inventory the returned controlled substances and use
    them with other patients who cannot afford to pay for
    prescriptions for these drugs because they do not have
    health insurance.
    C. It depends on regulations of State Board of Medical
    Licensure
    D. Flush the stuff down the toilet.
    E. Call a DEA agent to come and get the drugs.
A
  1. Answer: C
    Explanation:
    Reference: 21 C.F.R. § 1307.21 (Disposal of Controlled
    Substances).
    Explanation:
    A. This is not the best answer. You must understand the
    federal law in this area and then check to see if you state
    requires more of you when it comes to disposing of
    controlled substances. If you wait until after the patient
    leaves your offi ce to record the disposal or destruction of
    the returned controlled substances, you may be in
    violation of state licensing board regulations/rules or
    guidelines, policy, or position statements, or the
    controlled substances act. Further, the patient may try to
    argue that he/she returned more medication than what
    you wrote down in the medical record. The federal law
    states in § 1307.21(a) “[a]ny person in possession of any
    controlled substance and desiring or required to dispose
    of such substance may request assistance from the Special
    Agent in Charge of the Administration in the area in
    which the person is located for authority and instructions
    to dispose of such substance. The request should be made
    as follows: (1) If the person is a registrant, he/she shall list
    the controlled substance or substances which he/she
    desires to dispose of on DEA Form 41, and submit three
    copies of that form to the Special Agent in Charge in his/
    her area. Remember, your state guidelines, laws, and
    regulations may be stricter and prohibit certain actions or
    require more from you in this area.
    B. This is not the correct answer. Except in very limited
    circumstances which are outside the scope of this question pattern, the law prohibits the reintroduction of controlled
    substances in this manner.
    C. This is the best answer. Check with your state licensing
    board and/or state bureau of narcotics to determine
    whether you can dispose of these controlled substances at
    your offi ce and, if the board says it is appropriate to do so,
    have the patient inventory the controlled substances
    returned, write down the amount on a return form, sign
    the return form, use a witness to sign the return form, and
    then have the patient witnessed as he/she fl ushes the
    returned substances down the toilet. Alternatively, but only
    if your state allows this option, you may follow the
    guidance of 21 C.F.R. § 1307.21, when disposing of
    controlled substances.
    D. This is not the best answer. While fl ushing returned
    drugs down the toilet may seem like an easy option, some
    states actually prohibit this action. See the answers to A
    and C above for complete guidance.
    E. This is not the best answer. While 21 C.F.R. § 1307.21
    permits a registrant to contact the Special Agent in Charge
    of the nearest DEA offi ce, your state may require you to do
    something else. Thus, answer C is the best answer and you
    should check with your state licensing board and/or state
    bureau of narcotics for guidance on how to dispose of
    controlled substances returned by patients. In all cases, you
    should document the medical record accurately and
    completely.
    Source: Jennifer Bolen, JD, Sep 2005
How well did you know this?
1
Not at all
2
3
4
5
Perfectly
33
Q
  1. Which of the following is an action of a non-competitive
    antagonist?
    A. Alters the mechanism of action of an agonist
    B. Alters the potency of an agonist
    C. Shifts the dose-response curve of an agonist to the right
    D. Decreases the maximum response to an agonist
    E. Binds to the same site on the receptor as the agonist
A
  1. Answer: D
    Explanation:
    A noncompetitive antagonist decreases the magnitude of
    the response to an agonist but does not alter the agonist’s
    potency (i.e., the ED50 remains unchanged). A
    competitive antagonist interacts at the agonist binding site.
How well did you know this?
1
Not at all
2
3
4
5
Perfectly
34
Q

2508.Drug interactions involving antidepressants do NOT
include
A. Additive impairment of driving ability in patients taking
trazodone when ethanol is ingested
B. Behavioral excitation and hypertension in patients taking
MAO inhibitors with meperidine
C. Elevated plasma levels of lithium if fl uoxetine is administered
D. Increased antihypertensive effects of methyldopa when
tricyclics are administered
E. Prolongation of tricyclic drug half-life in patients with
cimetidine

A
  1. Answer: D
    Explanation:
    Tricyclic drugs block the uptake of guanethidine into
    sympathetic nerve endings, thus reversing its benefi cial
    effects on blood pressure. While the precise mechanism is
    not defi ned, the tricyclics may also block the
    antihypertensive effects of clonidine and methyldopa. All
    of the other drug interactions have been reported.
    Source: Katzung & Trevor’s Pharmacology, Examination
    and Board Review, 6th Ed., McGraw Hill, New York, 1998
How well did you know this?
1
Not at all
2
3
4
5
Perfectly
35
Q

2509.A substance abuser who decides to abstain checks in
to a county detoxifi cation facility and undergoes a 5
day detoxifi cation program. Assuming the abuser gets
no further treatment or aftercare, his or her chance at
remaining sober is about:
A.

A
  1. Answer: A
    Explanation:
    The rate of continued abstinence after simple
    detoxifi cation is about 2 to 3%. Prolonged treatment and
    aftercare markedly increase the success rate.
    Source: Roger Cicala, MD, Sep 2005
How well did you know this?
1
Not at all
2
3
4
5
Perfectly
36
Q

2510.A patient presents for treatment of mechanical and
radicular chronic spine pain resulting from severe
degenerative disease with multilevel stenosis. He has
had a 2 level fusion in the past, undergone multiple
interventional techniques and physical therapy without
benefi t. His radicular pain has improved somewhat
with gabapentin 400 mg QID but he continues to have
severe mechanical back pain. He has a past history of
cocaine and alcohol abuse, but has been clean and sober
for 7 years. When considering chronic opioid therapy in
this patient which of the following is the most correct
statement?
A. Opioid therapy may be appropriate, but the patient must
be advised he has increased risk of relapse or cross addiction.
B. Opioid therapy is not appropriate because of his past history
of substance abuse.
C. The patient is not at increased risk because opioids were
never his drug of choice.
D. Opioid therapy is acceptable, but only short-acting
agents such as hydrocodone or oxycodone should be
used.
E. The physician could be arrested for “aiding and abetting addiction” if he or she prescribes controlled substances
for the patient.

A
  1. Answer: A
    Explanation:
    A. The majority of recovering persons can be
    successfully treated with chronic opioid therapy, but they
    have some increased risk of addiction.
    B. The majority of recovering persons can be
    successfully treated with chronic opioid therapy, but they
    have some increased risk of addiction.
    C. Even though opioids were not the patient’s drug of
    choice, cross addiction can and does occur.
    D. Short acting agents may be more likely to trigger
    addictive disease than long acting agents. They certainly
    are no safer.
    E. There is no such legal ramifi cation, although a
    physician may face licensure issues for prescribing to a
    known active addict.
    Source: Roger Cicala, MD, Sep 2005
How well did you know this?
1
Not at all
2
3
4
5
Perfectly
37
Q
2511. A substance abuser enters and completes a 6 months long
term treatment program. His or her chance at remaining
sober is about:
A. 5%
B. 10%
C. 30%
D. 60%
E. 90%
A
2511. Answer: D
Explanation:
Long term treatment has the highest success rate for
continued abstinence, about 65%.
Source: Roger Cicala, MD, Sep 2005
How well did you know this?
1
Not at all
2
3
4
5
Perfectly
38
Q

2512.When compared to the general population, the mortality
rate of a prescription drug abuse is about:
A. The same
B. 3 times higher
C. 6 times higher
D. 10 times higher
E. 25 times higher

A
  1. Answer: B

Source: Roger Cicala, MD, Sep 2005

How well did you know this?
1
Not at all
2
3
4
5
Perfectly
39
Q

2513.You have agreed to see a new patient who comes to
you through a referral from a family physician in a city
approximately 50 miles from your practice. The patient
gives you a referral package that does not contain any
reference to the patient’s recent history with the other
physician. The referral package also does not contain
any reference to the patient’s history of compliance with
the family physician’s treatment plan or instructions
regarding the patient’s use of controlled drugs. The
patient asks you to prescribe controlled substances to
treat his/her pain and reports that he/she is using a high
dosage of a specifi ed Schedule II controlled substance.
The patient claims he/she has run out of medications
and will experience acute withdrawal symptoms if he/she
does not get the medications from you. What should
you do prior to minimize the potential for abuse and
diversion of these drugs by this patient?
A. Prescribe the patient a months’ worth of drugs and see
him/her back in your offi ce after you obtain the medical
records from the previous physician.
B. Call the previous physician and attempt to verify the
patient’s self-report of recent pain treatments and
medications, use an appropriate urine screen to verify
the presence of the drug the patient says he/she has
taken and to determine whether he/she is
C. Attempt to verify the patient’s self-report of recent pain
treatments and medications and perform an appropriate
urine screen.
D. Accept the patient’s self-report and continue with treatment
without verifi cation through the prior provider.
E. Send the patient back to the previous provider for necessary
controlled substances to treat acute withdrawal
symptoms and tell him/her that you will not prescribe
controlled substances until you receive all the necessary
records from the prior provider.

A
  1. Answer: B
    Explanation:
    Reference: Miscellaneous accepted standards of care; the
    Federation of State Medical Boards’ Model Policy for the
    Use of Controlled Substances for the Treatment of Pain
    (May 2004); and www.deadiversion.usdoj.gov; various
    Intractable Pain Treatment Acts from states like
    California, Tennessee, and Texas.
    Explanation:
    A. This is not the best answer because a provider has a
    responsibility to minimize the potential for abuse and
    diversion of controlled substances (DEA Interim Policy
    Statement, Nov. 16, 2004). If the patient is not known to
    you and you do not have reliable paperwork from which to
    make an informed judgment about the patient’s history of
    prior pain treatments, including the use of medications
    reported, then you should be very cautious about
    prescribing medication. Several states have policy
    statements cautioning physicians to “control the drug
    supply.”
    B. This is the best answer. The fact pattern shows that you
    have agreed to see this patient and establish a physicianpatient
    relationship. For this reason, you must balance
    your ethical duty to prevent the patient from experiencing
    acute withdrawal symptoms with your legal/regulatory
    obligation to minimize the potential for abuse and
    diversion of controlled substances. Do what you can to
    verify the patient’s self-report. Use an appropriate form of
    urine or serum screen, especially if the patient has a
    history of substance abuse (him/herself or through a fi rstdegree
    family relative). Control the initial supply of
    controlled substances to this patient. C. This is not the best answer. You should attempt to
    verify the patient’s self-report of recent pain treatments
    and medications and perform an appropriate urine screen.
    However, you should also control the drug supply and this
    answer omits that statement and fails to acknowledge the
    ethical duty to prevent or minimize the patient’s acute
    withdrawal from controlled substances.
    D. This is not the best answer. If you simply accept the
    patient’s self-report and continue to treat him/her without
    verifying the patient’s past pain treatments and use of
    controlled substances, you are likely violating one of your
    state’s regulations/rules or guidelines/policies/position
    statements on the use of controlled substances for the
    treatment of pain. More importantly, you are likely
    ignoring accepted standards of care and may place yourself
    in a position of prescribing controlled substances outside
    the usual course of professional practice.
    E. This may appear to be the best answer and it might be
    if you had not agreed to see the patient on a referral basis.
    In other words, if someone simply drops into your
    practice on a Friday afternoon claiming they want you to
    treat their pain and that their prior physician
    recommended you, then you are right to be very cautious
    about taking this patient without having the appropriate
    referral material. If you elect to send a patient back to his/
    her referring provider (or if they show up at your offi ce on
    a self-referral) make sure you contact the referring
    provider and/or caution the patient to go to the emergency
    room if they believe they are undergoing acute withdrawal
    symptoms. No answer is easy in this situation and your
    best bet is to document your medical record carefully and
    ensure you document your clinical rational for saying “no,”
    including any aspect of the patient’s (1) medical history,
    (2) behaviors, (3) physical indings, and (4) lab screen or
    test results.
    Source: Jennifer Bolen, JD, Sep 2005
How well did you know this?
1
Not at all
2
3
4
5
Perfectly
40
Q

2514.Which one of the following is characteristic of
cytochrome P-450?
A. Located in the lipophilic environment of mitochondrial
membranes
B. Catalyzes O-, S-, and N-methylation reactions
C. Catalyzes aromatic and aliphatic hydroxylations
D. Catalyzes conjugation reactions
E. Activity is not inducible by drugs

A
  1. Answer: C
    Explanation:
    Cytochrome P-450 catalyzes demethylation, not
    methylation or conjugation, reactions. The enzyme is
    located in the endoplasmic reticulum, not in
    mitochondria. Drugs can induce cytochrome P-450
    activity.
How well did you know this?
1
Not at all
2
3
4
5
Perfectly
41
Q
2515.All of the following describe psychological components
of pain, EXCEPT:
A. Catastrophic thinking
B. Helplessness
C. Compulsive search for a job
D. Blame
E. Chronic maladaptive coping patterns
A
  1. Answer: C

Source: Renee R. Lamm, MD, Sep 2005

How well did you know this?
1
Not at all
2
3
4
5
Perfectly
42
Q
2516.Demerol (meperidine) should not be used for chronic
pain because:
A. it is addictive
B. it is ineffective
C. the metabolite causes seizures
D. the medication is expensive
E. all of the above
A
  1. Answer: C
    Explanation:
    All opioids can potentially be abused. Meperidine may be
    useful for acute pain, and it is cheap. The metabolite
    normeperidine can cause seizures and can accumulate
    with chronic dosing, especially in renal failure
    Source: Trescot AM, Board Review 2004
How well did you know this?
1
Not at all
2
3
4
5
Perfectly
43
Q

2517.What are correct statements about drug interactions of
following statements?
A. Amitriptyline will increase morphine metabolism
B. Morphine will decrease desipramine levels
C. Erythromycin will decrease opioid levels
D. Tricyclic antidepressants will increase methadone levels
E. Propoxyphene will increase propranolol levels

A
  1. Answer: E
    Explanation:
    Propoxyphene will increase carbazepine, doxepin,
    metoprolol and propranolol levels, and decrease the
    excretion of benzodiazepines, leading to accumulation and
    overdose. Amitriptyline will decrease morphine
    breakdown, leading to increased blood levels. Morphine
    will decrease the breakdown of desipramine, leading to
    increased doses. Erythromycin will increase opioid effects,
    and methadone will increase TCA levels.
    References: Bergendal L, Friberg A, Schaffrath AM, et al,
    The clinical relevance of the interaction between
    carbamazepine and dextropropoxyphene in elderly
    patients in Gothenburg,
    Abernethy DR, Greenblatt DJ, Morse DS, et al, Interaction
    of propoxyphene with diazepam, alprazolam and
    lorazepam. Br J Clin Pharmacol 1985;19:51-7
    Source: Andrea M. Trescot, MD
How well did you know this?
1
Not at all
2
3
4
5
Perfectly
44
Q

2518.A urine drug test in a patient on hydrocodone was
positive for hydrocodone and hydromorphone. Choose
the correct option.
A. The patient is taking Dilaudid
B. Refuse to write any more opioids
C. Ignore the results as a false positive
D. Counsel the patient regarding taking drugs that have not
been prescribed
E. Avoid the use of SSRIs in this patient

A
  1. Answer: E
    Explanation:
    The hydromorphone is most likely from the 2D6
    metabolism of hydrocodone, not from abuse. Discharging
    the patient, refusing to write more opioids, or ignoring the
    results would be inappropriate. SSRIs, especially
    fl uoxetine and paroxetine, will inhibit 2D6 and prevent the
    metabolism of hydrocodone to hydromorphone, which
    will decrease his analgesia.
    Source: Andrea M. Trescot, MD
How well did you know this?
1
Not at all
2
3
4
5
Perfectly
45
Q

2519.Benzodiazepines differ from barbiturates in that
benzodiazepines :
A. Facilitate the action of aminobutyric acid (GABA) on
neuronal chloride channels
B. Have anticonvulsant activity
C. May induce physical dependence
D. Have a higher margin of safety than barbiturates
E. All of the above

A
  1. Answer: D
    Explanation:
    Benzodiazepines are much safer than barbiturates because
    they cause minimal central nervous system depression.
    Both drug classes facilitate the action of aminobutyric acid
    (GABA), although by different mechanisms of action.
    They are both used to prevent seizures, and both can result
    in physical dependence with long-term use.
How well did you know this?
1
Not at all
2
3
4
5
Perfectly
46
Q
2520.Methadone in addition to being a μ-receptor agonist has
been proposed to also act as a:
A. COX-2 inhibitor
B. Sodium Channel blocker
C. NMDA receptor antagonist
D. Delta receptor agonist
E. Opiod Antagonist
A
  1. Answer: C
    Explanation:
    One of the two rate limiting steps in prostaglandin
    synthesis is the conversion of arachidonic acid
    to the prostanoid precursor PGH2 by cycloxygenase
    (COX). COX-2 is an isozyme of COX and
    mediates responses to infl ammation, infection and
    injury.
    Methadone is a synthetic opioid derivative which seems
    to function both as a μ-receptor agonist
    and an NMDA receptor antagonist.
    The drug has a tendency to accumulate with repeated
    administration.
    It is excreted almost exclusively in the feces and can be
    given to patients with compromised renal
    function, however caution should be used
    Methadone is equipotent to morphine after parenteral administration.
    Source: Chopra P, 2004
How well did you know this?
1
Not at all
2
3
4
5
Perfectly
47
Q

2521.Alkalinization of the urine with sodium bicarbonate is
useful in the treatment of poisoning with
A. Aspirin (acetylsalicylic acid)
B. Amphetamine
C. Morphine
D. Phencyclidine
E. Cocaine

A
  1. Answer: A
    Explanation:
    Reference: Hardman, pp 16-20.
    A. Sodium Bicarbonate is excreted principally in the urine
    and alkalinizes it.
    Increasing urinary pH interferes with the passive renal
    tubular reabsorption of organic acids (such
    as aspirin and Phenobarbital) by increasing the ionic
    form of the drug in the tubular fi ltrate.
    This would increase their excretion.
    B-E. Excretion of organic bases (such as amphetamine,
    cocaine, phencyclidine, and morphine) would be enhanced
    by acidifying the urine.
    Source: Stern - 2004
How well did you know this?
1
Not at all
2
3
4
5
Perfectly
48
Q

2522.True statement(s) with regards to urine drug testing
include:
A. Thin layer chromatography is a relatively new technique,
most sensitive, labor intense, and expensive.
B. Gas chromatography is most sensitive, most reliable,
inexpensive, an old and established technique.
C. Enzyme immunoassay is less sensitive than thin layer
chromatography, more sensitive than gas chromatography,
and has ability to screen multiple drugs at a time.
D. Rapid drug screens are similar to other enzyme immunoassay
tests but may be more expensive or less
expensive.
E. Rapid drug screens are less sensitive than enzyme immunoassay,
gas chromatography, and thin layer chromatography
and highly unreliable, but least expensive.

A
  1. Answer: D
    Explanation:
    Urine Drug Testing
    * Thin-layer chromatography (TLC)
    - Relatively old technique, testing the migration of a
    drug on a plate or fi lm, which is compared to a known
    control
    * Gas chromatography: liquid and mass spectometry
    (CGMS)
    - Most sensitive and specifi c tests
    - Most reliable
    - Labor intensive/costly
    - Several days to know results
    - Used to confi rm results of other tests
    * Enzyme immunoassay
    - Easy to perform/highly sensitive
    - More sensitive than TLC
    - Less expensive than GC/MS
    - Common tests
    EMIT (enzyme multiplied immunoassay test)
    FPIA (fl uorescent polarization immunoassay)
    RIA (radioimmunoassay)
    - Screen only one drug at a time
    - Rapid drug screens
    - Similar to other enzyme immunoassay tests
    - May be more expensive
How well did you know this?
1
Not at all
2
3
4
5
Perfectly
49
Q
2523.Which of the following opioids is vagolytic?
A. Morphine
B. Meperidine
C. Sufentanil
D. Nalbuphine
E. Alfentanil
A
  1. Answer: B
    Source: American Board of Anesthesilogy, In-trainnig
    examination
How well did you know this?
1
Not at all
2
3
4
5
Perfectly
50
Q

2524.Which of the following factors will determine the
number of drug-receptor complexes formed?
A. Effi cacy of the drug
B. Receptor affi nity for the drug
C. Therapeutic index of the drug
D. Half-life of the drug
E. Rate of renal secretion

A
  1. Answer: B
    Explanation:
    Receptor affi nity for the drug will determine the number
    of drug-receptor complexes formed. Effi cacy is the ability
    of the drug to activate the receptor after binding has
    occurred. Therapeutic index (TI) is related to safety of the
    drug. Half-life and secretion are properties of elimination
    and do not infl uence formation of drug-receptor
    complexes.
    Source: Laxmaiah Manchikanti, MD
How well did you know this?
1
Not at all
2
3
4
5
Perfectly
51
Q

2525.A former heroin addict is maintained on methadone, but
succumbs to temptation and buys an opioid on the street.
He takes it and rapidly goes into withdrawal. Which
opioid did he take?
A. Meperidine
B. Heroin
C. Pentazocine
D. Codeine
E. Propoxyphene

A
  1. Answer: C
    Explanation:
    Reference: Hardman, p 546.
    Pentazocine is a mixed agonist-antagonist of opioid
    receptors. When a partial agonist, such as pentazocine,
    displaces a full agonist, such as methadone, the receptor is
    less activated; this leads to withdrawal syndrome in an
    opioid-dependent person.
    Source: Stern -2004
How well did you know this?
1
Not at all
2
3
4
5
Perfectly
52
Q

2526.Concerning the proposed mechanisms of action of
antidepressant drugs, which one of the following
statements is accurate?
A. Bupropion (Wellbutrin®) can effective inhibitor of Nor-
Epinephrine and 5-HT transporters
B. Chronic treatment with an antidepressant often leads to
the up-regulation of adrenoceptor
C. Elevation in amine metabolites in cerebrospinal fl uid is
characteristic of most depressed patient prior to drug
therapy
D. MAO inhibitors used as antidepressants selectively decrease
the metabolism of norepinephrine
E. The acute effect of most tricyclics is to block the neuronal
reuptake of both norepinephrine and serotonin in the CNS

A
  1. Answer: E
    Explanation:
    The mechanism of action of bupropion is unknown, but
    the drug does not inhibit amine transporters. Levels of
    norepinephrine and serotonin metabolites in the
    cerebrospinal fl uid of depressed patients prior to drug
    treatment are not higher than normal. Some studies have
    reported decreased levels of these metabolites. Downregulation
    of adrenoceptor appears to be a common
    feature of all mode3s of chronic drug treatment of
    depression, including the use of drugs that have no direct
    actions o catecholamine receptors. MAO inhibitors used
    in depression are nonselective.
    Source: Katzung & Trevor’s Pharmacology, Examination
    and Board Review, 6th Ed., McGraw Hill, New York, 1998
How well did you know this?
1
Not at all
2
3
4
5
Perfectly
53
Q
  1. The Drug Abuse Warning Network (DAWN) is a network
    that:
    A. Treats addicted patients
    B. Is a self help and support group
    C. Collects information on hospital emergency department
    admissions for drug-related episodes
    D. Is an arm of the Drug Enforcement Agency
    E. Monitors physicians’ prescription habits
A
  1. Answer: C

Source: Hans C. Hansen, MD

How well did you know this?
1
Not at all
2
3
4
5
Perfectly
54
Q
2528.Which of the following opioids is not a good choice in
patients with renal failure:
A. Fentanyl®
B. Sufentanil®
C. Morphine®
D. Alfentanil®
E. Meperidine®
A
  1. Answer: E
    Explanation:
    Ref: Murphy. Chapter 16. Opioids. In: Clinical
    Anesthesia, 2nd Edition. Barash, Cullen, Stolling;
    Lippincott, 1992, pg 431
    Source: Day MR, Board Review 2003
How well did you know this?
1
Not at all
2
3
4
5
Perfectly
55
Q

2529.What are the correct statements about drug abuse in
patients receiving opioids?
A. Illicit drug use and abuse of prescription controlled substances
is non-existent in patients receiving long-acting
opioids in appropriate doses.
B. Short-acting opioids show signifi cantly higher use of illicit
drugs and abuse of controlled prescription drugs.
C. Illicit drug use and controlled substance abuse in chronic
non-cancer patients is similar whether they are on
short-acting or long-acting opioids.
D. Patients on long-acting opioids are highly compliant
and functional and therefore not necessary to monitor
them.
E. Patients on short-acting opioids signifi cantly abuse
drugs. Thus, they should be monitored every month
with urine drug testing.

A
  1. Answer: C
    Explanation:
    Source: Manchikanti L - Pain Physician 2005; 8:257-262.
How well did you know this?
1
Not at all
2
3
4
5
Perfectly
56
Q

2530.A 28-year-old woman presents with symptoms of major
depression that are unrelated to a general medical
condition, bereavement, or substance abuse. She is
not currently taking any prescription or over-thecounter
medications. Drug treatment is to be initiated
with a selective serotonin reuptake inhibitor. In your
information to the patient, you would NOT tell her that
A. Divided doses may help to reduce nausea and gastrointestinal
distress
B. Muscle cramps and twitches sometimes occur
C. She must inform you if she anticipates using other
medications
D. Taking the drug in the evening will ensure a good night’s
sleep
E. The drug may require 2 weeks or more to become effective

A
  1. Answer: D
    Explanation:
    The SSRIs have CNS-stimulating effects. They may cause
    agitation, anxiety, “the jitters”, and insomnia. The evening
    is not the best time to take such drugs. Anorexia and
    nausea, akathisia, dyskinesias, and dystonic reactions may
    occur. Because of the possibility of drug interactions, the
    physician needs to be informed of changes in drug
    regimens when maintaining a patient on antidepressants.
    Source: Katzung & Trevor’s Pharmacology, Examination and Board Review, 6th Ed., McGraw Hill, New York, 1998
How well did you know this?
1
Not at all
2
3
4
5
Perfectly
57
Q

2531.Choose the correct statement with regards to the
comparison of effectiveness and safety profi les of longacting
versus short-acting opioids in chronic non-cancer
pain.
A. There was conclusive evidence from high quality randomized
and systematic trials to determine that longacting
opioids as a class were more effective and safer
with no adverse events than short-acting opioids.
B. There was no good-quality data available to assess comparative
effi cacy and adverse event risks in sub-populations
of patients with chronic non-cancer pain.
C. There was insuffi cient evidence from available trials to
determine whether long-acting opioids as a class are
more effective or associated with fewer adverse events
than short-acting opioids.
D. There were approximately 20 randomized trials investigating
long-acting oxycodone versus short-acting oxycodone
showing conclusive evidence that long-acting
oxycodone was superior to short-acting oxycodone.
E. Recent GAO reports, DEA statements, and media attention
and case reports of abuse, addiction, and overdose
from long-acting opioids represent a decreased risk
proportionate to prescribing pattern changes as these
are widely used.

A
  1. Answer: C
    Source: Chou et al. - J Pain Manage Symptom Manage Vol.
    25, No. 5 Nov. 2003, 1026-1048.
How well did you know this?
1
Not at all
2
3
4
5
Perfectly
58
Q

2532.Choose the correct statement about effectiveness of
one or more long-acting opioids in reducing pain and
improving functional outcomes.
A. There was insuffi cient evidence to prove that different
long-acting opioids are associated with different effi -
cacy or adverse event rates.
B. OxyContin was shown to be more effective with a lesser
side effect profi le compared to morphine.
C. MS Contin was superior to transdermal fentanyl.
D. The combination of OxyContin, Xanax, and Soma were
superior to all other drugs.
E. The combination of morphine, diazepam, and oxycodone
were superior to MS Contin alone.

A
  1. Answer: A
    Source: Chou et al. - J Pain Manage Symptom Manage Vol.
    25, No. 5 Nov. 2003, 1026-1048.
How well did you know this?
1
Not at all
2
3
4
5
Perfectly
59
Q

2533.Which of the following statements applies to a drug
exhibiting a saturated elimination process?
A. Upon multiple dosing, steady-state plasma concentrations
will be reached in approximately 4 to 5 biologic
half-lives
B. The fraction of drug eliminated per unit time is constant
C. The biologic half-life (t1/2) is affected by dose
D. First-order kinetics are operable
E. The rate of drug elimination is dependent on plasma
drug concentration

A
  1. Answer: C
    Explanation:
    The biologic half-life (t1/2) will be affected by dose in a
    drug when the elimination process is saturated. In that
    case, the drug will accumulate on repeated dosings, and
    elimination will be independent of plasma concentration.
    The amount (not fraction) of drug eliminated per unit
    time will be constant, ad zero-order elimination will be
    observed.
How well did you know this?
1
Not at all
2
3
4
5
Perfectly
60
Q

2534.When selecting and dosing opioids:
A. There is no ceiling dose for combination analgesics
B. Use long acting opioids for as-needed pain
C. Use short acting opioids for around-the-clock pain
D. There is no ceiling dose for pure agonists
E. Agonist-antagonist opioids are appropriate breakthrough
medications

A
  1. Answer: D
    Explanation:
    There is no specifi c ceiling dose for pure opioid agonists,
    though we are becoming aware of the potential for
    hyperalgia from at least morphine metabolites.
    Combination drugs are limited by the APAP or NSAID
    content. Use long acting for baseline pain, and short acting
    opioids for “breakthrough” pain. Agonist-antagonists have
    very little role in chronic pain management in general,
    and specifi cally will trigger withdrawal when used as
    breakthrough meds for other agonists.
    Source: Andrea M. Trescot, MD
How well did you know this?
1
Not at all
2
3
4
5
Perfectly
61
Q

2535.N-methyl-D-aspartate (NMDA) receptors are best
defi ned as:
A. NMDA receptors are calcium-permeable ion channels
that require only glutamate for activation.
B. NMDA receptors are calcium-permeable ion channels that
require only glycine for activation.
C. NMDA receptors are calcium-permeable ion channels
that require both glutamate and glycine for activation.
D. At the molecular level, NMDA receptors are composed
of a single subunit.
E. NMDA receptors have small intracellular C-termini that
interact with a single protein that regulates receptor
phosphorylation.

A
  1. Answer: C
    Explanation:
    A. NMDA receptors require glutamate and glycine for
    activation.
    B. NMDA receptors require glycine, but also glutamate for
    activiation.
    C. NMDA receptors are calcium-permeable ion channels
    that require both glutamate and glycine for activation.
    The amino acid-binding sites on the receptor are
    contributed by two different subunits, NR1 and NR2.
    Antagonism of
    either of these two sites is suffi cient to completely block
    the ion fl ocks that normally follows receptor activation.
    D. At the molecular level, NMDA receptors are composed
    of multiple subunits that co-assemble to form functional
    channels.
    NR1 subunits exist as a family of 8 splice variants
    generated by alternative splice of 1N-terminal
    cassette and to intracellular C-terminal cassettes.
    The presence of 1 or more of the NR1 and NR2
    subunits in a single receptor complex confers
    unique biophysical and pharmacologic properties to the
    NMDA receptor.
    E. NMDA receptors have large intracellular C-termini
    that interact with a variety of important proteins that
    regulate receptor phosphorylation and clustering to
    important signaling complexes.
How well did you know this?
1
Not at all
2
3
4
5
Perfectly
62
Q

2536.According to the DSM-IV, which of the following is a
criterion for diagnosing substance ABUSE?
A. Using a substance for a purpose other than that described
in the PDR
B. Substance use in hazardous situations
C. Development of tolerance to the substance
D. Attempts to cut down substance use
E. An episode of withdrawal

A
  1. Answer: B
    Explanation:
    A. Using a substance for purpose other than described
    in the PDR has nothing to do with substance abuse.
    B. Use of a substance in hazardous situations is one of
    the criteria for substance abuse.
    C. Development of tolerance to the substance can occur
    in any individual who takes the substance, even properly as
    prescribed. It MAY indicate substance dependence but is
    not a criteria for indicating substance abuse.
    D. Attempts to cut down substance use are on of the
    criteria for substance dependence, not substance abuse.
    E. An episode of withdrawal substance can occur in any
    individual who takes the substance in suffi cient quantities,
    even properly as prescribed. It MAY indicate substance
    dependence but is not a criteria for indicating substance
    abuse.
    Source: Roger Cicala, MD, Sep 2005
How well did you know this?
1
Not at all
2
3
4
5
Perfectly
63
Q
2537. Compared to morphine, butorphanol would be expected
to display which one of the following pharmacologic
characteristics?
A. Kappa receptor antagonist activity
B. Analgesic ceiling effect
C. No respiratory depression
D. Histamine release
E. Mu receptor agonist activity
A
  1. Answer: B
    Explanation:
    Butorphanol exhibits opioid kappa receptor agonist and
    mu receptor antagonist activity. Respiratory depression
    with butorphanol is similar to that produced by equivalent
    morphine doses. Histamine release is prominent with
    morphine and not butorphanolStoelting RK.
    Pharmacology and Physiology in Anesthesia Practice. 3rd
    ed. Lippincott-Raven, Philadelphia, 1999.
    Evers AS, Maze M. Anesthetic Pharmacology: Physiologic
    Principles and Clinical Practice. Churchill Livingstone,
    Philadelphia, 2004.
    Stoelting RK. Pharmacology and Physiology in Anesthesia
    Practice. 3rd ed. Lippincott-Raven, Philadelphia, 1999
    Source: James D. Colson, MS, MD
How well did you know this?
1
Not at all
2
3
4
5
Perfectly
64
Q
  1. Which of the following characteristics of buprenorphine
    best account for its effectiveness in the treatment of
    opioid dependence and detoxifi cation?
    A. Sublingual formulation with naloxone
    B. High opioid mu-1 receptor affi nity and slow receptor
    dissociation
    C. Partial kappa receptor agonist activity
    D. Low oral bioavailability necessitating parenteral administration
    E. High intrinsic activity at the opioid mu receptor
A
  1. Answer: B
    Explanation:
    Buprenorphine has low oral bioavailability, but is well
    absorbed sublingually. Naloxone has poor sublingual
    bioavailability, but is formulated along with
    buprenorphine to prevent misuse when administered IV.
    Buprenorphine is a partial agonist at the mu-opiate
    receptor and an antagonist at the kappa receptor.
    While buprenorphine has high opioid mu receptor affi nity,
    it has a low intrinsic activity. Malinoff HL, Barkin RL, Wilson G. Sublingual
    buprenorphine is effective in the treatment of chronic pain
    syndrome. American Journal of Therapeutics 2005; 12(5):
    379-384
    http://buprenorphine.sahsa.gove/about.html
    Source: James D. Colson, MS, MD
How well did you know this?
1
Not at all
2
3
4
5
Perfectly
65
Q

2539.Which of the following is NOT considered the imposition
of discipline:
A. Revocation or suspension.
B. An order that a physician pay the actual costs of the
investigation.
C. A reprimand.
D. An order to obtain 20 hours of Category I continuing
medical education in the area of management of persistent
pain.
E. An order that a physician pay a civil forfeiture or penalty.

A
  1. Answer: B
    Explanation:
    As an order to pay costs is considered merely a costshifting
    measure which keeps the cost of licensure down
    for physicians who have not committed unprofessional
    conduct.
How well did you know this?
1
Not at all
2
3
4
5
Perfectly
66
Q

2540.A patient who had called for an early refi ll of opioid
medication is rather agitated and jittery when seen in
the offi ce. She states she had not run out of her opioid
medication but simply thought she needed a higher dose.
All of the following symptoms would suggest opiate
withdrawal EXCEPT :
A. Diarrhea
B. Piloerection
C. Pinpoint pupils
D. Sweating
E. Rhinorrhea

A
  1. Answer: C
    Explanation:
    Pinpoint pupils are a sign of opiate intoxication, dilated
    pupils would be more likely in withdrawal. All of the other
    symptoms are associated with opiate withdrawal.
    Source: Roger Cicala, MD, Sep 2005
How well did you know this?
1
Not at all
2
3
4
5
Perfectly
67
Q
  1. A PET scan is performed on a known substance abuser
    who receives their drug of choice during the procedure.
    The PET scan would show activation of :
    A. Nucleus Accumbens
    B. Ventral Tegmental Areas
    C. Central Nucleus of the Amygdala
    D. All of the above
    E. None of the above, they would actually be suppressed
A
  1. Answer: D
    Explanation:
    Areas throughout the mesolimbic system, including all of
    the above, the stria terminalis, and portions of the frontal
    lobes are all activated during intake of a substance of
    abuse.
    Source: Roger Cicala, MD, Sep 2005
How well did you know this?
1
Not at all
2
3
4
5
Perfectly
68
Q

2542.Neurochemically, ALL substances considered abusable
eventually activate:
A. Cholinergic pathways in the brainstem
B. Adrenergic pathways in the frontal lobes
C. GABA-B receptors diffusely throughout the brain
D. Serotonergic centers in the diencephalon
E. Dopaminergic neurons in the mesolimbic system

A
  1. Answer: E
    Explanation:
    Different substances may activate different primary
    receptors in the brain, but dopaminergic neurons in the
    mesolimbic system are activated either directly, or
    secondarily with all substances of abuse.
    Source: Roger Cicala, MD, Sep 2005
How well did you know this?
1
Not at all
2
3
4
5
Perfectly
69
Q

2543.Which of the following statements are true?
A. Opioid calculators are very useful and reliable.
B. If a patient complains of breakthrough pain, you should
double the dose of long acting opioid every day until
pain relief.
C. Nerve pain might respond better to anticonvulsants
than opioids
D. Bone pain might respond better to anticonvulsants than
opioids
E. Opioid hyperalgia may be due to M6G accumulation

A
  1. Answer: C
    Explanation:
    Opioid calculators are potentially dangerous to use
    secondary to metabolic polymorphism. Long acting
    opioids have a long half-life, reaching steady state in
    approximately 5 days, so increasing doses too frequently
    can lead to overdose. Bone pain probably responds better
    to NSAIDs. Hyperalgia may be due to M3G.
    Source: Andrea M. Trescot, MD
How well did you know this?
1
Not at all
2
3
4
5
Perfectly
70
Q
  1. According to the DSM-IV, which of the following is NOT
    a criterion for diagnosing substance DEPENDENCE?
    A. Continued use despite physical or psychological problems
    caused by use.
    B. Recurrent substance related legal problems
    C. Development of tolerance to the substance
    D. Attempts to cut down substance use
    E. An episode of withdrawal
A
  1. Answer: B
    Explanation:
    Recurrent substance related legal problems are a clear sign
    of abuse, but do not in themselves indicate there is
    dependence upon the substance.
    Source: Roger Cicala, MD, Sep 2005
How well did you know this?
1
Not at all
2
3
4
5
Perfectly
71
Q

2545.Which of the following is the most accurate defi nition
of tolerance:
A. The medication stops working after a few months
B. Stopping the medication causes withdrawal symptoms
C. A given dose is less effective, increasing the dose restores
the effect
D. Side effects of a given dose are less severe over time
E. A medication is less effective, changing to a different
medication restores the effect

A
  1. Answer: C
    Explanation:
    A. Tolerance involves reduced effectiveness, not
    complete loss of effect
    B. Withdrawal can occur without tolerance, nor does
    withdrawal always occur when tolerant patients stop the
    substance in question.
    C. Tolerance indicates less effectiveness of a given dose.
    Increasing the dose can restore the effect, up to a point.
    D. Many opiate side effects decrease over time, but not
    necessarily in parallel with developing tolerance.
    E. A different medication may be more effective, but this
    does not of itself demonstrate tolerance.
    Source: Roger Cicala, MD, Sep 2005
How well did you know this?
1
Not at all
2
3
4
5
Perfectly
72
Q

2546.A heroin addict comes to the emergency room in an
anxious and agitated state. He complains of chills, muscle
aches, and diarrhea; he has also been vomiting. His
symptoms include hyperventilation and hyperthermia.
He claims to have had an intravenous “fi x” approximately
12 hours ago. The attending physician notes that pupil
size is greater than normal. What is the most likely cause
of these signs and symptoms?
A. The patient has overdosed with an opioid
B. These are early signs of the toxicity of MPTP, a contaminant
in “street heroin”
C. The signs and symptoms are those of the abstinence syndrome
D. In addition to opioids, the patient has been taking barbiturates
E. The patient has hepatitis B

A
  1. Answer: C
    Explanation:
    Explanation: The signs and symptoms are those of
    withdrawal in a patient physically dependent on an opioid
    agonist. Such signs and symptoms usually start within 6-
    10 hours after the last dose; their intensity depends on the
    degree oh physical dependence that has developed. Peak
    effects usually occur at 36-48 hours. Mydriasis is a
    prominent feature of the abstinence syndrome; other
    symptoms include rhinorrhea, lacrimation, piloerection,
    muscle jerks, and yawning.
    Source: Katzung & Trevor’s Pharmacology, Examination
    and Board Review, 6th Ed., McGraw Hill, New York, 1998
How well did you know this?
1
Not at all
2
3
4
5
Perfectly
73
Q
2547.Which one of the following drugs is most likely to
increase plasma levels of alprazolam, theophylline, and
warfarin:
A. Desipramine (Pamelor®)
B. Fluvoxamine (Luvox®)
C. Imipramine (Tofranil®)
D. Nefazodone (Serazone®)
E. Venlafaxine (Effexor®)
A
  1. Answer: B
    Explanation:
    Fluvoxamine inhibits liver drug-metabolizing enzymes.
    Dosages of alprazolam, theophylline, and warfarin must be
    reduced if any of these drugs are given concomitantly with
    fl uvoxamine. Nefazodone may also decrease the
    metabolism of benzodiazepines, and venlafaxine may
    inhibit haloperidol metabolism.
    Source: Katzung & Trevor’s Pharmacology, Examination
    and Board Review, 6th Ed., McGraw Hill, New York, 1998
How well did you know this?
1
Not at all
2
3
4
5
Perfectly
74
Q

2548.A patient injured in an auto accident received 80 mg
of meperidine. He subsequently developed a severe
reaction characterized by tachycardia, hypertension,
hyperpyrexia, and seizures. When a questioned, the
uninjured spouse revealed that the patient had been
taking a drug for a psychiatric condition. Which of the
following drugs is most likely to be responsible for this
untoward interaction with meperidine?
A. Alprazolam (Xanax®)
B. Amitriptyline (Elavil®)
C. Lithium
D. Mirtazapine (Norbil®)
E. Phenelzine (Nardil®)

A
  1. Answer: E
    Explanation:
    Concomitant administration of meperidine and MAO
    inhibitors has resulted in life-threatening hyperpyrexic
    reactions that may culminate in seizures or coma. Such
    reactions have even occurred when phenelzine was
    administered 14 days after a patient had been treated with
    meperidine! Note that concomitant use of SSRIs and
    meperidine has resulted in the serotonin syndrome,
    another life-threatening drug interaction.
    Source: Katzung & Trevor’s Pharmacology, Examination
    and Board Review, 6th Ed., McGraw Hill, New York, 1998
How well did you know this?
1
Not at all
2
3
4
5
Perfectly
75
Q
2549.Fentanyl patches have been used to provide analgesia.
The most dangerous adverse effect of this mode of
administration is
A. Cutaneous reactions
B. Diarrhea
C. Hypertension
D. Relaxation of skeletal muscle
E. Respiratory depression
A
  1. Answer: E
    Explanation:
    The fentanyl transdermal patch releases the drug over 72
    hours. The blood levels achieved will often provide
    analgesia for postoperative pain but at the same time will
    increase arterial PCO2 due to depression of the brain stem
    respiratory center. This effect has contributed to severe
    respiratory depression with occasional fatalities.
    Source: Katzung & Trevor’s Pharmacology, Examination
    and Board Review, 6th Ed., McGraw Hill, New York, 1998
How well did you know this?
1
Not at all
2
3
4
5
Perfectly
76
Q

2550.A recently bereaved 74-year-old female patient was
treated with benzodiazepine for several weeks after the
death of her husband, but she did not like the daytime
sedation it caused. She has no major medical problems but
appears rather infi rm for her age and has poor eyesight.
Because her depressive symptoms are not abating, you
decide on trial of an antidepressant medication. Which
one of the following drugs would be the most appropriate
choice for this patient?
A. Amitriptyline
B. Mirtazapine
C. Paroxetine
D. Phenelzine
E. Trazodone

A
  1. Answer: C
    Explanation:
    The elderly patient may be especially sensitive to
    antidepressant drugs that cause sedation, atropine-like side
    effects, or postural hypotension. Paroxetine (or another
    SSRI) is the best choice for this patient because it is the
    least likely of the drugs listed to exert such actions.
    Source: Katzung & Trevor’s Pharmacology, Examination
    and Board Review, 6th Ed., McGraw Hill, New York, 1998
How well did you know this?
1
Not at all
2
3
4
5
Perfectly
77
Q

2551.Pharmacologic options for pain:
1. opioids work for peripheral, nerve, spinal cord, or brain
pathologies
2. local anesthetics work at the brain level
3. AEDs work at the nerve root and the brain
4. Anti-infl ammatories work at the periphery only.

A
2551. Answer: A (1, 2, & 3)
Explanation:
Anti-infl ammatories work on peripheral as well as central
tissues.
Source: Andrea M. Trescot, MD
How well did you know this?
1
Not at all
2
3
4
5
Perfectly
78
Q
2552.According to NIDA household surveys and the National
Comorbidity Survey, the prevalence of substance abuse
among U. S. adults is about:
A. 1%
B. 3%
C. 7%
D. 17%
E. 33%
A
2552. Answer: C
Explanation:
Most studies indicate the prevalence rate of substance
abuse is 6% to 7%.
Source: Roger Cicala, MD, Sep 2005
How well did you know this?
1
Not at all
2
3
4
5
Perfectly
79
Q

2553.Regarding the use of opioids, which of the following
statements is true?
A. Opioid medications have predictable side effects at certain
doses.
B. Blood level of the drug needed for perceived therapeutic
effect is consistent through the day.
C. If a pain signal is too weak to be perceived, it has no
biophysiologic effect.
D. Complete pain relief is the goal of the use of opioid
medication.
E. Treating pain aggressively early may decrease the risk of
sensitization.

A
  1. Answer: E
    Explanation:
    Opioids are unpredictable, and the blood levels needed for
    analgesia vary with level of activity and time of day. Even
    when a pain signal is below threshold, there are potential
    physiologic effects. Early aggressive treatment is the goal
    of preemptive analgesia.
    Source: Andrea M. Trescot, MD
How well did you know this?
1
Not at all
2
3
4
5
Perfectly
80
Q
2554.The most commonly used illicit drug in America is:
A. Oxycontin®
B. Cocaine
C. Morphine®
D. Marijuana
E. Alcohol
A
  1. Answer: D
    Explanation:
    You could easily argue that alcohol far exceeds marijuana
    in use, based on national data. 120 million Americans, or
    50% of the population consume alcohol. The defi nition
    would easily move to alcohol as the number one used illicit
    drug, if alcohol was uniformly considered “a drug”. It
    does have physical dependence, psychic and toxic effects,
    but for purposes of terminology and the use of controlled
    substances, marijuana will be considered the number one
    drug of abuse. Ironically, it is now not a controlled
    substance, as the Supreme Court has defi ned no legitimate
    medical need for marijuana. Marijuana is properly termed
    a drug of abuse, Schedule I. There are 7 thousand new
    users per day.
    Source: Hans C. Hansen, MD
How well did you know this?
1
Not at all
2
3
4
5
Perfectly
81
Q

2555.Adults who have completed four years of college are:
A. Less likely to use an illicit drug
B. More likely to use an illicit drug
C. Adults with four years of college who are alcoholics are
more likely to use an illicit drug.
D. Adults who have not completed high school, are dropouts,
or live in metropolitan areas are more likely to use
illicit drugs.
E. There is no evidence that education has an effect on the
statistical prevalence of illicit drug use.

A
  1. Answer: B
    Explanation:
    It may be surprising, but adults who have completed four
    years of college are more likely to experience an illicit
    drug, 51.1% of the American population. 38% of those
    who did not complete high school have used an illicit
    drug. The use is higher in metropolitan than nonmetropolitan
    areas.
    Source: Hans C. Hansen, MD
How well did you know this?
1
Not at all
2
3
4
5
Perfectly
82
Q
  1. Which of the following is NOT a condition to an oral
    refi ll of a Schedule III or IV prescription?
    A. The total quantity authorized, including the amount of
    the original prescription, does not exceed fi ve refi lls nor
    extend beyond six months from the date of the original
    prescription.
    B. The pharmacist obtaining the oral authorization must
    record on the reverse of the original prescription the
    date, quantity of refi ll, and the number of additional
    refi lls authorized, and must initial the prescription.
    C. The quantity of each additional refi ll authorized is equal
    to or less than the quantity authorized for the initial fi lling
    of the original prescription.
    D. The pharmacist must verify that the oral authorization
    came from a physician or other practitioner by, for example,
    calling the physician back at the number for the
    physician listed in the telephone directory.
    E. The prescribing practitioner must execute a new and
    separate prescription for any additional quantities beyond
    the fi ve-refi ll, six-month limitation.
A
  1. Answer: A
    Explanation:
    While a callback requirement might be a good idea, it is
    not a condition to an oral authorization for a refi ll of a
    Schedule III or IV prescription.
    Reference: 21 CFR 1306.22
    Source: Erin Brisbay McMahon, JD, Sep 2005
How well did you know this?
1
Not at all
2
3
4
5
Perfectly
83
Q

2557.The non-medical use of pain medication has been
followed since 1965. The largest growth of non-medical
use of pain medication since this time has been between:
A. 1986 and 1990
B. 1995 and 2000
C. 2001 and 2002
D. 1965 and 1972
E. 1968 and 1971

A
  1. Answer: B
    Explanation:
    The non-medical use of pain medication experienced its
    largest growth between 1995 and 2000, and has somewhat
    slowed in the past couple of years.
    Source: Hans C. Hansen, MD
How well did you know this?
1
Not at all
2
3
4
5
Perfectly
84
Q

2558.With regard to narcotic addiction treatment, which of
the following statements is most accurate?
A. A practitioner who dispenses Schedule II narcotic drugs
for maintenance or detoxifi cation treatment must obtain
a separate registration every fi ve years as a narcotic
treatment program.
B. Registration as a narcotic treatment program allows a
practitioner to administer, dispense, and prescribe,
Schedule II drugs approved by the Food and Drug Administration
for treatment of narcotic addiction.
C. The only Schedule II drugs approved by the Food and
Drug Administration for treatment of narcotic addiction
are Methadone and levo-alpha-acetyl-methadol
(LAAM).
D. Registration as a narcotic treatment program is contingent
on proper registration with the appropriate state
attorney general.
E. To obtain registration as a narcotic treatment program, a
practitioner must have been engaging in narcotic addiction
treatment for at least fi ve years.

A
  1. Answer: C
    Explanation:
    A) Registration is required every year.
    B) Schedule II drugs cannot be prescribed for narcotic
    addiction.
    C) This is correct.
    D) Registration as an NTP is contingent on proper
    registration with the State Methadone Authority and the
    Department of Health and Human Services.
    E) This is incorrect; a practitioner must obtain SAMHSA
    certifi cation.
    Reference: 21 U.S.C. § 823(g).
    Source: Erin Brisbay McMahon, JD, Sep 2005
How well did you know this?
1
Not at all
2
3
4
5
Perfectly
85
Q
  1. Is it legal for Internet pharmacies to approach a physician
    to write prescriptions based on on-line consultations
    with customers/consumer?
    A. Yes, this is legal and on-line consultations qualify as a
    proper physician-patient relationship.
    B. Yes, but the physician must see the patient in person and
    establish a valid physician-patient relationship prior to
    issuing Internet prescriptions.
    C. No, a physician cannot do this under existing law.
    D. No, a physician cannot do this unless he/she obtains a
    special Internet certifi cation from the DEA.
    E. Yes, but the physician must obtain a special Internet certifi
    cation from DEA.
A
  1. Answer: B
    Explanation:
    Reference: 21 C.F.R. § 1306.04 (prescriptions) and DEA
    Guidance Document on Dispensing and Purchasing
    Controlled Substances over the Internet, Fed. Reg. Vol. 66,
    No. 82 (April 27, 2001);
    http://www.deadiversion.usdoj.gov/fed_regs/notices/2001
    /fr0427.htm.
    Explanation:
    A. This is not the correct answer. A physician may use the
    Internet to provide information and to communicate with
    the patient, but the Internet communications cannot be
    the sole basis for authorizing the prescriptions. If a valid
    physician-patient relationship exists, a physician may use
    the Internet to communicate with patients. For example, a
    physician may use the Internet to receive requests for
    treatment. However, all requests for treatment should be
    logical based upon the physician’s knowledge of the
    patient’s medical history and the presenting complaint.
    DEA states that, assuming a valid physician-patient
    relationship, it is permissible for a physician to use the
    Internet to receive requests for refi lls of prescriptions
    from patients. Practitioners should check to see whether
    their licensing state places additional requirements on
    those who engage in telemedicine.
    B. This is the best answer. A physician may use the
    Internet to communicate with patients, but the Internet
    communications may not form the sole basis for the
    physician-patient relationship. The physician must
    establish a valid physician-patient relationship with each
    patient in accordance with federal and state laws governing
    telemedicine. Most states have internet prescribing
    policies requiring physicians to obtain a thorough medical
    history and conduct an appropriate physical examination
    before prescribing any medication for the fi rst time.
    C. This is not the correct answer. See explanations above.
    D. This is not the correct answer. There is no such Internet
    certifi cation from the DEA allowing physicians to
    prescribe controlled substances to patients over the
    Internet.
    E. This is not the correct answer. There is no such Internet
    certifi cation from the DEA.
    Source: Jennifer Bolen, JD, Sep 2005
How well did you know this?
1
Not at all
2
3
4
5
Perfectly
86
Q
  1. According to the Federal Controlled Substances Act of
    1970, a Schedule III drug could be considered to possess
    which one of the following characteristics:
    A. Lack of accepted safety and indication for medical use
    B. Limited, if any, physical or psychological dependence
    C. High potential for abuse
    D. No withdrawal syndrome noted with abrupt discontinuation
    of use
    E. Analgesic, anabolic, sedative, and/or hypnotic effects
A
  1. Answer: E
    Explanation:
    Schedule III substances are comprised of drugs possessing
    analgesic, anabolic steroid, sedative, and/or hypnotic
    properties.
    Controlled Substances Act-U.S. Drug Enforcement
    Administration (http://www.usdoj.gov/dea/agency/
    csa.htm)
    Source: James D. Colson, MS, MD
How well did you know this?
1
Not at all
2
3
4
5
Perfectly
87
Q
  1. Which of the following statements concerning an opioid
    treatment program (OTP) is false?
    A. An OTP must apply to the Substance Abuse and Mental
    Health Services Administration for certifi cation.
    B. To become certifi ed by the Substance Abuse and Mental
    Health Services Administration (SAMHSA), an OTP
    must be accredited by a SAMHSA-approved accreditation
    body.
    C. OTPs must comply with the Federal Law on Confi dentiality
    of Substance Abuse Patient Records and with the
    HIPAA privacy rule.
    D. OTPs must notify the Substance Abuse and Mental
    Health Services Administration within sixty days of
    any replacement or change in the status of the program
    sponsor or medical director.
    E. OTPs must be registered by the Drug Enforcement Administration
    before administering or dispensing any
    opioid agonist treatment medications.
A
  1. Answer: D
    Explanation:
    Answer (d) is wrong; the time limit is 3 weeks.
    Reference: 42 CFR 8.11
    Source: Erin Brisbay McMahon, JD, Sep 2005
How well did you know this?
1
Not at all
2
3
4
5
Perfectly
88
Q
  1. Compared to a short-acting, immediate release opioid, a
    long-acting, sustained release opioid differs in which one
    of the following characteristics?
    A. Higher level of opioid receptor affi nity and intrinsic
    activity
    B. Greater degree of tachyphylaxis
    C. Faster development of tolerance
    D. Protracted withdrawal syndrome following abrupt discontinuation
    E. No ceiling effect for analgesia
A
  1. Answer: D
    Explanation:
    Drug properties, such as receptor affi nity, intrinsic activity
    or the propensity to develop tolerance or tachyphylaxis
    aremore inherent to the drug itself and not specifi cally to
    its duration of action or particular formulation. Opioids as
    a class do not have a ceiling effect for analgesia. Longacting
    opioids tend to have a delayed onset and protracted
    course of withdrawal following abrupt discontinuation.
    Stoelting RK. Pharmacology and Physiology in Anesthesia
    Practice. 3rd ed. Lippincott-Raven, Philadelphia, 1999.
    Savage SR. Critical clinical issues in pain and addiction.
    Pain Management Rounds 2005; 2(9):
    Source: James D. Colson, MS, MD
How well did you know this?
1
Not at all
2
3
4
5
Perfectly
89
Q

2563.A patient complains of worsening chronic temporal
headache, despite daily treatment with aspirin,
butalbital, caffeine and ergotamine. MRI of the head was
normal, but MRI of the neck demonstrated spondylosis.
Headache most likely is due to:
A. Migraine
B. Drug rebound phenomenon
C. Cervical spondylosis
D. Pseudo-tumor cerebri
E. Vasodilation due to ergotamine

A
  1. Answer: B

Source: Andrea M. Trescot, MD

How well did you know this?
1
Not at all
2
3
4
5
Perfectly
90
Q

2564.For the following statements, fi rst decide if each is true or
false and then select the correct pattern from the choices
given.1)Narcotic agonist/antagonist drugs are useful
agents for pain management in many situations.2)The
metabolism of opioid medication is predictable within
ethnic subsets or behavioral subsets of the general
population.3)CYP2D6 extensive metabolizers (normals)
comprise a majority of the patients tested for the
genotype.
A. 1 is True, 2 is True, 3 is True
B. 1 is False, 2 is True, 3 is True
C. 1 is True, 2 is False, 3 is True
D. 1 is False, 2 is True, 3 is False
E. 1 is False, 2 is False, 3 is False

A
  1. Answer: E
    Explanation:
    Agonist/antagonists are rarely useful in chronic pain
    (other than buprenorphine.) Opioid metabolism is very
    variable, and the (normal)population makes up barely
    half of the population.
    Source: Andrea M. Trescot, MD
How well did you know this?
1
Not at all
2
3
4
5
Perfectly
91
Q

2565.For the following statements, fi rst decide if each
is true or false and then select the correct pattern
from the choices given.1)Codeine is metabolized to
morphine.2)Methadone has no signifi cant drug interacti
ons.3)Hydrocodone is metabolized to hydromorphone.
A. 1 is True, 2 is True, 3 is True
B. 1 is False, 2 is True, 3 is True
C. 1 is True, 2 is False, 3 is True
D. 1 is False, 2 is True, 3 is False
E. 1 is False, 2 is False, 3 is False

A
  1. Answer: C
    Explanation:
    Codeine is metabolized to morphine, and hydrocodone to
    hydromorphone, Methadone has multiple drug
    interactions.
    Source: Andrea M. Trescot, MD
How well did you know this?
1
Not at all
2
3
4
5
Perfectly
92
Q

2566.An individual abruptly discontinuing long-term, high
dose use of an opioid drug will likely experience which
one of the following conditions?
A. Opioid-induced hyperalgesia
B. Increased opioid mu receptor affi nity
C. Increased opioid tolerance
D. Loss of physical dependence and addiction
E. Increased sympathetic nervous system activity

A
  1. Answer: E
    Explanation:
    A withdrawal syndrome with symptoms consistent with
    increased sympathetic activity will occur following the
    abrupt discontinuation of long-term opioid use.
    Hyperalgesia is associated with continued use of high dose
    opiates. Mu receptor affi nity is an inherent pharmacologic
    property of the opioid and would not be expected to
    change
    with its discontinued use. Tolerance will be expected to
    diminish with discontinuation of use, while physical
    dependence is a physiologic state in which abrupt
    cessation of opioid results in a withdrawal syndrome.
    Stoelting RK. Pharmacology and Physiology in Anesthesia
    Practice. 3rd ed. Lippincott-Raven, Philadelphia, 1999.
    Savage SR. Critical clinical issues in pain and addiction.
    Pain Management Rounds 2005; 2(9):
    Source: James D. Colson, MS, MD
How well did you know this?
1
Not at all
2
3
4
5
Perfectly
93
Q
  1. Which one of the following statements about pentazocine
    (Talwin®) is FALSE?
    A. Analgesia is at least equivalent to that of codeine
    B. Causes sedation
    C. Classifi ed as a mixed agonist-antagonist
    D. Full agonist at mu receptors
    E. May interfere with the analgesic effects of morphine
A
  1. Answer: D
How well did you know this?
1
Not at all
2
3
4
5
Perfectly
94
Q
2568. Among persons with a known substance abuse disorder,
the substance most commonly abused is:
A. Marijuana
B. Prescription opiates
C. Cocaine
D. Benzodiazepines
E. Alcohol
A
  1. Answer: E
    Explanation:
    Alcohol accounts for 60% of all cases of substance abuse.
    Source: Roger Cicala, MD, Sep 2005
How well did you know this?
1
Not at all
2
3
4
5
Perfectly
95
Q
  1. Regarding low back pain in a primary care practice:
    A. Less than 10% of new low back pain patients followed up
    with a doctor at 12 months.
    B. Less than 10% of new low back pain patients still had low
    back pain at 12 months
    C. Less than 10% of new low back patients still had diffi culties
    with ADLs at 12 months
    D. Less than 10% of new low back pain patients still had
    pain complaints at 3 months
    E. Less than 10% of new low back pain patients followed up
    with a doctor at 3 months
A
  1. Answer: A
    Explanation:
    Only 8% of new low back pain patients made a return
    appointment after 12 months. 25% still had pain at 12
    months, and 50% still had diffi culties with ADLs at 12
    months. 79% still had pain at 3 months, and 32% made
    follow up at 3 months.
    Croft PR, Macfarlane GJ, Papageorgiou AC, et al. Outcome
    of low back pain in general practice: a prospective study.
    BMJ (1998);316(7141):1356-9
    Source: Andrea M. Trescot, MD
How well did you know this?
1
Not at all
2
3
4
5
Perfectly
96
Q

2570.Which one of the following drugs has been used in
the management of alcohol withdrawal states and in
maintenance treatment of patients with tonic-clonic
or partial seizure states? Its chronic use may lead to an
increased metabolism of warfarin and phenytoin.
A. Chlordiazepoxide
B. Meprobamate
C. Phenobarbital
D. Triazolam
E. Zolpidem

A
  1. Answer: C
    Explanation:
    Chronic administration of phenobarbital increases the
    activity of hepatic drug-metabolizing enzymes, including
    cytochrome P450 isozymes. This often increases the rate
    of metabolism of drugs administered concomitantly, with decreases in the intensity and duration of their effects.
    Source: Katzung & Trevor’s Pharmacology, Examination
    and Board Review, 6th Ed., McGraw Hill, New York, 1998
How well did you know this?
1
Not at all
2
3
4
5
Perfectly
97
Q
  1. The greatest risk in a patient on 300 mg of daily Tramadol
    (Ultram®) and paroxetine (Paxil®) combination is:
    A. Withdrawal
    B. Increased depression
    C. Seizures
    D. Increased pain
    E. GI bleeding
A
  1. Answer: C
    Explanation:
    Because tramadol is not only activated by CYP2D6, but
    also metabolized for excretion by CYP2D6, the patient is at
    risk for accumulation of the drug, leading to seizures.
    Source: Andrea M. Trescot, MD
How well did you know this?
1
Not at all
2
3
4
5
Perfectly
98
Q

2572.The thalamus:
1. Is a collection of cell bodies that process incoming sensory
signals.
2. Can tell the difference between different receptors.
3. Connects to various areas of the cortex.
4. Is made up of wide dynamic response neurons.

A
  1. Answer: B (1 & 2)
    Explanation:
    The thalamus is a collection of cell bodies in the brain that
    process pain signals, and connect to variousareas of the
    cortex . It determines the type of interpretation based on
    region of signal input (all action potentials look alike) and
    is not made up of WDN.
    Source: Andrea M. Trescot, MD
How well did you know this?
1
Not at all
2
3
4
5
Perfectly
99
Q
2573.A 17-year old who ingests “mushrooms” would present
with symptoms best described as:
A. Anticholinergic
B. Adrenergic
C. Cholinergic
D. Alpha adrenergic
E. Dopaminergic
A
  1. Answer: C

Source: Stimmel, B

How well did you know this?
1
Not at all
2
3
4
5
Perfectly
100
Q

2574.A known heroin addict underwent emergency surgery
after a motor vehicle injury. He received morphine 10
mg IV, three doses in 2 hours which helped him only 15
minutes. The course of treatment in this patient is:
A. To titrate morphine
B. To obtain a surgical consult to rule out complications
C. To evaluate for drug abuse
D. Start on transdermal fentanyl
E. Start on methadone maintenance program

A
  1. Answer: A
How well did you know this?
1
Not at all
2
3
4
5
Perfectly
101
Q
  1. The common belief that most people who misuse, divert,
    or assign illicit use to controlled substances are felt to be
    unemployed and from the inner city. The following is
    true of demographics according to SAMHSA:
    A. 75% of illicit drug users were employed full or part time
    B. The highest use of illicit controlled substances were
    among American Indians and Alaskan natives.
    C. Blacks and whites were approximately equal in the use
    and misuse of controlled substances.
    D. Inner city blacks were of the highest percentage of illicit
    drug users.
    E. 60% of illicit drug users were on Medicaid
A
  1. Answer: A
    Explanation:
    75% of illicit drug users were employed full or part time,
    with American Indian and Alaskan natives at about 12%
    of the population of abuse of illicit drug users. Blacks and
    whites were essentially equal, with Asians lowest.
    Source: Hans C. Hansen, MD
How well did you know this?
1
Not at all
2
3
4
5
Perfectly
102
Q
2576.The most commonly used illicit drug is:
A. Marijuana
B. LSD
C. Ecstasy
D. Methamphetamine
E. OxyContin®
A
  1. Answer: A
    Explanation:
    The most commonly used illicit drug remains marijuana,
    with a lifetime use at roughly 20%. It is the most
    commonly used illicit drug on a regular basis, 6.2%, 14.6
    million Americans. In contrast 1% were cocaine users,
    and ectasy is on the decline, from 3.2 million to 2.1 million
    users.
    Source: Hans C. Hansen, MD
How well did you know this?
1
Not at all
2
3
4
5
Perfectly
103
Q
2577.Which of the following opioids is least likely to cause
bradycardia in high doses?
A. Fentanyl®
B. Meperidine®
C. Morphine®
D. Hydromorphone®
E. Oxycodone®
A
  1. Answer: B

Source: Day MR, Board Review 2005

How well did you know this?
1
Not at all
2
3
4
5
Perfectly
104
Q

2578.Identify evidence-based recommendations of meperidine
use in chronic pain.
A. Given no more frequently than every four hours for 6
months
B. Used in standard doses in the elderly indefi nitely if response
is positive
C. Used in individuals with impaired renal function as meperidine
is shown to have only hepatotoxicity
D. Always used in conjunction with non-steroidal anti-infl
ammatory agents.
E. Reserved for very brief therapy in otherwise healthy patients
who cannot tolerate other opioids

A
  1. Answer: E

Source: Stimmel, B

How well did you know this?
1
Not at all
2
3
4
5
Perfectly
105
Q
  1. What is the fi rst step in a patient on 300 mg Tramadol
    per day with paroxetine(Paxil®) with inadequate pain
    relief ?
    A. Change the paroxetine
    B. Add a benzodiazepine
    C. Increase the tramadol
    D. Switch to a more potent opioid
    E. Switch to an agonist/antagonist opioid
A
  1. Answer: A
    Explanation:
    It would be tempting to increase the tramadol, since he is
    not on the maximum of 400mg/day. However, that would
    increase his risk of seizures even more. The simplest
    treatment would be to change his antidepressant to one
    that was not a CYP2D6 inhibitor, which would then allow
    the tramadol to be more active, while at the same time
    decreasing the risk of seizures because of excretion of the
    drug. Codeine, hydrocodone, and oxycodone are also
    metabolized to active forms by CYP2D6, and therefore
    would also be less effective in the face of CYP2D6
    inhibition. Agonist/antagonist opioids are not usually a
    good choice for chronic pain management.
    Source: Andrea M. Trescot, MD
How well did you know this?
1
Not at all
2
3
4
5
Perfectly
106
Q
  1. The Golden Crescent and the Golden Triangle refer to:
    A. A geographic region of Africa where opium is grown.
    B. A geographic region of Asia where marijuana is grown.
    C. Various names from heroin.
    D. Symbols used in drug traffi cking.
    E. A geographic region of Asia where opium is grown.
A
  1. Answer: E

Source: Stimmel, B

How well did you know this?
1
Not at all
2
3
4
5
Perfectly
107
Q
2581.Adverse effects of opioids that can be used to advantage
include:
A. Dysphoria
B. Respiratory depression
C. Decreased GI motility
D. Pupil constriction
E. Sexual dysfunction
A
  1. Answer: C
    Explanation:
    Decreased GI motility is the deliberate effect of poorly
    absorbed opioids such as loperamide (Lomotil). The rest
    are usually undesirable effects.
    Source: Andrea M. Trescot, MD
How well did you know this?
1
Not at all
2
3
4
5
Perfectly
108
Q
  1. Meperidine
    A. Has been used for many years and is appropriate for
    chronic use
    B. Is metabolized by CYP2D6
    C. Is safe to use in elderly patients
    D. May cause side effects that cannot be reversed by opioid
    antagonists.
    E. Becomes more effective over time, because of an active
    metabolite
A
  1. Answer: D
    Explanation:
    The seizures from normeperidine cannot be reversed by
    naloxone. Although it has been used for many years,
    recognition of its poor analgesia, metabolite accumulation,
    and abuse potential has lead to gradual condemnation by
    the pain community. It is metabolized by glucuronidation,
    is not safe in the elderly and does not become more
    effective over time.
    Source: Andrea M. Trescot, MD
How well did you know this?
1
Not at all
2
3
4
5
Perfectly
109
Q
2583. Which of the statements about tolerance is most true
in a patient taking repeated daily doses of lysergic acid
diethylamide (LSD)?
A. No tolerance develops
B. Tolerance develops in 3 to 4 days
C. Tolerance develops in 2 to 3 weeks.
D. Tolerance develops in 2 to 3 months.
E. Tolerance develops in 6 months
A
  1. Answer: B

Source: Stimmel, B

How well did you know this?
1
Not at all
2
3
4
5
Perfectly
110
Q

2584.A physician determines that an emergency situation
exists justifying a Schedule II emergency oral
prescription. Which of the following statements is true?
A. Within 7 days after the oral authorization to the pharmacist,
the physician must deliver to the pharmacist a
written prescription for the emergency quantity prescribed,
and the written prescription must have written
on its face “Authorization for Emergency Dispensing”
and the date of the oral authorization.
B. Within 14 days after the oral authorization to the pharmacist,
the physician must deliver to the pharmacist a
written prescription for the emergency quantity prescribed
and the written prescription must have written
on its face “Authorization for Emergency Dispensing”
and the date of the oral authorization.
C. Within 30 days after the oral authorization to the pharmacist,
the physician must deliver to the pharmacist a
written prescription for the emergency quantity prescribed
and the written prescription must have written
on its face “Authorization for Emergency Dispensing”
and the date of the oral authorization.
D. No further action is required.
E. Within 60 days after the oral authorization to the pharmacist,
the physician must deliver to the pharmacist a
written prescription for the emergency quantity prescribed
and the written prescription must have written
on its face “Authorization for Emergency Dispensing”
and the date of the oral authorization.

A
  1. Answer: A
    Explanation:
    Within 7 days after the oral authorization to the
    pharmacist, the physician must deliver to the pharmacist a
    written prescription for the emergency quantity
    prescribed, and the written prescription must have written
    on its face “Authorization for Emergency Dispensing” and
    the date of the oral authorization.
    Reference: 21 CFR 290.10, 1306.11(d).
    Source: Erin Brisbay McMahon, JD, Sep 2005
How well did you know this?
1
Not at all
2
3
4
5
Perfectly
111
Q

2585.To avoid criminal investigations into prescribing
patterns for controlled substances, a physician should . . .
Choose the answer that best completes this sentence.
A. Warn patients to fi ll prescriptions at different drugstores.
B. Make sure there is a logical relationship between the
drugs prescribed and the treatment of the condition
allegedly existing.
C. Issue prescriptions to patients known to be selling drugs
to others.
D. Prescribe controlled substances at intervals inconsistent
with legitimate medical treatment.
E. Use street slang when talking about the drugs prescribed

A
  1. Answer: B
    Explanation:
    Answers (a) and (c)-(e) are noted in the DEA’s Interim
    Policy Statement as behaviors that lead to criminal
    convictions.
    Reference: 69 Fed. Reg. 67170.
    Source: Erin Brisbay McMahon, JD, Sep 2005
How well did you know this?
1
Not at all
2
3
4
5
Perfectly
112
Q
  1. Which of the following statements is NOT true with
    regard to a continuing criminal enterprise?
    A. Conviction for being the manager or organizer of a continuing
    criminal enterprise results in the person being
    sentenced to not less than 20 years and not more than
    life imprisonment, a fi ne of $2 million or more, and
    forfeiture of assets under 21 USC 853.
    B. To be convicted of being the manager or organizer of a
    continuing criminal enterprise, one must obtain substantial
    income or resources.
    C. To be convicted of being the manager or organizer of a
    continuing criminal enterprise, at least fi ve other persons
    must be involved.
    D. To be convicted of being the manager or organizer of a
    continuing criminal enterprise, a person must violate a
    provision of the Controlled Substances Act, the punishment
    for which is a misdemeanor.
    E. To be convicted of being the manager or organizer of a
    continuing criminal enterprise, a person must engage
    in a continuing series of violations of the Controlled
    Substances Act.
A
  1. Answer: D
    Explanation:
    Answer (d) is wrong because, to be convicted of being the
    manager or organizer of a continuing criminal enterprise,
    a person must violate a provision of the Controlled
    Substances Act, the punishment for which is a felony.
    Reference: 21 U.S.C. § 848.
    Source: Erin Brisbay McMahon, JD, Sep 2005
How well did you know this?
1
Not at all
2
3
4
5
Perfectly
113
Q

2587.A practitioner intending to dispense and prescribe
Schedule III, IV, or V controlled substances for
maintenance and detoxifi cation treatment must submit
to the Secretary of the Department of Health and Human Services a notifi cation that he or she intends to do so. The
notifi cation must state . . .Which one of the following
would not correctly complete this sentence?
A. That the practitioner has the capacity to refer patients to
whom the practitioner provides narcotic drugs for appropriate
counseling and other ancillary services.
B. If the practitioner is in solo practice, that s/he will not
treat more than thirty patients at any one time with
Schedule III, IV, or V drugs for detoxifi cation or maintenance.
C. If the practitioner is in group practice, that the group
practice will not treat more than sixty patients at any
one time with Schedule III, IV, or V drugs for detoxifi -
cation or maintenance.
D. That the practitioner is a qualifi ed physician, as that term
is defi ned in the federal statutes.
E. If the practitioner is in group practice, the names and
DEA registration numbers for all practitioners in a
group practice.

A
  1. Answer: C
    Explanation:
    Answer (c) should be limited to thirty patients.
    Reference: 21 USC 823(g); 21 CFR 1306.07(d).
    Source: Erin Brisbay McMahon, JD, Sep 2005
How well did you know this?
1
Not at all
2
3
4
5
Perfectly
114
Q
  1. Axis IV provides important information about which of
    the following?
    A. Ability to pay for all necessary medical services.
    B. Overall satisfaction with scope of ongoing medical services.
    C. Desire to serve as a subject in an upcoming clinical trial
    for a new medication.
    D. Impact of psychosocial and environmental problems
    that may impact the patient.
    E. Personality disorders
A
  1. Answer: D

Source: Cole EB, Board Review 2003

How well did you know this?
1
Not at all
2
3
4
5
Perfectly
115
Q

2589.A 15-year-old male high school freshman presents to an
emergency department with a blank stare, belligerence,
psychomotor agitation, horizontal nystagmus, vertical
nystagmus, blood pressure 160/110 mm Hg, ataxia
dysarthria, and diminished responsiveness to pain.
He appears to be hallucinating, and as he is being
interviewed he assaults one of the attendants. The most
likely diagnosis is:
A. Phencyclidine (PCP) intoxication.
B. Atropine intoxication
C. Benzodiazepine intoxication
D. Mescaline intoxication
E. Solvent intoxication

A
  1. Answer: A

Source: Stimmel, B

How well did you know this?
1
Not at all
2
3
4
5
Perfectly
116
Q
  1. Under the Drug Addiction Treatment Act of 2000,
    practitioners who are qualifi ed physicians may
    dispense and prescribe Schedule III, IV, or V controlled
    substances specifi cally approved by the Food and Drug
    Administration for narcotic addiction treatment to a
    narcotic dependent person if the practitioner meets
    several requirements.Which one of the following
    credentials will NOT make a physician a qualifi ed
    physician?
    A. The physician holds a subspecialty board certifi cation
    in addiction psychiatry from the American Board of
    Medical Specialties.
    B. The physician holds an addiction certifi cation from the
    American Society of Addiction Medicine.
    C. The physician holds a subspecialty board certifi cation
    in addiction medicine from the American Osteopathic
    Association.
    D. The physician has completed not less than 8 hours of
    training regarding the treatment and management of
    opiate-dependent patients.
    E. The physician has published a peer-reviewed article on
    the treatment of opiate-dependent patients.
A
  1. Answer: E
    Explanation:
    Answer (e) is not one of the credentials that will render a
    physician a qualifi ed physician to dispense and prescribe
    Schedule III, IV, or V controlled substances specifi cally
    approved by the Food and Drug Administration for
    narcotic addiction treatment to a narcotic dependent
    person.
    Reference: 21 USC 823(g)(2)(B)
    Source: Erin Brisbay McMahon, JD, Sep 2005
How well did you know this?
1
Not at all
2
3
4
5
Perfectly
117
Q
  1. The US work loss related to pain:
    A. Half the workforce report having pain in the last two
    weeks.
    B. 10% of the work force was absent from work one or more
    days per week
    C. Abdominal pain was the most common complaint.
    D. Half of the workforce lost productive time due to pain
    E. An average of 8 hours of work per week is lost because of
    pain
A
  1. Answer: A
    Explanation:
    Only 1% of the work force was absent from work,
    headaches and low back pain were the most common
    complaints, 12% of the workforce lost productive time due
    to pain, and an average of 4.6% hours per week are lost
    because of pain.
    Stewart WF, Ricci JA, et al. Lost Productive Time and Cost
    Due to Common Pain Conditions in the US Workforce.
    JAMA (2003)290:18, p. 2446.
    Source: Andrea M. Trescot, MD
How well did you know this?
1
Not at all
2
3
4
5
Perfectly
118
Q

2592.Which of the following statements about the CSA is
true?
A. If a practitioner needs only a small number of dosage
units of a controlled substance for offi ce use , a prescription
order may be issued to permit a pharmacy to
dispense them.
B. The Act prohibits treating a known addict with an
opioid/opiate product for persistent (as opposed to
acute) pain.
C. The defi nes the term “narcotic” as those controlled substances
which are opioids or opiates, whether natural,
synthetic, or semi-synthetic.
D. Cocaine is classifi ed by the Act as a C-I substance.
E. The Act permits DEA Diversion Investigators to inspect
your controlled substances records (inventories, storage
facility, order forms, etc.) at any time during your normal
practice hours.

A
  1. Answer: E
    Explanation:
    Only E is correct. A is expressly prohibited; B is not
    discussed in the Act at all, C is incorrect because “narcotic”
    is defi ned to include not only the opioids and opiates, but
    also cocaine. D is incorrect because medical cocaine is a
    C-II substance, and is often used as a topical anesthetic in
    sinus surgery.
    Source: Arthur Thexton
How well did you know this?
1
Not at all
2
3
4
5
Perfectly
119
Q

2593.Which of the following is true about the federal
Controlled Substances Act:
A. It prohibits self-prescribing.
B. It permits DEA Diversion Investigators to inspect patient
records without patient consent, upon written request.
C. It allows most practitioners to prescribe C-I substances
for IRB-approved research
D. It permits a “Do Not Fill Until” instruction on a prescription
order.
E. It prohibits prescribing methadone without a special
registration as a methadone clinic.

A
  1. Answer: D
    Explanation:
    A is not covered at all in the CSA, but is a matter of state
    law. B is incorrect because the CSA does not permit this;
    the HIPAA rule does this. C is incorrect, in that a special
    registration is required to order C-I substances for any
    purpose. And, E is incorrect because methadone may be
    prescribed for pain relief or for any other medically
    appropriate purpose, EXCEPT the treatment of addiction,
    without any special registration.
    Source: Arthur Thexton
How well did you know this?
1
Not at all
2
3
4
5
Perfectly
120
Q
2594. A meperidine (Demerol) dose equivalent to morphine 10
mg every 3 to 4 hours by injection is:
A. 100 mg every 3 hours
B. 75 mg every 3 hours
C. 50 mg every 3hours
D. 100 mg every 4 hours
E. 75 mg every 4 hours
A
  1. Answer: A

Source: Stimmel, B

How well did you know this?
1
Not at all
2
3
4
5
Perfectly
121
Q
  1. A 22 year old weight lifter who has been observed to have
    a gradual behavior change marked by aggression and
    mood swings may be demonstrating the effects of:
    A. Increased protein intake
    B. Creatine and chromium nutritional supplements.
    C. Prednisone tablets
    D. Anabolic steroids
    E. Massive doses of vitamins
A
  1. Answer: D

Source: Stimmel, B

How well did you know this?
1
Not at all
2
3
4
5
Perfectly
122
Q

2596.Identify the statement describing the withdrawal when
chronic opioid use is discontinued?
A. Is of no clinical signifi cance
B. Can be prevented by administering of a benzodiazepine
C. Can be prevented by administering of an amphetamine
D. Is best managed by slowly tapering the opioid dose by no
more than 10% every few days.
E. Can be alleviated by immediately starting on an opioid
agonist-antagonist

A
  1. Answer: D

Source: Stimmel, B

How well did you know this?
1
Not at all
2
3
4
5
Perfectly
123
Q

2597.While on a maintenance dose of methadone 80 mg per
day, a patient “shoots up” the heroin equivalent of 10 mg.
The most likely effect will be:
A. Euphoria and sedation
B. Sedation only.
C. Neither euphoria nor sedation
D. Lacrimation, piloerection, and abdominal cramps
E. Respiratory distress, chest pain, and tachycardia

A
  1. Answer: C

Source: Stimmel, B

How well did you know this?
1
Not at all
2
3
4
5
Perfectly
124
Q

2598.When stimulating a receptor:
A. Increased stimulation causes increased amplitude of the
action potential
B. Increased stimulation causes increased fi ring of the action
potential
C. Increased stimulation causes increased voltage of the
action potential
D. Increased stimulation causes increased recruitment of
other receptors

A
  1. Answer: B
    Explanation:
    Increased stimulation causes an increase in action
    potentials of the receptor, which is perceived as a stronger
    stimulus.
    Source: Andrea M. Trescot, MD
How well did you know this?
1
Not at all
2
3
4
5
Perfectly
125
Q

2599.A 23-year-old male presents with a history of smoking
“crack cocaine.” He is disoriented, confused, and in a
dissociative state. Physical examination reveals rotary
nystagmus. Pending the results of his urine drug
screen, you would be suspicious that his drug had been
adulterated with:
A. Methamphetamine.
B. LSD
C. Lidocaine
D. Phencyclidine
E. Ketamine.

A
  1. Answer: D

Source: Stimmel, B

How well did you know this?
1
Not at all
2
3
4
5
Perfectly
126
Q

2600.A patient of yours is brought in for evaluation by his
wife, 3 days after cervical epidural steroid injection with
multiple complaints. On examination he is found to
be disoriented, confused and in a fugue-like state with
nystagmus on upward gaze, tachycardia and elevated
blood pressure. The most likely diagnosis is:
A. Cocaine intoxication
B. Cannabis intoxication
C. Barbiturate intoxication
D. Phencyclidine (PCP) intoxication
E. Epidural abscess

A
  1. Answer: D

Source: Stimmel, B

How well did you know this?
1
Not at all
2
3
4
5
Perfectly
127
Q
2601.Death from acute intoxication with phencyclidine is
most likely to occur as a result of:
A. Violence
B. Cholinergic crisis.
C. Hypertensive crisis.
D. Cardiac arrest.
E. Status epilepticus.
A
  1. Answer: A

Source: Stimmel, B

How well did you know this?
1
Not at all
2
3
4
5
Perfectly
128
Q

2602.Supportive management (“talking down”) of most acute
reactions to lysergic acid diethylamide (LSD):
A. Should be accompanied by an injection of diazepam
(Valium).
B. Should be attempted in a hospital or emergency room
setting.
C. Is not indicated for these reactions.
D. Can usually be accomplished without medication or
hospitalization.
E. Should be accompanied by an injection of haloperidol
(Haldol).

A
  1. Answer: D

Source: Stimmel, B

How well did you know this?
1
Not at all
2
3
4
5
Perfectly
129
Q
2603.Methylenedioxyamphetamine (DMA) and its analog met
hylenedioxymethamphetamine (MDMA) are reported to
cause all of the following EXCEPT:
A. Mild stimulation
B. A feeling of well-being.
C. Visual illusions or hallucinations.
D. Auditory hallucinations.
E. Anxiety
A
  1. Answer: D

Source: Stimmel, B

How well did you know this?
1
Not at all
2
3
4
5
Perfectly
130
Q

2604.Flashbacks can occur:
A. Long after the hallucinogenic intoxication has dissipated.
B. Only in patients with pre-existing psychological problems.
C. As a result of impurities in street psychedelics.
D. Usually a half hour after the drug has been ingested.
E. Only in patients with combined use of hallucinogens and
opioids

A
  1. Answer: A

Source: Stimmel, B

How well did you know this?
1
Not at all
2
3
4
5
Perfectly
131
Q
  1. Choose true statements concerning infective endocarditis
    in heroin addictions, compared to endocarditis in nonaddicted
    patients:
    A. The tricuspid valve is affected more often.
    B. Mixed fl ora of bacteria and fungi are more prevalent.
    C. Staphylococcus aureus is found more often as a causative
    organism.
    D. Surgical treatment is rarely necessary.
    E. Easily identifi ed by an aortic murmur
A
  1. Answer: A

Source: Stimmel, B

How well did you know this?
1
Not at all
2
3
4
5
Perfectly
132
Q

2606.The states of a patient in which repetitions of the same
dose of a drug has progressively less effect or in which the
dose needs to be increased to obtain the same degree of
pharmacological effect as was caused by the original dose
is defi ned as:
A. Physical dependence
B. Synergistic effect
C. Additive effects
D. Disuse supersensitivity
E. Tolerance

A
  1. Answer: E

Source: Stimmel, B

How well did you know this?
1
Not at all
2
3
4
5
Perfectly
133
Q

2607.Choose the correct statements describing the risks of malprescribing
A. Never given re-education options
B. Not reportable to databank
C. State Board Investigation or Sanction
D. Usually results in publicity to increase your practice
E. Not liable to civil lawsuits, only criminal liability

A
2607. Answer: C
Explanation:
Risks of Malprescribing
State Board Investigation or Sanction
Often given re-education options
May result in databank report
Usually results in adverse publicity
Attention attracts civil lawsuits
Source: Laxmaiah Manchikanti, MD
How well did you know this?
1
Not at all
2
3
4
5
Perfectly
134
Q
2608.Methadone metabolism may be signifi cantly altered by the co-administration of all of the following EXCEPT:
A. Ciprofl oxin
B. Phenytoin
C. Oxycodone
D. Biaxin
E. Rifampin
A
  1. Answer: D
    Explanation:
    Ciprofl oxin inhibits CYP3A4 and may result in withdrawal. Phenytoin lowers methadone concentration
    by about half in 3-4 days. Rifampin may cause withdrawal
    by interference with CYP3A4. Oxycodone metabolism is
    inhibited by methadone and the combination may
    increase oxycodone levels or effects.
    Reference: Drugs for Pain, Smith 2003, Page 459
    Source: Art Jordan, MD, Sep 2005
How well did you know this?
1
Not at all
2
3
4
5
Perfectly
135
Q

2609.Federal Control over the use of controlled substances
comes from which one of the following:
A. Balanced budget act of 1997
B. Food and drug Cosmetic Act administered by the F.D.A.
C. Judicial branch of the government
D. Executive branch of the government
E. National Narcotics Bureau (N.N.B.)

A
2609. Answer: C
Explanation:
Pain Medicine: A Comprehensie Review, Second Edition;
P. Prithvi Raj: Mosby, Page 390
Source: Art Jordan, MD, Sep 2005
How well did you know this?
1
Not at all
2
3
4
5
Perfectly
136
Q
2610.Which one of the following is the most frequently abused
opiate in the United States?
A. Morphine
B. Toradol
C. Hydrocodone
D. Oxycodone
E. Fentanyl lollypops
A
  1. Answer: C
    Explanation:
    D.A.W.N. Drug Abuse Warning Network Reports
    Hydrocodone is a Schedule II controlled substance only
    when prescribed alone. With acetaminophen, it is a
    Schedule III. It has an average serum half-life of 3.8 hours.
    It is widely used as an antitussive.
    Disposition of Toxic Drugs and Chemicals in man. Fifth
    Edition. Randall C. Baselt 2000
    Drug Enforcement Administration website
    Source: Art Jordan, MD, Sep 2005
How well did you know this?
1
Not at all
2
3
4
5
Perfectly
137
Q
2611.How often must a practitioner renew his/her DEA
registration?
A. Every 2 years.
B. Every 6 years.
C. Every 3 years.
D. Every 4 years.
E. Every 5 years.
A
  1. Answer: C
    Explanation:
    Reference: 21 U.S.C. § 823; 21 C.F.R. § 1301.01 through
    1301.55.
    Explanation:
    DEA requires practitioners to renew their DEA
    registration number every 3 years. This may be done online
    through www.deadiversion.usdoj.gov. Moreover,
    DEA requires practitioners to request modifi cations of
    their DEA registration numbers when they move to a new
    medical practice or open another offi ce requiring
    additional registration.
    Source: Jennifer Bolen, JD, Sep 2005
How well did you know this?
1
Not at all
2
3
4
5
Perfectly
138
Q

2612.All of the following statements about Food and Drug Act
are correct, EXCEPT:
A. 1906 - Wiley Act was founded the F. D. A., and centered
on foods and meat packing
B. 1938 - Food, Drugs, and Cosmetics Act mandated premarket
approval of drugs
C. Marijuana Tax Act 1938
D. 1932 - Food, Drugs, and Cosmetics Act required proof
safety
E. 1906 - Wiley Act required concentration standards for all
medications

A
2612. Answer: D
Explanation:
Food and Drug Acts
* 1906 - Wiley Act
- Founded the F. D. A.
- Centered on foods and meat packing
- Required ingredients and concentration standards for
all medications - largely to regulate patent medicines.
* 1938 - Food, Drugs, and Cosmetics Act
- Mandated premarket approval of drugs
- Required proof of safety
- Prohibited false therapeutic claims
* Marijuana Tax Act 1938
Source: Roger Cicala, MD, Sep 2005
How well did you know this?
1
Not at all
2
3
4
5
Perfectly
139
Q

2613.As a DEA registrant you have certain responsibilities,
including (1) proper registration and renewal; (2)
proper record-keeping; and (3) what newly explained
responsibility as stated in the Interim Policy Statement,
published by DEA in the Federal Register on November
16, 2004?
A. A responsibility to report to DEA about the misuse of a
DEA number.
B. A responsibility to seriously consider any sincerely
expressed concerns made by family members about a
patient’s potential abuse of controlled substances.
C. A responsibility to see all patients every thirty days.
D. A responsibility to issue drugs for a legitimate medical
purpose.
E. A responsibility to review patient records every thirty
days.

A
  1. Answer: B
    Explanation:
    Reference: DEA Interim Policy Statement, Fed. Reg. Vol.
    69, No. 220, pp. 67170-67172 (Nov. 16, 2004).
    Explanation:
    A. This is not the correct answer. While a physician must
    report to DEA if their DEA number has been
    compromised, the Interim Policy Statement does not
    discuss this duty.
    B. This is the best answer. The Interim Policy Statement
    states: “[g]iven the addictive and sometimes deadly nature
    of prescription narcotic abuse, the tremendous volume of
    such drug abuse in the United States, and the propensity
    of many drug addicts to attempt to deceive physicians in
    order to obtain controlled substances for the purpose of
    abuse, a physician should seriously consider any sincerely
    expressed concerns about drug abuse conveyed by family
    members and friends.”
    C. This is not the best answer because the Interim Policy
    Statement did not assign a time frame within which to see
    patients and does not say that you must see your patients
    every thirty days.
    D. This is not the correct answer. To be valid, a physician
    must issue a prescription for controlled substances for (1)
    a legitimate medical purpose, and (2) within the usual
    course of professional practice. This has been the law of
    the Controlled Substances Act of 1970 and was not a new
    directive from the Interim Policy Statement.
    E. This is not the correct answer. The Interim Policy
    Statement did not assign a time frame within which a
    physician must review patient records.
    Source: Jennifer Bolen, JD, Sep 2005
How well did you know this?
1
Not at all
2
3
4
5
Perfectly
140
Q
2614.Patients usually develop tolerance to all opioid effects
EXCEPT:
A. Sedation
B. Pruritus
C. Constipation
D. Pain relief
E. Respiratory depression
A
  1. Answer: C
    Explanation:
    Sedation and pruritus (due to direct histamine release)
    abate over time. Although tolerance to pain relief can
    occur, with long acting narcotics (especially methadone) it
    is less likely. Constipation, however, should be expected to
    be a problem for the entire length of treatment.
    Source: Trescot AM, Board Review 2004
How well did you know this?
1
Not at all
2
3
4
5
Perfectly
141
Q

2615.The Controlled Substances Act of 1970 does not give DEA
the authority to do which of the following:
A. Scheduled drugs
B. Regulate medical practice
C. Administer the CSA and create policy related to the
CSA
D. Establish quotas for the manufacture of controlled substances
E. Reschedule drugs depending on their potential for
abuse

A
  1. Answer: B
    Explanation:
    Reference: The Controlled Substances Act of 1970, codifi ed
    at 21 U.S.C. § 801 and sections that follow; see also 1994,
    Joranson DE, Gilson AM. Chapter 8 - Controlled
    substances, medical practice and the law. In: Schwartz HI.
    Psychiatric Practice Under Fire: The Infl uence of
    Government, the Media and Special Interests on Somatic
    Therapies. Washington, DC: American Psychiatric Press,
    Inc., 1994:173-194. Explanation:
    A. This is not the correct answer. The Controlled
    Substances Act of 1970 does give DEA the authority to
    schedule drugs according to whether the drug has been
    approved by the Food and Drug Administration for
    medical use and according to the drug’s potential for
    abuse.
    B. This is the correct answer. The states, not the federal
    government, have the authority to regulate medical
    practice. State authority derives from both federal and
    state constitutions. States create medical practice acts to
    regulate the practice of medicine and protect the public.
    The CSA does not give DEA the authority to regulate
    medical decisions and it does not permit DEA to change
    or
    limit indications for which a drug may be prescribed.
    Likewise, the CSA does not give DEA the authority to
    regulate the quantity or chronicity of prescribing relative
    to controlled substances.
    C. This is not the correct answer because the Controlled
    Substances Act of 1970 does give DEA authority to
    administer the CSA and create policy related to the various
    provisions of the CSA.
    D. This is not the correct answer because the Controlled
    Substances Act of 1970 does give DEA authority to
    establish manufacturing quotas that drug manufacturers
    must follow when producing controlled substances. There
    are many facets to DEA’s authority to establish
    manufacturing quotes and the important point is that
    DEA must ensure that there are suffi cient controlled drugs
    available to meet legitimate health care demands.
    E. This is not the correct answer because the Controlled
    Substances Act of 1970 does give DEA authority to
    reschedule drugs into higher or lower schedules,
    depending on whether a drug remains approved for
    medical use and depending on developments and trends
    regarding the trends for the abuse of these drugs.
    Source: Jennifer Bolen, JD, Sep 2005
How well did you know this?
1
Not at all
2
3
4
5
Perfectly
142
Q

2616.You treat patients who suffer from conditions producing
chronic, non-malignant pain. You prescribe controlled
substances to your patients (1) for a legitimate medical
purpose, and (2) within the usual course of professional
practice. The Federation of State Medical Boards and
many state licensing boards require practitioners to keep
medical records that include which of the following items
in connection with their use of controlled substances to
treat pain?
A. The name of the drug and the amount prescribed.
B. The medical history and physical examination, diagnostic,
therapeutic and laboratory results, evaluations
and consultations, treatment objectives, discussion of
risks and benefi ts, informed consent and treatment
agreements, treatments, medications (including date,
type, dosage and quantity prescribed), instructions and
agreements, and periodic reviews.
C. An inventory of all the drugs prescribed to each patient,
a record of all communications with the patient, and all
office forms.
D. A carbon copy of the prescriptions issued, all contact
information for the patient, and all clinical rationale for
the drugs prescribed.
E. A list of each office visit you have with the patient and a
statement of all treatments rendered.

A
  1. Answer: B
    Explanation:
    Reference: The Federation of State Medical Boards’ May
    2004 Model Policy for the Use of Controlled Substances
    for the Treatment of Pain, www.fsmb.org.
    Explanation:
    A. This is not the best answer because it only involves
    some of the records state licensing boards require
    practitioners to keep when they prescribe controlled
    substances for the treatment of pain.
    B. This is the best answer. The Federation of State Medical
    Boards’ Model Policy for the Use of Controlled Substances
    for the Treatment of Pain sets forth the following
    categories of medical records that practitioners should
    keep when they treat pain. Many states have adopted these
    categories in prescribing guidelines and, quite frequently,
    in prescribing regulations or rules.
  2. the medical history and physical examination,
  3. diagnostic, therapeutic and laboratory results,
  4. evaluations and consultations,
  5. treatment objectives,
  6. discussion of risks and benefi ts,
  7. informed consent,
  8. treatments,
  9. medications (including date, type, dosage and
    quantity prescribed),
  10. instructions and agreements and
  11. periodic reviews.
    C. This is not the best answer because it only involves
    some of the records state licensing boards require
    practitioners to keep when they prescribe controlled
    substances for the treatment of pain.
    D. This is not the best answer because it only involves
    some of the records state licensing boards require
    practitioners to keep when they prescribe controlled
    substances for the treatment of pain.
    E. This is not the best answer because it only involves
    some of the records state licensing boards require
    practitioners to keep when they prescribe controlled
    substances for the treatment of pain.
    Source: Jennifer Bolen, JD, Sep 2005
How well did you know this?
1
Not at all
2
3
4
5
Perfectly
143
Q
2617.Use of which the following opioids by breast-feeding
mothers depresses the behavior of the infant more than
the equianalgesic dose of morphine:
A. Fentanyl®
B. Meperidine®
C. Nalbuphin®
D. Buprenorphine®
E. Tramadol®
A
  1. Answer: B

Source: Raj, Pain Review 2nd Edition

How well did you know this?
1
Not at all
2
3
4
5
Perfectly
144
Q

2618.Which of the following is NOT true about
benzodiazepines when used long term in chronic pain?
A. Decrease REM and slow wave sleep
B. Decrease serotonin levels
C. May produce a dangerous withdrawal syndrome if suddenly
discontinued
D. Have a primary analgesic effect
E. May have a much higher rate of cognitive dysfunction
than opioids

A
  1. Answer: D
    Explanation:
    Benzodiazepines decrease REM and Stage and Stage 4
    sleep, resulting in increased fatigue and sleep deprivation.
    Some studies have shown a cognitive dysfunction rate as
    high as 70% in patients on long term benzodiazepines.
    Withdrawal from benzodiazepines may be worse than
    opioids. There is no evidence that benzodiazepines have a
    primary analgesic effect.
    Source: Art Jordan, MD, Sep 2005
How well did you know this?
1
Not at all
2
3
4
5
Perfectly
145
Q
2619.Convulsions caused by drug poisoning are most
commonly associated with
A. Phenobarbital
B. Diazepam
C. Strychnine
D. Chlorpromazine
E. Phenytoin
A
  1. Answer: C

Reference: Hardman, pp 89-90.
Strychnine acts as a competitive antagonist of glycine, the
predominant postsynaptic inhibitory transmitter in the
brain and spinal cord. The fatal adult dose is 50 to 100mg.
Persons poisoned by strychnine suffer convulsions that
progress to full tetanic convulsions. Because the
diaphragm and thoracic muscles are fully contracted, the
patient cannot breathe. Hypoxia eventually causes
medullary parasysis and death. Control of the convulsions
and respiratory support are the immediate objectives of
the therapy. Diazepam may be preferred to a barbiturate in
controlling the convulsions because it offers less
concomitant respiratory depression. Poisoning caused by
the other drugs listed in the question is not associated with
convulsions but with depression of the CNS.
Source: Stern - 2004

How well did you know this?
1
Not at all
2
3
4
5
Perfectly
146
Q
2620.Disadvantages of long-term morphine therapy in a
cancer patient are:
A. Withdrawal when drug is stopped
B. Cognitive dysfunction
C. Addiction
D. Liver dysfunction
E. Constipation
A
  1. Answer: E
How well did you know this?
1
Not at all
2
3
4
5
Perfectly
147
Q
2621.Under the Controlled Substances Act (CSA), the
following drug schedules is thought to have the highest
potential for abuse?
A. Schedule II
B. Schedule III
C. Schedule V
D. Schedule I
E. Schedule IV
A
  1. Answer: D
    Explanation:
    A. Schedule II drugs have a high potential for abuse and a
    high rate of psychological or physical dependence.
    Narcotic analgesics are schedule II drugs.
    Non-narcotic drugs such as amphetamines,
    methylphenidate, and pentobarbital are also schedule II.
    B. Schedule III drugs have a potential for abuse less than
    the drugs or other substances in schedules I and II. Abuse
    of the drug or other substance may lead to moderate or
    low physical dependence or high psychological
    dependence.
    C. Schedule V drugs have the lowest potential for abuse of
    the drugs under the jurisdiction of the Controlled
    Substances Act. Antitussives and antidiarrheal
    preparations that contain narcotics in limited quantities
    are schedule V drugs.
    D. Schedule I is reserved for the most dangerous drugs
    without recognized medical value.
    E. Schedule IV drugs are thought to have less potential for
    abuse than schedule I, II, and III drugs. Examples of
    schedule IV drugs are benzodiazepines, phenobarbital,
    meprobamate, chloral hydrate, and dextropropoxyphene
    (Darvon).
How well did you know this?
1
Not at all
2
3
4
5
Perfectly
148
Q

2622.Cocaine, produced from the leaves of Erythroxylon
species,
A. Produces bradycardia and vasodilation
B. Is directly related chemically to opioid analgesics
C. Is metabolized by the microsomal metabolizing system
D. Blocks nerve conduction effectively
E. Blocks norepinephrine receptors directly

A
  1. Answer: D
    Explanation:
    Reference: Hardman, pp 338, 570.
    A. Peripherally, cocaine produces sympathomimetic
    effects including tachycardia and vasoconstriction.
    B. Cocaine is an ester of benzoic acid and is closely related
    to the structure of atropine. Death from acute overdose
    can be from respiratory depression or cardiac failure.
    C. Cocaine is biotransformed by plasma esterases to
    inactive products.
    D. Cocaine has local anesthetic properties; it can block the
    initiation or conduction of a nerve impulse.
    E. Cocaine also blocks the reuptake of norepinephrine.
    This action produces CNS stimulant effects including
    euphoria, excitement, and restlessness.
    Source: Stern - 2004
How well did you know this?
1
Not at all
2
3
4
5
Perfectly
149
Q

2623.When prescribing sublingual or buccal route of
administration, the best drug (55%) for absorption is
A. Methadone
B. Fentanyl
C. Morphine
D. Buprenorphine
E. Demerol

A
  1. Answer: D
    Source: Raj P, Pain medicine - A comprehensive Review -
    Second Edition
How well did you know this?
1
Not at all
2
3
4
5
Perfectly
150
Q
  1. In order for a prescription to be valid under federal and
    state law, it must be issued
    A. With the proper date and the physician’s signature.
    B. With the proper date, patient information, drug identification and instructions for use, physician signature,
    and it must be issued for a legitimate medical purpose
    within the usual course of professional practice.
    C. To a patient who does not have a criminal history.
    D. Only by a licensed physician and not by any mid-level
    practitioner.
    E. Within three days of seeing the patient.
A
  1. Answer: B
    Explanation:
    Reference: Code of Federal Regulations, 21 C.F.R. §
    1306.04 (Prescriptions).
    Explanation:
    A. This is not the best answer. Prescriptions must meet
    both substantive and technical requirements to be valid. As
    a technical matter, a prescription must be dated and
    signed
    on the date it is actually issued to the patient or guardian.
    The prescription must contain not only the date and the
    properly licensed and registered provider’s signature, but
    also information about the patient and the drug
    prescribed. However, date and signature alone are not
    enough to make a prescription valid and the prescription
    must meet the substantive requirements of (1) legitimate
    medical purpose, and (2) usual course of professional
    practice to be wholly valid.
    B. This is the best answer. To be valid, the federal and state
    laws and regulations require a prescription be issued for a
    legitimate medical purpose within the usual course of
    professional practice. State licensing boards often add to
    this requirement by requiring physicians and other health
    care practitioners to document the medical record to show
    compliance with applicable laws and regulations
    governing controlled substance prescribing.
    C. This is not the correct answer because federal and state
    laws do not prohibit a physician from prescribing
    controlled substance to a person with a criminal history.
    The only caveat to this statement concerns the patient’s
    history of substance abuse, as it may call for restrictive
    monitoring on the patient’s use of controlled substances or
    other restrictions to ensure that the controlled substances
    prescribed get used as they are intended – for pain relief.
    D. This is not the correct answer because most states allow
    physicians to use mid-level practitioners in the treatment
    of patients. These working relationships allow for
    practitioners to see more patients on a given day. While some state laws may require the physician to sign all
    prescriptions, in many cases mid-level practitioners also
    have DEA registrations and thus may prescribe controlled
    substances for the treatment of pain. Anyone who signs a
    controlled substances prescription must be licensed
    properly by state authorities and registered with the DEA.
    E. This is not the correct answer. Federal and state laws do
    not require physicians to issue controlled substances
    prescriptions within three days of seeing the patient.
    Source: Jennifer Bolen, JD, Sep 2005
How well did you know this?
1
Not at all
2
3
4
5
Perfectly
151
Q
  1. You are a solo practitioner in a pain specialist capacity.
    You have a patient who wants to be treated in your offi ce
    for opioid addiction and pain. Do you need a separate
    registration to provide Narcotic Treatment Services?
    A. No, I can issue 72-hour emergency prescriptions as
    needed to detoxify a patient.
    B. No, I can prescribe methadone to help them detoxify
    because I prescribe methadone to treat their pain.
    C. No, I have the proper training on the use of Schedule
    III-V drugs in the offi ce based treatment of opioid addiction.
    D. Yes, I must have a separate registration to provide narcotic
    treatment services.
    E. Yes, so I can prescribe drugs in Schedule II-V to detoxify
    a patient due to opioid addiction.
A
  1. Answer: D
    Explanation:
    Reference: 21 U.S.C. § 823; 21 C.F.R. 1306.07; and
    www.deadiversion.usdoj.gov.
    Explanation:
    A. This is not the correct answer because the 72-hour
    exception to the federal law requirement of a separate
    registration for detoxifi cation or maintenance treatment
    only allows a practitioner to administer or dispense (but
    not prescribe) (1) one day’s worth of emergency
    medication to the patient at one time, (2) for not more
    than a total of 72-hours, and (3) the practitioner may not
    extend or renew the 72-hour period. This 72-hour
    exception is known as the “three day rule” and it is found
    in 21 C.F.R. § 1306.07(b). Thus, if a practitioner is not
    separately registered as a narcotic treatment program,
    he/she may administer BUT NOT prescribe narcotic drugs
    to a patient for the purpose of relieving acute withdrawal
    symptoms while arranging for the patient’s referral for
    treatment. Congress intended § 1306.07(b) to give
    practitioners fl exibility in emergency situations when
    confronted with a patient undergoing withdrawal. Thus,
    Congress established this exception to “augment, not to
    circumvent” the separate registration requirement set
    forth in the CSA.
    B. This is not the correct answer because no one is legally
    permitted to prescribe methadone to detoxify or maintain
    a patient for addiction.
    C. This is not the correct answer because having only the
    training on the use of Schedule III-V controlled substances
    to perform the offi ce-based treatment of opioid addiction
    is not enough. Practitioners must obtain an “X”
    certifi cation from DEA to use Schedule III-V controlled
    substances to treat patients in their offi ces for opioid
    addiction. Also, you may administer or dispense, but not
    prescribe, methadone according to the “three day rule”
    described above in answer A.
    D. This is the best answer. If a practitioner wants to use
    Schedule II narcotic drugs for maintenance and/or
    detoxifi cation, federal and state law require the
    practitioner to obtain separate registration from the DEA
    as a narcotic treatment program pursuant to the Narcotic
    Addict Treatment Act of 1974. Signifi cantly, this
    registration allows a practitioner to administer or
    dispense, but not prescribe, scheduled narcotic drugs that
    are approved by the United States Food and Drug
    Administration (FDA) for the treatment of narcotic
    addiction. Until October 2000, methadone and levo-alphaacetyl-
    methadol (LAAM) were the only scheduled
    narcotics approved by FDA for use in maintenance and
    detoxifi cation treatment. In October 2000, pursuant to the
    Drug Abuse Treatment Act of 2000 (DATA 2000),
    Congress amended the CSA (21 U.S.C. § 823(g)) to allow
    properly registered and trained practitioners to dispense
    or prescribe Schedule III-V controlled substances
    specifi cally approved by the FDA for the offi ce-based
    treatment of opioid addiction.
    E. This is not the correct answer. While a separate
    registration is required to render narcotic treatment
    services, no DEA registration to do so allows any
    practitioner to prescribe Schedule II controlled substances
    to detoxify or maintain an individual with an addiction. As
    stated above in answer D, those properly registered and
    trained may prescribe those drugs in Schedules III-V
    approved by FDA for the purpose of detoxifi cation from
    opioids.
    Source: Jennifer Bolen, JD, Sep 2005
How well did you know this?
1
Not at all
2
3
4
5
Perfectly
152
Q
  1. Consequences of undertreatment of pain may include all
    of the following EXCEPT:
    A. Possible jail time
    B. Civil lawsuits
    C. Loss or restriction of prescribing abilities
    D. No effect of medical license
    E. Exclusion from Medicare/Medicaid
A
2626. Answer: D
Explanation:
* Possible jail time
* Civil Lawsuits
* Loss or restriction of prescribing abilities
* Loss or restriction of medical license
* Exclusion from Medicare/Medicaid
* Loss of patients
* Loss of your practice
Source: Erin Brisbay McMahon, JD, Sep 2005
How well did you know this?
1
Not at all
2
3
4
5
Perfectly
153
Q
  1. In performing urine drug testing, a physician must know
    all of the following EXCEPT:
    A. The characteristics of testing procedures, since many
    drugs are not routinely detected by all UDTs.
    B. Although no aberrant behavior is pathognomonic of
    abuse or addiction, such behavior should never be
    ignored.
    C. Reliance on aberrant behavior to trigger a UDT will miss
    more than 50% of those individuals using unprescribed
    or illicit drugs.
    D. Always prescribe “on-demand” for the patient until you
    are comfortable with the situation.
    E. A history of drug abuse does not preclude treatment
    with a controlled substance, when indicated, but does
    require a treatment plan with firmly defined boundaries.
A
  1. Answer: D
    Explanation:
    Urine Drug Testing
    Know the characteristics of testing procedures, since many
    drugs are not routinely detected by all UDTs.
    Although no aberrant behavior is pathognomonic of
    abuse or addiction, such behavior should never be ignored.
    Reliance on aberrant behavior to trigger a UDT will miss
    more than 50% of those individuals using unprescribed or
    illicit drugs.
    Never prescribe “on-demand” for the patient until you are
    comfortable with the situation. A history of drug abuse does not preclude treatment with
    a controlled substance, when indicated, but does require a
    treatment plan with fi rmly defi ned boundaries.
    Source: Laxmaiah Manchikanti, MD
How well did you know this?
1
Not at all
2
3
4
5
Perfectly
154
Q
  1. All of the following are signs of controlled substance in
    work place, EXCEPT:
    A. Progressive deterioration in personal appearance and
    hygiene
    B. Uncharacteristic deterioration of handwriting and charting
    C. Wearing long sleeves when inappropriate
    D. Personality change - mood swings, anxiety, depression,
    lack of impulse control, suicidal thoughts or gestures
    E. Increased personal and professional activities
A
  1. Answer: E
    Explanation:
    Watch for signs:
    Progressive deterioration in personal appearance and
    hygiene;
    Uncharacteristic deterioration of handwriting and
    charting;
    Wearing long sleeves when inappropriate;
    Personality change - mood swings, anxiety, depression,
    lack of impulse control, suicidal thoughts or gestures;
    Patient and staff complaints about health care provider’s
    changing attitude/behavior;
    Increasing personal and professional isolation.
    Source: Erin Brisbay McMahon, JD, Sep 2005
How well did you know this?
1
Not at all
2
3
4
5
Perfectly
155
Q
  1. The management of phencyclidine toxicity is likely to
    include all of the following except:
    A. Naloxone for respiratory depression
    B. Diazepam for seizures
    C. Propranolol for adrenergic crisis
    D. Haloperidol for disorganized, disruptive behavior
    E. Low-stimulus environment
A
  1. Answer: A
    Explanation:
    Phencyclidine (“angel dust,” “crystal,” “hog”) toxicity
    induces organize mental disorders, intoxication, delirium,
    delusional mood, and fl ashback disorders with physical
    problems related to high blood pressure, muscle rigidity,
    ataxia, coma, nystagmus (particularly vertical), and dilated
    pupils. Treatment is with IV diazepam as the drug of fi rst
    choice. Propranolol can be used for an adrenergic crisis,
    and haloperidol is effective for psychotic and disruptive
    behavior. Elimination of the drugs is enhanced by
    ammonium chloride in the acute stage and later by
    ascorbic acid. Environmental stimuli should be kept to a
    minimum. The urine is positive for PCP up to seven days,
    but there can be false negatives.
    Source: Psychiatry specialty Board Review By William M.
    Easson, MD and Nicholas L. Rock, MD
How well did you know this?
1
Not at all
2
3
4
5
Perfectly
156
Q
  1. Which of the following is associated with crack (the freebase
    form of cocaine)?
    A. Flashbacks (recurrences of effects) may occur months
    after the last use of the drug
    B. It may cause seizures and cardiac arrhythmias
    C. It acts by blocking adrenergic receptors
    D. It is the salt form of cocaine
    E. It is primarily administered intranasally
A
  1. Answer: B
    Explanation:
    Reference: Katzung, p 538.
    A. Flashbacks can occur with use of LSD and mescaline
    but have not been associated with the use of cocaine.
    B. Use of crack cocaine has led to seizures and cardiac
    arrhythmias.
    C. Some of cocaine’s effects (sympathomimetic) are due to
    blockade of norepinephrine reuptake into presynaptic
    terminals; it does not block receptors.
    D. Crack is the
    free-base (nonsalt) form of the alkaloid cocaine. It is called
    crack because, when heated, it makes a crackling sound.
    E.Heating crack enables a person to smoke it; the drug is
    readily absorbed through the lungs and produces an
    intense euphoric effect in seconds.
    Source: Stern - 2004
How well did you know this?
1
Not at all
2
3
4
5
Perfectly
157
Q

2631.What are some of the signs of abuse of controlled
substances by your offi ce workers and co-workers?
A. No extra time is spent near a drug supply
B. Extreme reliability in keep appointments and meeting
deadlines
C. Never volunteer for overtime
D. Only at work when scheduled
E. Absenteeism, frequent disappearances or long unexplained
absences, making improbable excuses and
taking frequent or long trips to the bathroom or to the
stockroom where drugs are kept

A
  1. Answer: E
    Explanation:
    Watch for signs:
    Absenteeism, frequent disappearances or long unexplained
    absences, making improbable excuses and taking frequent
    or long trips to the bathroom or to the stockroom where
    drugs are kept;
    Excessive amounts of time spent near a drug supply.
    Volunteer for overtime and at work when not scheduled;
    Unreliability in keeping appointments and meeting
    deadlines;_
    Source: Erin Brisbay McMahon, JD, Sep 2005
How well did you know this?
1
Not at all
2
3
4
5
Perfectly
158
Q

2632.If you move the location of your practice, you are
required to do what regarding your DEA registration?
A. Send a request for modifi cation of registration in writing
to the nearest DEA fi eld offi ce, and obtain approval for
the modifi ed registration prior to the move.
B. Wait until the next renewal date for your controlled substances
registration to notify DEA of the move.
C. Tell only your state drug bureau about the move and
seek a new registration with them that you can use
with DEA.
D. Send DEA a request for a new registration number after
you move.
E. Wait until DEA asks you for updated information about
your registration and current address.

A
  1. Answer: A
    Explanation:
    Reference: 21 U.S.C. § 823; 21 C.F.R. § 1301.11 (separate
    registrations) and § 1301.51 (modifi cation of
    registration); www.deadiversion.usdoj.gov.
    Explanation:
    A. This answer is the best answer. If you move your
    principal registered location, you must send DEA a request
    for modifi cation of registration in writing and obtain
    approval for the modifi ed registration prior to the move.
    This request for modifi cation should be addressed to the
    DEA fi eld offi ce nearest to your currently registered
    location. You may obtain an Address Change Request from
    DEA’s internet site at www.deadiversion.usdoj.gov. You
    must manually sign and fax or mail it to the local DEA
    offi ce. The form must include (1) a copy of your current
    state medical/professional license for the new address
    along with a copy of the practitioner’s corrected state
    controlled substance registration, if applicable.
    B. This is not the correct answer. You may not wait until
    the next renewal date for your controlled substances
    registration to notify DEA of the move. If you do so, you
    may lose your registration number and you will be
    considered in violation of the law.
    C. This is not the best answer. You must fi rst get
    permission from DEA to modify your registration, and
    thus simply telling your state drug bureau is not suffi cient.
    You should learn what your state requires from you if you
    move principal locations from which you administer,
    dispense, and prescribe controlled substances
    D. This is not the correct answer. You may not wait until
    after you move to send DEA a request for a new registration number. Because you are administering and/
    or dispensing controlled substances from your currently
    registered location, you must fi rst obtain DEA’s approval
    to use the new location to store controlled substances.
    E. This is not the correct answer. You may not wait until
    DEA asks you for updated information about your
    registration and current address. DEA puts the burden on
    you, the registrant, to stay current with the paperwork
    surrounding the registration of individuals and locations.
    Source: Jennifer Bolen, JD, Sep 2005
How well did you know this?
1
Not at all
2
3
4
5
Perfectly
159
Q

2633.Which of the following statements is accurate with
administration of psychostimulants in opioid induced
sedation?
A. They are safe in patients with a history of paranoid
disorders
B. They are safe in patients with a history of hypertension
C. They should be given morning and noon so as not to
disturb sleep
D. They are safe in patients with a history of delirium
E. They should be administered only at bedtime

A
  1. Answer: C
    Explanation:
    They are contraindicated in patients with a history of
    hallucinations, delirium, or paranoid disorders. They are
    relatively contraindicated with a history of substance
    abuse or hypertension. Their use may exacerbate the
    above.
    Examples include methylphenidate, destroamphetamine,
    and pemoline.
    Reference: Melzack and Wall 2003, page 390
    Source: Art Jordan, MD, Sep 2005
How well did you know this?
1
Not at all
2
3
4
5
Perfectly
160
Q
  1. You have a patient that you suspect may be altering
    your prescriptions. You want to fax a copy of a Schedule
    II prescription to the pharmacist the patient uses to fi ll
    prescriptions. Is this legal and what must happen before
    the pharmacist dispenses the prescription?
    A. No, it is not legal to fax a Schedule II prescription to a
    pharmacist.
    B. Yes, but only if you have a HIPAA consent from the
    patient.
    C. Yes, and the patient must present the original prescription
    to the pharmacist.
    D. No, because faxes apply only to emergency prescriptions
    for Schedule II drugs.
    E. No, because faxes apply only to hospice and nursing
    home situations.
A
  1. Answer: C
    Explanation:
    Reference: 21 C.F.R. § 1306.11(a) (Requirement of a
    Prescription), which states “[a] prescription for a
    Schedule II controlled substance may be transmitted by
    the practitioner or the practitioner’s agent to a pharmacy
    via facsimile equipment, provided that the original
    written, signed prescription is presented to the pharmacist
    for review prior to the actual dispensing of the
    controlled substance, except as noted in paragraph (e), (f),
    or (g) of this section. The original prescription shall be
    maintained in accordance with § 1304.04(h) of the CSA.”
    Explanation:
    A. This is not the best answer because it states only one of
    the two requirements concerning the faxing of
    prescriptions. While it is legal to fax a Schedule II
    prescription to a pharmacist, the pharmacist may not
    dispense the prescription to the patient without the
    original prescription.
    B. This is not the correct answer because HIPAA consent
    has nothing to do with this law regarding the faxing of
    Schedule II controlled substances prescriptions to
    pharmacists. If you want to discuss the patient’s
    prescription with the pharmacist in connection with your
    treatment of the patient, you may do so and HIPAA does
    not require the patient’s consent for such conversations.
    C. This is the best answer because it is legal to fax a
    Schedule II prescription to a pharmacist and the patient
    must present the original prescription to the pharmacist
    prior to dispensation of the prescription. This measure
    would prove helpful in determining whether the patient in
    question has attempted to alter your prescriptions. Do not
    tell the patient that you are faxing a copy of the
    prescription to the pharmacist.
    D. This is not the correct answer because faxes are not
    limited to emergency prescriptions.
    E. This is not the correct answer because faxes are not
    limited to hospice and nursing home situations. However,
    it is important to note that the requirement of the original
    prescription does not apply to hospice and nursing home
    situations.
    Source: Jennifer Bolen, JD, Sep 2005
How well did you know this?
1
Not at all
2
3
4
5
Perfectly
161
Q
2635. Which controlled substance produces a toxic metabolite
which may cause seizures with accumulation with
repeated dosing?
A. Pentazocine
B. Propoxyphene
C. Morphine
D. Ketamine
E. Meperidine
A
  1. Answer: E
    Explanation:
    A major metabolite of meperidine is to normeperidine.
    Accumulation of normeperidine with repeated doses may
    cause seizures.
    Meperidine should only be used for short term acute pain
    management, if at all.
    Pentazocine, morphine, and ketamine are not associated
    with toxic metabolites at normal repeated dosing.
    Reference: Melzack and Wall 2003, Page 381
    Source: Art Jordan, MD, Sep 2005
How well did you know this?
1
Not at all
2
3
4
5
Perfectly
162
Q

2636.Every practitioner who administers, prescribes, or
dispenses any controlled substance must be registered
with the DEA and must maintain the DEA certifi cate of
registration at the registered location. If a practitioner
has more than one offi ce where controlled substances
are administered and/or dispensed, then the practitioner
must:
A. Register only the principal offi ce location.
B. Register each offi ce location where controlled substances
are administered and/or dispensed.
C. Register only those offi ce locations where controlled
substances are prescribed.
D. Do nothing.
E. Register every location that uses prescription pads.

A
  1. Answer: B
    Explanation:
    Reference: 21 U.S.C. § 823; 21 C.F.R. § 1301.12;
    www.deadiversion.usdoj.gov.
    Explanation:
    A. This is not the correct answer. If a practitioner
    administers and/or dispenses controlled substances at
    more than one location, he/she must register and post a
    DEA registration number at each principal place of
    business or professional practice where controlled
    substances are stored, administered, or dispense by a
    person. Thus, under this hypothetical registering only one
    principal offi ce location is insuffi cient.
    B. This is the best answer. A separate registration is
    required for each principal place of business or
    professional practice where controlled substances are
    stored, administered or dispensed by a person. If a
    practitioner will only be prescribing from an additional
    location located within the same state, then an additional
    registration is not necessary.
    C. This is not the best answer. A practitioner must register
    each principal place of business or professional practice
    where controlled substances are stored, administered, or
    dispensed by a person.
    D. This is not the correct answer. Do nothing is not the answer here as a registration is required at all locations
    where practitioners administer and/or dispense controlled
    substances. This is because the DEA requires registration
    of locations that keep controlled substances on the
    premises. The practice is different if the only thing kept on
    the premises is a prescription pad.
    E. This is not the correct answer. Registering a principal
    place of business where one administers and/or dispenses
    controlled substances is different from registering every
    place where the practitioner travels and issues
    prescriptions for controlled substances. Practitioners
    must register their principal place of business. If a
    practitioner has several offi ces, he/she must register
    separately those offi ces where he/she administers and/or
    dispenses controlled substances. However, he/she is not
    required to register every location where he/she uses
    prescription pads for controlled substances prescriptions.
    Source: Jennifer Bolen, JD, Sep 2005
How well did you know this?
1
Not at all
2
3
4
5
Perfectly
163
Q
  1. What are the basic requirements for prescribing
    controlled substances?
    A. DEA Registration, state DEA registration (where required),
    state medical license allowing the provider
    to prescribe controlled substances, and a legitimate
    medical purpose within the usual course of professional
    practice.
    B. DEA Registration, state DEA registration (where required),
    and a state medical license allowing the provider
    to prescribe controlled substances.
    C. DEA Registration and a state medical license.
    D. DEA Registration, a state medical license, and evidence
    showing that you have had 10 or more hours training in
    controlled substance prescribing.
    E. State DEA registration and a state medical license.
A
  1. Answer: A
    Explanation:
    Reference: 21 U.S.C. § 829; 21 C.F.R. § 1306.01 through §
    1306.26; www.deadiversion.usdoj.gov.
    Explanation:
    A. This is the best answer. Only practitioners acting in the
    usual course of their professional practice may prescribe
    controlled substances. Practitioners must be registered
    with DEA and licensed to prescribe controlled substances
    by the State(s) in which they operate. In addition, if the
    state requires a separate state DEA registration number,
    practitioners must obtain this registration prior to
    applying for a federal DEA registration. Finally, a
    prescription must be issued in the usual course of
    professional practice and for a legitimate medical purpose
    (or authorized research).
    B. This is not the correct answer because it omits the
    requirement of (1) legitimate medical purpose within (2)
    the usual course of professional practice.
    C. This is not the correct answer because it omits the
    requirement of a state DEA registration number, which
    many states require.
    D. This is not the correct answer because the federal law
    does not contain a training requirement currently. Note,
    however, this may change in the near future due to the
    growing abuse and diversion of prescription controlled
    drugs in the United States. Several states require some
    level of training in the use of controlled substances to treat
    pain.
    E. This is not the correct answer because it omits two
    elements: (1) the state DEA registration number (where
    required), and (2) legitimate medical purpose within the
    usual course of professional practice.
    Source: Jennifer Bolen, JD, Sep 2005
How well did you know this?
1
Not at all
2
3
4
5
Perfectly
164
Q

2638.You have a patient you have seen for several years. The
patient is stable and has been on the same controlled
substances, including a Schedule II drug, for one year.
The patient has been relatively compliant with your
treatment plan and fully compliant with medication
issues. Identify the federal legal/regulatory material that
prohibits you from issuing this patient multiple schedule
II medications with different fi ll dates or “do not fi ll
before” language on your prescriptions.
A. The Interim Policy Statement of November 16, 2004.
B. The Controlled Substances Act of 1970.
C. The Code of Federal Regulations pertaining to the issuance
of prescriptions.
D. A, B, and C.
E. My state allows this so there is nothing in the federal
legal/regulatory material that prevents me from using
multiple schedule II prescriptions with different fi ll
dates with my patients.

A
  1. Answer: D
    Explanation:
    Reference: DEA Interim Policy Statement, Fed. Reg. Vol.
    69, No. 220, pp. 67170-7172 (Nov. 16, 2004).
    Explanation:
    A. This is not the best answer. Although the Interim Policy
    Statement discusses the prohibition against the use of
    “multiple schedule II prescriptions with different fi ll
    dates,” the Interim Policy Statement refers to the
    Controlled Substances Act of 1970’s prohibition against
    refi lls of Schedule II controlled substances. Likewise, the
    federal law on prescriptions for scheduled drugs is found
    in the CFR. For the answer to be correct, you would have
    to look to all three levels of federal materials on the subject
    matter.
    B. This is not the best answer. Although the Controlled
    Substances Act of 1970 prohibits refi lls of Schedule II
    prescriptions, it is the Interim Policy Statement that
    actually discusses the prohibition against the use of
    “multiple schedule II prescriptions with different fi ll
    dates.” Likewise, the federal law on prescriptions for
    scheduled drugs is found in the CFR. For the answer to be
    correct, you would have to look to all three levels of federal
    materials on the subject matter.
    C. This is not the best answer. Although the Code of
    Federal Regulations contains references to prescriptions
    and the prohibition against refi lls on Schedule II
    controlled substances, it is the Interim Policy Statement
    that actually discusses the prohibition against the use of
    “multiple schedule II prescriptions with different fi ll
    dates.” Likewise, the federal law on prescriptions for
    scheduled drugs is found in the CSA. For the answer to be
    correct, you would have to look to all three levels of federal
    materials on the subject matter.
    D. This is the best answer. DEA discussed the prohibition
    against the use of multiple schedule II prescriptions with
    different fi ll dates in the Interim Policy Statement.
    However, DEA makes reference to the CSA and the federal
    law also involves the Code of Federal Regulations. Thus,
    all three levels of federal materials apply.
    E. This is not the correct answer. Federal law takes priority
    over state law in this area. States may create laws and rules
    that are stricter than the federal rule prohibiting the use of
    multiple schedule II prescriptions with different fi ll dates,
    but states may not permit their providers to use
    prescriptions in this format - with “Do not fi ll before”
    dates and language.
    Source: Jennifer Bolen, JD, Sep 2005
How well did you know this?
1
Not at all
2
3
4
5
Perfectly
165
Q
  1. What are the main types of legal/regulatory material at
    the federal level governing record-keeping for the use of
    controlled substances for the treatment of pain?
    A. Acts and Laws, regulations and rules, and guidelines,
    policy or position statements.
    B. Laws and regulations.
    C. Controlled Substances Act of 1970.
    D. DEA Policy Statements.
    E. Food and Drug Act.
A
  1. Answer: A
    Explanation:
    Reference: Federal and state materials nationwide; Bolen, J
    Pain Medicine News; Bolen, J Journal of Opioid
    Management (forthcoming publication 2005). Explanation:
    A. This is the best answer. At the federal level there are
    three main types of legal/regulatory materials governing
    the parameters of and record-keeping for the use of
    controlled substances to treat pain: (1) laws, like the
    Controlled Substances Act of 1970, (2) regulations
    governing the issuance of prescriptions, as found in the
    Code of Federal Regulations, and (3) policy statements
    and rules, as used by DEA to explain the federal
    interpretation or position on the laws and regulations.
    B. This is not the best answer. Laws and regulations do
    exist at the federal level. However, the DEA uses an
    additional level of legal/regulatory material called policy
    statements and rules to explain the laws. It is vital for DEA
    Registrants to read and understand these policy statements
    and rules because they give the registrant a better
    understanding of how the DEA applies and interprets the
    federal legal/regulatory materials governing recordkeeping
    and the use of controlled substances to treat pain.
    C. This is not the best answer. Although the Controlled
    Substances Act of 1970 is a law, it is just part of the body of
    federal materials governing the use of controlled
    substances in the United States.
    D. This is not the best answer. The DEA policy statements,
    such as the Interim Policy Statement of November 16,
    2004, are only a part of the body of federal materials
    governing the use of controlled substances in the United
    States.
    E. This is not the best answer. The Food and Drug Act is a
    law that, in general, pertains to the development, approval,
    and marketing of drugs in the United States. While it
    certainly has much to do with the drugs that ultimately get
    scheduled under the controlled substances act, it is only
    part of the body of federal materials governing the use of
    controlled substances in the United States.
    Source: Jennifer Bolen, JD, Sep 2005
How well did you know this?
1
Not at all
2
3
4
5
Perfectly
166
Q

2640.A registered individual practitioner is not required to
keep records of controlled substances in Schedules II,
III, IV, and V which are prescribed in the lawful course of
professionalpractice, unless
A. He/she prescribes controlled substances in the course of
maintenance or detoxifi cation treatment of a patient.
B. He/she prescribes controlled substances at more than
one practice location.
C. He/she uses the Internet for all patient contact.
D. He/she has been registered for less than three years.
E. He/she uses electronic medical records.

A
  1. Answer: A
    Explanation:
    A. This is the best answer. Unless a practitioner prescribes
    controlled substances in the course of detoxifi cation or
    maintenance of opioid addiction, he/she is NOT
    REQUIRED to keep the records described by the
    Controlled Substances Act of 1970. However, most states
    have Uniform Controlled Substances Act, a medical
    practice act, regulations or rules, and/or guidelines or
    policy statements (sometimes called position statements)
    containing recordkeeping requirements for controlled
    substances.
    B. This is not the correct answer. The fact that a provider
    prescribes at more than one location does not change
    his/her obligation to follow federal and state controlled
    substances recordkeeping mandates.
    C. This is not the correct answer. A provider who uses the
    Internet to interact with patients must do so pursuant to a
    valid physician patient relationship. Likewise, providers
    must follow federal and state controlled substances
    recordkeeping mandates.
    D. This is not the correct answer. It does not matter how
    long a provider has been registered with the DEA. As long
    as a practitioner holds a DEA registration, he/she is
    responsible for following federal and state controlled
    substances recordkeeping mandates.
    E. This is not the correct answer. Using electronic medical
    records does not excuse a provider from following federal
    and state controlled substances recordkeeping mandates.
    Source: Jennifer Bolen, JD, Sep 2005
167
Q

2641.All of the statements are correct about Drug Abuse
Prevention and Control Act of 1970 EXCEPT:
A. It is Title 21, Chapter 13 of US Code
B. Established current schedules, registrations, agencies,
enforcement and penalties.
C. There has been little change in laws since that time.
D. Enforcement since inception has changed signifi cantly.
E. Described schedules of controlled drugs

A
  1. Answer: E
    Explanation:
    Drug Abuse Prevention and Control Act of 1970
    * Title 21, Chapter 13 of US Code
    * Established current schedules, registrations, agencies,
    enforcement and penalties.
    * Little change in laws since that time, although
    enforcement varies and case law has modifi ed
    interpretation.
    Source: Roger Cicala, MD, Sep 2005
168
Q
  1. Phencyclidine may best be characterized by which of the
    following statements?
    A. It has opioid activity
    B. Its mechanism of action is related to its anticholinergic
    properties
    C. It can cause signifi cant hallucinogenic activity
    D. It causes signifi cant withdrawal symptoms
    E. Treatment of overdose is with an opiate
A
  1. Answer: C
    Explanation:
    Reference: Hardman, pp 574-575
    A. Phencyclidine has no opioid activity
    B. Its mechanism of action is amphetamine-like with
    opioid activity.
    C. Phencyclidine is a hallucinogenic compound with no
    opioid activity. Its mechanism of action is amphetaminelike.
    D. A withdrawal syndrome has not been described for this
    drug in human subjects.
    E. In overdose, the treatment of choice for the psychotic
    activity is the antipsychotic drug haloperidol.
    Source: Stern - 2004
169
Q

2643.The federal law governing the scheduling of drugs as
“controlled substances” is called:
A. The Food, Drug, and Cosmetic Act of 1962.
B. The Federal Uniform Controlled Substances Act.
C. The Code of Federal Regulations.
D. The Controlled Drug Act.
E. The Controlled Substances Act of 1970.

A
  1. Answer: E
    Explanation:
    Reference: The Controlled Substances Act of 1970, codifi ed
    in 21 U.S.C. § 801 and sections that follow.
    Explanation: A. The Food, Drug, and Cosmetic Act of 1962, establishes
    the law on drug development, approval for medical use,
    and marketing in the United States. However, this Act does
    not contain the law governing the scheduling of drugs as
    controlled substances.
    B. There is no such federal act. Instead, many states have
    adopted a Uniform Controlled Substances Act (sometimes
    referred to as a Uniform Controlled Drugs Act).
    C. The Code of Federal Regulations explains various
    components of the Controlled Substances Act of 1970, but
    the CFR does not contain the law relating to the
    scheduling of drugs in the United States.
    D. There is no such federal act.
    E. The Controlled Substances Act of 1970 contains the law
    in the United States governing the scheduling of drugs as
    “controlled substances.” The CSA places controlled
    substances into fi ve schedules. Schedule I contains drugs
    with no accepted medical use. Schedule I drugs are
    available only for scientifi c research. Schedules II-V
    contain drugs that have been approved for medical and
    schedules them according to potential for abuse, with
    drugs having the highest potential for abuse assigned to
    Schedule II.
    Source: Jennifer Bolen, JD, Sep 2005
170
Q

2644.A deliberate (dealing) practitioner characteristics
include all of the following EXCEPT:
A. Practitioner becomes a mercenary
B. Sells drugs for money, sex, street drugs, etc.
C. Offi ce becomes a pill factory—full of drug seekers
D. Prescribes for known addicts who will likely sell drugs
to others
E. Keeps close contact with DEA

A
  1. Answer: E
    Explanation:
    Deliberate (Dealing)
    Practitioner becomes a mercenary
    Sells drugs for money, sex, street drugs, etc.
    Offi ce becomes a pill factory—full of drug seekers
    Prescribes for known addicts who will likely sell drugs to
    others
    Source: Roger Cicala, MD, Sep 2005
171
Q

2645.Which one of the following statements best describes the
mechanism of action of benzodiazepines?
A. Benzodiazepines activate GABA B receptors in the spinal
cord
B. Their inhibition of GABA transaminase leads to increased
levels of GABA
C. Benzodiazepines block glutamate receptors in hierarchical
neuronal pathways in the brain
D. They increase the frequency of opening of chloride ion
channels that are coupled to GABA A receptors
E. They are direct-acting GABA receptor agonists in the
CNS

A
  1. Answer: D
    Explanation:
    Benzodiazepines are thought to exert most of their CNS
    effects by increasing the inhibitory effects of GABA.
    Benzodiazepines interact with specifi c receptors (BZ
    receptors) that are components of the GABA A receptorchloride
    ion channel macromolecular complex to increase
    the frequency of chloride ion channel opening.
    Benzodiazepines are not GABA receptor agonists because
    they do not interact directly with this component of the
    complex.
    Source: Katzung & Trevor’s Pharmacology, Examination
    and Board Review, 6th Ed., McGraw Hill, New York, 1998
172
Q

2646.Concerning the clinical uses of benzodiazepines and
related drugs, which one of the following statements is
accurate?
A. Alprazolam is effective in the management of obsessivecompulsive
disorders
B. Clonazepam has effectiveness in patients who suffer
from phobic anxiety states
C. Diazepam is used for chronic management of bipolar
affective disorder in patients who are unable to tolerate
lithium
D. Intravenous buspirone is useful in status epilepticus
E. Symptoms of the alcohol withdrawal state may be alleviated
by treatment with zaleplon

A
  1. Answer: B
    Explanation:
    Benzodiazepines have no signifi cant therapeutic benefi t in
    the management of obsessive-compulsive disorders.
    Drugs effective for this condition increase the activity of
    serotonergic systems in the brain. Clonazepam has been
    used commonly as an anticonvulsant and also has effi cacy
    in anxiety states, including agoraphobia. Clonazepam (not
    diazepam) has also been used as a back-up drug in bipolar
    affective disorder.
    Source: Katzung & Trevor’s Pharmacology, Examination
    and Board Review, 6th Ed., McGraw Hill, New York, 1998
173
Q

2647.Choose the accurate statements concerning the
Barbiturates and Benzodiazepines:
A. Compared with benzodiazepines, barbiturates exhibit a
fl atter dose-response relationship
B. Respiratory depression caused by barbiturate overdosage
can be reversed by fl umazenil
C. An increase in urinary pH will accelerate the elimination
of phenobarbital
D. Barbiturates may increase the half-lives of drugs metabolized
by the liver
E. Symptoms of the abstinence syndrome are less severe
during the withdrawal from secobarbital than from
phenobarbital

A
  1. Answer: C
    Explanation:
    A. The dose-response curve for benzodiazepines is fl atter
    than that for barbiturates.
    B. Flumazenil is an antagonist at Benzodiazepine
    receptors and is used to reverse CNS depressant effects of
    benzodiazepines.
    C. As a weak acid (pKa +=7), phenobarbital will exist
    mainly in the ionized (nonprotonated) form in the urine
    at alkaline pH and will not be reabsorbed in the renal
    tubule.
    D. Induction of liver drug-metabolizing enzymes occurs
    with barbiturates and may lead to decreases in half-life of
    other drugs.
    E. Withdrawal symptoms from use of the shorter-acting
    barbiturate secobarbital are more severe than with
    phenobarbital.
    Source: Katzung & Trevor’s Pharmacology, Examination
    and Board Review, 6th Ed., McGraw Hill, New York, 1998
174
Q
  1. A morbidly obese patient with low back pain complains
    of not sleeping well and feeling tired during the day. His
    wife wakes him up several times during the night due to
    his loud snores. Patient wants a prescription for sleep
    medicine. Your next step is:
    A. Prescribe Clorazepate
    B. Prescribe Flurazepam
    C. Prescribe Secobarbital
    D. Prescribe Triazolam
    E. Refer to sleep disorder clinic
A
  1. Answer: E
    Explanation:
    A-D. Benzodiazepines and barbiturates are
    contraindicated in breathing-related sleep disorders
    because they will further compromise ventilation. In the
    obstructive sleep apnea syndrome (pickwickian
    syndrome), obesity is a major risk factor. The best
    prescription you can give this patient is to lose weight.
    E. Patient probably suffers with sleep apnea syndrome and
    should be referred to sleep disorder clinic or sleep study.
    Source: Katzung & Trevor’s Pharmacology, Examination
    and Board Review, 6th Ed., McGraw Hill, New York, 1998
175
Q
  1. Choose the most likely effect resulting from treatment
    with 10 mg of diazepam 3 times daily?
    A. Retrograde amnesia
    B. Improved performance on tests of psychomotor function
    C. Alleviation of the symptoms of major depressive disorder
    D. Increased porphyrin synthesis
    E. Agitation and possible hyperreflexia with abrupt discontinuance
    after chronic use
A
  1. Answer: E
    Explanation:
    A. At high doses, benzodiazepines may cause anterograde
    but not retrograde, amnesia.
    B. Diazepam use can cause a decrease in psychomotor
    function.
    C. Diazepam has no more effectiveness than placebo in
    the treatment of major depression
    D. Benzodiazepines do not increase activity of liver drugmetabolizing
    enzymes or of enzymes involved in
    porphyrin synthesis.
    E. With abrupt discontinuance following chronic use,
    anxiety and agitation may occur, sometimes with
    hyperrefl exia and, rarely, seizures.
    Source: Katzung & Trevor’s Pharmacology, Examination
    and Board Review, 6th Ed., McGraw Hill, New York, 1998
176
Q
  1. All of the following statements are sources of confusion
    in describing chronic pain EXCEPT:
    A. Pain can be described in terms of its physiological underpinnings
    and its felt experience
    B. Patients are ashamed of acknowledging the psychological
    contributors to their pain.
    C. Practitioners are uncomfortable with acknowledging
    psychological contributors
    D. Pain is an unpleasant sensory and emotional experience
    associated with actual or potential tissue damage or
    described in terms of such damage (IASP).
    E. Pain is best described if only structural abnormalities are
    utilized.
A
  1. Answer: E

Source: Murray McAllister, PsyD, LP - Spring 2004

177
Q
  1. Applied to the skin in a transdermal patch (transdermal
    therapeutic delivery system), this drug is used to prevent
    or reduce the occurrence of nausea and vomiting that are
    associated with motion sickness
    A. Diphenhydramine
    B. Chlorpromazine
    C. Ondansetron
    D. Dimenhydrinate
    E. Scopolamine
A
  1. Answer: E
    Explanation:
    Reference: Hardman, p 930.
    All the drugs listed in the question are used as antiemetics.
    A. Diphenhydramine and dimenhydrinate are used orally
    for the active and prophylactic treatment of motion
    sickness.
    B. Chlorpromazine is a general antiemetic, used orally,
    rectally, or by injection for the control of nausea and
    vomiting that is caused by conditions that are not
    necessarily defi ned.
    C. Ondansetron is indicated in the oral or intravenous
    route for the prevention of nausea and vomiting caused by
    cancer chemotherapy.
    D. Dimenhydrinate is used for prophylaxis and treatment
    of motion sickness.
    E. Scopolamine is a transdermal preparation used in the
    prevention of motion sickness.
    The drug is incorporated into a bandage-like adhesive
    unit that is placed behind the ear.
    The scopolamine delivered in this manner is well
    absorbed and maintains an effect for up to
    72h.
    Other drugs that are prepared for transdermal
    delivery include clonidine (an antihypertensive agent),
    estradiol (an estrogen), fentanyl ( an opioid analgesic),
    nicotine (a smoking deterrent), nitroglycerin (an
    antianginal drug), and testosterone ( an androgen).
    Source: Stern - 2004
178
Q
  1. What are the correct statements about long-term use of
    opioids in chronic non-cancer pain?
    A. No reliable long term studies (> 8 months) that demonstrate
    effi cacy and safety of long term opioid therapy
    for chronic pain
    B. Overall relief with opioids is high (75%)
    C. Physicians must be liberal when prescribing long term
    opioids
    D. Outcomes are not patient-specifi c
    E. No risk of adverse events, addiction, diversion, or noncompliance
    have been noted
A
  1. Answer: A
    Explanation:
    * No reliable long term studies (> 8 months) that
    demonstrate effi cacy and safety of long term opioid
    therapy for chronic pain
    * Overall relief with opioids is modest (35%)
    * Physicians must be cautious when prescribing long term
    opioids
    * Outcomes are patient-specifi c
    * Be aware of risk of adverse events
    - Addiction; diversion; noncompliance
    - Concomitant psychiatric illness
    - Accidental overdose; etc
    * What are your true practice costs?
    Source: Mark V. Boswell, MD, KSIPP 2005
179
Q
  1. Compared with midazolam, diazepam has which of the
    following characteristics?
    A. Greater solubility in water
    B. Shorter beta half-life
    C. More potent ventilatory depressant effect
    D. Lower risk for thrombophlebitis
    E. A pharmacologically active metabolite
A
  1. Answer: E
    Source: American Board of Anesthesilogy, In-trainnig
    examination
180
Q
2654.The second most common opioid of choice for intrathecal
infusion following morphine is:
A. fentanyl
B. sufentanil
C. hydromorphone
D. morphine
E. mepiridine
A
  1. Answer: C

Source: Nader and Candido – Pain Practice. June 2001

181
Q
2655.A patient complains of inadequate analgesia and
increases his use of his medication. This behavior may
represent:
A. Addiction
B. Drug abuse
C. Tolerance
D. Drug diversion
E. All or any of the above
A
  1. Answer: E
    Explanation:
    If a patient increases the medication despite the knowledge
    that he will be discharged, this may be addiction. If he
    increases the medication because it is no longer effective,
    that may be tolerance.
    Source: Trescot AM, Board Review 2004
182
Q
  1. An 82-year-old woman, otherwise healthy for her age, has
    diffi culty sleeping. Triazolam (Halcion®) is prescribed
    for her at one-half of the conventional adult dose. The
    most likely explanation for the increased sensitivity of
    elderly patients to a single dose of triazolam and other
    sedative-hypnotic drugs is
    A. Changes in plasma protein binding
    B. Decreased renal function
    C. Increased cerebral blood fl ow
    D. Decreased hepatic metabolism of lipid-soluble drugs
    E. Changes in brain function that accompany the aging
    process
A
  1. Answer: E
    Explanation:
    Decreased blood fl ow to vital organs, including the liver
    and kidney, occurs during the aging process. These
    changes may contribute to cumulative effects of sedativehypnotic
    drugs. However, this does not explain the
    enhanced sensitivity of the elderly patient to a single dose
    of central depressant, which appears to be due to changes
    in
    brain function that accompany aging.
    Source: Katzung & Trevor’s Pharmacology, Examination
    and Board Review, 6th Ed., McGraw Hill, New York, 1998
183
Q

2657.Choose the correct statement about a duped
practitioner.
A. Never assumes the best about his patients and is gullible
B. Never leaves script pads lying around
C. Does not believe in hydrophilic medicine “fell” into the
toilet or the sink
D. Believes when patients only want specifi c medications
(i.e. OxyContin or Percocet)
E. Never co-dependent - always tells patients “No” when
they ask for narcotics

A
  1. Answer: D
    Explanation:
    Duped
    Always assumes the best about his patients and is gullible
    Leaves script pads lying around
    Hydrophilic medicine—fell into the toilet or the sink
    Patients only want specifi c medications (i.e. OxyContin or
    Percocet)
    Co-dependent—cannot tell patients “No” when they ask
    for narcotics
    Source: Roger Cicala, MD, Sep 2005
184
Q
2658. The Schedule I substance among the following drugs is?
A. Buprenorphine
B. Hydromorphone
C. Heroin
D. Diazepam
E. Morphine
A
  1. Answer: C
    Explanation:
    The Controlled Substances Act has divided drugs under its
    jurisdiction into fi ve schedules. Schedule I drugs have a
    high potential for abuse and no accepted medical use in
    the
    United States. Examples of schedule I drugs include
    heroin, marijuana, LSD, etc.
    A. Buprenorphine is Schedule III drug
    B. Hydromorphone is Schedule II
    C. Heroin is Schedule II
    D. Diazepam is Schedule IV
    E. Morphine is Schedule II
    Source: Manchikanti L, Board Review 2005
185
Q
  1. Your friend’s daughter whom you have known for several
    years makes an appointment with you. During the visit,
    she tells you that she is a heroin addict and requests
    a prescription for Hydrocodone. Your options in this
    situation are as follows:
    A. Immediately call her father and give hydrocodone.
    B. Immediately tell father and give her Methadone.
    C. Start rapid detoxifi cation in your offi ce.
    D. Provide her with a prescription for Methadone maintenance
    E. Do not tell the father and do not give Hydrocodone.
A
  1. Answer: E
    Explanation:
    A. A physician has to maintain patient confi dentiality.
    Further, she may be addicted to not only heroin, but
    Hydrocodone.
    It is not certain at this point.
    She may be receiving methadone from other sources.
    B. A physician has to maintain patient confi dentiality.
    Further, she may be addicted to not only heroin, but
    Hydrocodone.
    It is not certain at this point.
    She may be receiving methadone from other sources.
    D. Methadone maintenance treatment requires special
    licensure.
    C. Similarly, rapid detoxifi cation also requires a special
    license.
    E. The best option is to maintain confi dentiality, protect
    the patient, and yourself.
    Source: Manchikanti L, Board Review 2005
186
Q
  1. The following statements are true with typical detection
    times for urine testing of common drugs of abuse.
    A. Methadone, 2 to 4 days
    B. Chronic use of marijuana, 1 to 3 days
    C. Morphine, 15 days
    D. Cocaine, 15 days
    E. Benzodiazepines, 15 days
A
  1. Answer: A

Explanation:

187
Q
  1. A patient presents to you with chronic low back pain. He
    is being treated with OxyContin 40 mg twice a day and
    hydrocodone 10 mg four times daily. You performed a
    drug testing. The results of the drug testing were positive
    for oxycodone, hydromorphone, and hydrocodone. The
    results indicate the following:
    A. The patient abusing controlled substances by taking
    non-prescribed drugs.
    B. He is non-compliant by not taking the prescribed drugs.
    C. He is selling drugs
    D. The results of the drug test show a normal pattern
    E. The drug test indicates the patient is taking his mother’s
    hydromorphone
A
  1. Answer: D

Source: Manchikanti L, Board Review 2005

188
Q
  1. What are the correct statements about a defi cient (dated
    practitioner) ?
  2. Too busy to keep up with CME
  3. Only aware of a few treatments or medications
  4. Prescribes for friends or family without a patient record
  5. Well aware of controlled drug categories
A
2662. Answer: E (All)
Explanation:
Defi cient (Dated Practitioner)
Too busy to keep up with CME
Unaware of controlled drug categories
Only aware of a few treatments or medications
Prescribes for friends or family without a patient record
Unaware of symptoms of addiction
Remains isolated with peers
Only education from reps
Source: Laxmaiah Manchikanti, MD
189
Q
  1. True statements concerning patient controlled analgesia
    include the following:
  2. Better patient satisfaction with pain control.
  3. Shorter hospital stays.
  4. Less total analgesic use.
  5. A greater potential for subsequent opiate dependence
A
  1. Answer: A (1,2, & 3)

Source: Stimmel, B

190
Q
  1. What were the physician opioids of choice in the 1990s?
  2. Oxycodone 20%
  3. Hydrocodone 70%
  4. Dilaudid 20%
  5. Fentanyl class 10%
A
2664. Answer: C (2 & 4)
Explanation:
Physician Opioid of Choice
1990s
Hydrocodone 70%
Fentanyl class 10%
Dilaudid < 5%
Oxycodone < 5%
Codeine < 5%
2002
Ultram 70%
Hydrocodone 40%
Oxycodone 25%
Fentanyl class < 5%
Dilaudid < 5%
Source: Roger Cicala, MD, Sep 2005
191
Q
  1. What were the physician opioids of choice in 2002?
  2. Hydrocodone 40%
  3. Oxycodone 25%
  4. Ultram 70%
  5. Dilaudid 25%
A
2665. Answer: A (1,2, & 3)
Explanation:
Physician Opioid of Choice
1990s
Hydrocodone 70%
Fentanyl class 10%
Dilaudid < 5%
Oxycodone < 5%
Codeine < 5%
2002
Ultram 70%
Hydrocodone 40%
Oxycodone 25%
Fentanyl class < 5%
Dilaudid < 5%
Source: Roger Cicala, MD, Sep 2005
192
Q
  1. Choose all side effects of clonidine (Catapres):
  2. Drowsiness
  3. Hypotension
  4. Dizziness
  5. Dry mouth
A
  1. Answer: E (All)

Source: Stimmel, B

193
Q
  1. What are the advantages of prolonged, high-dose opioid
    therapy?
  2. No evidence to support high dose therapy (>200 to 300
    mg/day or more)
  3. Opioid doses should not be limited in the name of improving
    effi cacy and safety
  4. Anecdotal evidence that pain relief not better
  5. There is solid evidence that 3600 mg of oxycodone per
    day is effective in neuropathic pain
A
  1. Answer: B (1 & 3)
    Explanation:
    Prolonged, High-Dose Opioid Therapy:
    No evidence to support high dose therapy (> 200 to 300
    mg/day or more)
    Anecdotal evidence that pain relief not better
    Opioid doses should be limited to improve effi cacy and
    safety
    Source: Mark V. Boswell, MD, KSIPP 2005
194
Q
2668. Analgesia of six hours or longer in duration can be
obtained with all the following drugs:
1. Levo Dromoran
2. MS Contin
3. Methadone
4. Oxycodone
A
  1. Answer: A (1,2, & 3)

Source: Stimmel, B

195
Q
  1. Opioids recommended for lactating patients include
  2. Morphine
  3. Hydromorphone
  4. Hydrocodone
  5. Meperidine
A
2669. Answer: A (1,2, & 3)
Explanation:
Meperidine is contraindicated for lactation because
normeperidine collects in the neonate
Source: Boswell MV, Board Review 2005
196
Q
  1. What were the physician drugs of abuse in 2001?
  2. Opioids 30%
  3. Alcohol 20% to 30%
  4. Benzodiazepines 20%
  5. Marijuana 2%
  6. What are the pitfalls of prescription practices?
  7. 4 D’s - Defi cient, Duped, Deliberate, Dependent Practitioner
  8. Never say “NO” - Family, Friends, Patients
  9. Ignore complaints
  10. Focus on positive aspects of regulations and reimbursement
A
2670. Answer: B (1 &amp; 3)
Explanation:
Physician Drugs of Abuse 2001
Alcohol 50% to 60%
Opioids 30%
Benzodiazepines 20% (40% for females)
Marijuana 20%
Cocaine 10%
Amphetamines, Ritalin 10%
Source: Roger Cicala, MD, Sep 2005
  1. Answer: A (1,2, & 3)
    Explanation:
    Top 10 Pitfalls
  2. 4 D’s – Defi cient, Duped, Deliberate, Dependent
    Practioner
  3. Weak heart – pretend addiction doesn’t exist
  4. Never say “NO” – Family, Friends, Patients
  5. Poor documentation
  6. No policies – No agreements
  7. Ignore complaints
  8. Focus on negative aspects of regulations and
    reimbursement
  9. Not nice to investigators from Board, DEA!!
  10. Reckless disregard to law with prescription pads and
    regulations
  11. Know it all – Do it all
    Source: Laxmaiah Manchikanti, MD
197
Q
  1. What are the clinical implications of non-responsiveness
    to opioids in chronic pain? Choose the correct
    statements.
  2. Diffi cult to distinguish pharmacologic tolerance from
    opioid-induced abnormal pain sensitivity
  3. Treating increasing pain with increasing doses may be
    futile
  4. High dose therapy may have adverse consequences
  5. Push the dose to highest level or combine 2 opioids
A
  1. Answer: A (1,2, & 3)
    Explanation:
    Clinical Implications:
    * Difficult to distinguish pharmacologic tolerance from
    opioid-induced abnormal pain sensitivity
    * Treating increasing pain with increasing doses may be
    futile
    * High dose therapy may have adverse consequences
    Source: Mark V. Boswell, MD, KSIPP 2005
198
Q
2673. Which of the following include subjective reports of
marijuana effects?
1. Increased sexual desire
2. Increased appetite
3. Enhanced tactile sensitivity
4. Increased motivation
A
  1. Answer: A (1,2, & 3)

Source: Stimmel B

199
Q
2674. The most signifi cant increase in prescriptive medication
for illicit use is:
1. THC (tetrahydrocannabinol)
2. Cocaine
3. Benzodiazepines
4. Pain relievers
A
  1. Answer: D (4 Only)
    Explanation:
    2002 and 2003 saw a signifi cant increase in non-medical
    use of pain relievers. This seems to be a trend over the
    past
    decade. Hydrocodone leads at 15 million, followed by
    oxycodone (Oxycontin? specifi cally mentioned as a subset
    of oxycodone), methadone, and tramadol.
    Source: Hans C. Hansen, MD
200
Q
  1. A 34-year-old recently married man seeks help from
    a methadone clinic. Chose all of the true statements meeting criteria for acceptance into the program:
  2. He has AIDS
  3. He has no legal charges pending
  4. He has used heroin for 2 years
  5. He has been married to a heroin addict
A
  1. Answer: A (1,2, & 3)

Source: Stimmel, B

201
Q
  1. What are the risks of malprescribing related to practice
    management?
  2. Loss of “Provider Status”
  3. Insurers frequently report to Boards
  4. Plans may remove providers for “overprescribing”
  5. Insurers are unable to report any type of national databank
    for malprescribing
A
  1. Answer: A (1,2, & 3)
    Explanation:
    Risks of Malprescribing
    Loss of “Provider Status”
    Insurors frequently report to Boards Insurors frequently
    report to Boards now
    Several plans have removed providers for
    “overprescribing.” Seems more common with more
    expensive agents (Duh!).
    Insurors can report to a separate national data bank, not
    available to public, but available to hospitals and other
    insurors.
    Source: Laxmaiah Manchikanti, MD
202
Q
  1. A 16-year-old boy is brought for emergency evaluation
    after taking some of his mother’s medication in order to
    get “high.” He is fl ushed and his pupils are dilated and
    only poorly reactive. He complains of dry mouth. He
    is restless, confused at times, and may be having visual
    hallucinations. Which of the following medications is he
    likely to have taken?
  2. Phenelzine
  3. Disulfi ram
  4. Alprazolam
  5. Benztropine
A
  1. Answer: D (4 Only)
    Explanation:
    Benzotropine (Cogentin) has atropine-like side effects;
    dilated pupils, dry mouth, urinary retention, restlessness,
    confusion and toxic psychosis.1. Phenelzine, a MAOI,
    reacts with tyramine-containing substances causing a
    “cheese reaction,” which consists of sweating, palpitations,
    headache, and increased blood pressure resulting in a
    possible intracerebral hemorrhage.2. Disulfi ram
    (Antabuse), if taken with alcohol, cuases fl ushing,
    throbbing, sweating, thirst, respiratory diffi culty, nausea,
    vomiting, tachycardia, hypotension, vertigo, blurred
    vision, and confusion.3.Alprazolam, a benzodiazepine,
    causes sedation, impairment of performance, and
    dependency.
    Source: Psychiatry specialty Board Review By William M.
    Easson, MD and Nicholas L. Rock, MD
203
Q
2678. Cocaine abuse is likely to produce symptoms similar to
which of the following?
1. Major depression
2. Obsessive compulsive disorder
3. Generalized anxiety disorder
4. Paranoid schizophrenia
A
  1. Answer: D (4 Only)
    Explanation:
    Cocaine blocks neuronal dopamine, serotonin, and
    norepinephrine reuptake. With prolonged cocaine use and
    abuse, a delusional psychosis similar to paranoid schizophrenia may develop.
    Source: Psychiatry specialty Board Review By William M.
    Easson, MD and Nicholas L. Rock, MD
204
Q
  1. Drugs can be altered by:
  2. Absorbsion
  3. Distribution
  4. Metabolism
  5. Excretion
A
2679. Answer: E (All)
Explanation:
All of these actions can infl uence the drug-drug
interaction.
Source: Andrea M. Trescot, MD
205
Q
  1. “Opioid rotation”
  2. May work because of variable affi nity for the μ receptor
  3. May work because of differing opioid metabolic pathways
  4. May work because of avoidance of metabolite antagonists
  5. May work because of differing drug interactions
A
  1. Answer: E (All)
    Explanation:
    Different opioids have different receptor affi nity, so that
    switching from a low affi nity opioid like hydrocodone to a
    high affi nity opioid like fentanyl may allow for better
    analgesia at a lower than equipotent dose. Different
    metabolic pathways may explain why hydrocodone (a prodrug
    metabolized by CYP2D6) might not be effective
    when propoxyphene is, especially in a patient who is
    CYP2D6 defi cient. High doses of morphine can lead to
    accumulation of M3G which is hyperalgesic; switching to
    an opioid without this type of antagonist would give better
    analgesia. Inhibition of one enzyme system (such as 3A4
    and methadone) would have no effect on an opioid
    metabolized by another system (such as 2D6 and
    hydrocodone).
    Source: Andrea M. Trescot, MD
206
Q
  1. What are the risks of malprescribing?
  2. Legal charges, probably jail time
  3. Conviction rate is currently almost 30%
  4. Felony conviction will likely prevent or at least severely
    limit future practice
  5. Duped and Dated are highly viable defenses
A
2681. Answer: B (1 &amp; 3)
Explanation:
Risks of Malprescribing
Legal charges, probably jail time
Conviction rate is currently almost 90%
Felony conviction will likely prevent or at least severely
limit future practice
Duped and Dated aren’t viable defense
Source: Laxmaiah Manchikanti, MD
207
Q
  1. The scope of the problem of prescription opioids among
    physicians is as follows:
  2. Up to 20% of prescribed opioids are diverted.
  3. 567 physician arrests and sanctions in 2002.
  4. No more federal diversion program for malprescribing physicians.
  5. State boards may not take any actions.
A
  1. Answer: A (1,2, & 3)
    Explanation:
    Up to 20% of prescribed opioids are diverted.
    567 physician arrests and sanctions in 2002.
    No more Federal diversion program for malprescribing
    physicians.
    State Boards may react to arrests of physicians, increasing
    sanctions.
    Source: Roger Cicala, MD, Sep 2005
208
Q
  1. Drugs able to diminish the dose of opioids analgesics
    required to relieve pain include the following:
  2. Amphetamines
  3. Tricyclic antidepressants
  4. Non-steroidal anti-infl ammatory agents
  5. Acetaminophen (Tylenol)
A
  1. Answer: E (All)

Source: Stimmel, B

209
Q
  1. Identify reasons for a standard order for Meperidine 75
    mg every four hours to be inappropriate?
  2. Effective analgesia lasts only 2.5 to 3 hours.
  3. If a person is also receiving a monoamine oxidase inhibitor,
    severe toxicity can occur.
  4. That dose is equivalent only to 5 to 7.5 mg of morphine.
  5. In the presence of impaired renal function toxicity may
    occur.
A
  1. Answer: E (All)

Source: Stimmel, B

210
Q
2685. Which of the following have been identifi ed as algogenic
substances?
1. Serotonin
2. Leukotrienes
3. Acetylcholine
4. Histamine
A
  1. Answer: E (All)
211
Q

2686.Findings of illicit drug use based on insurance in
interventional pain management setting was:
1. Third party - 17%
2. Medicare with/without third party - 10%
3. Medicare/Medicaid - 24%
4. Medicaid - 39%

A
  1. Answer: E (All)

Source: Laxmaiah Manchikanti, MD

212
Q
2687. Which of the following drugs will increase blood levels
of oxycodone?
1. Fluoxetine (Prozac®)
2. Sertraline (Zoloft®)
3. Paroxetine (Paxil®)
4. Carbamazepine (Tegretol®)
A
2687. Answer: A (1,2, & 3)
Explanation:
Carbamazepine is an inducer of 2D6 and will reduce
oxycodone levels.
Source: Boswell MV, Board Review 2004
213
Q
2688. An opiate overdose patient may present with all of the
following:
1. Increase in respiratory rate
2. Small pupils
3. Hypertension
4. Coma
A
  1. Answer: C (2 & 4)

Source: Stimmel, B

214
Q
2689. Intravenous heroin use causes or is associated with all of
the following:
1. Affective disorder
2. Liver disease
3. Endocarditis
4. Gall bladder disease
A
  1. Answer: A (1,2, & 3)

Source: Stimmel, B

215
Q
  1. Opioids exert their action by:
  2. Inhibiting the release of substance P
  3. Activating G proteins
  4. Inhibiting adenylate cyclase
  5. Activating dopaminergic neurons
A
  1. Answer: E (All)
    Explanation:
    Opioid receptors, concentrated in the ventral tegmental
    and periaqueductal grey areas, presynaptically inhibit the
    transmission of excitatory pathways: acetylcholine,
    catecholamine, serotonin, and substance P. Activation of
    the opioid receptor inhibits adenylate cyclase. All opioid
    receptors are G protein-linked structures embedded in the
    plasma membrane of neurons; activation releases a
    portion of the G protein, which moves in the membrane
    until it reaches its target (either an enzyme or an ion
    channel). These targets alter protein phosphorylation and/
    or gene transcription. Opioids and endogenous opioids
    activate presynaptic receptors on GABA neurons, which
    inhibit the release of GABA in the ventral tegmental area.
    This allows dopaminergic neurons to fi re more vigorously,
    and the extra dopamine in the nucleus accumbens is
    intensely pleasurable.
    Source: Andrea M. Trescot, MD
216
Q
  1. After starting a hospital patient on a morphine PCA, you
    get a call from the nurse that the patient’s face is itching.
    You tell the nurse that:
  2. This is an allergic reaction, and to stop the medicine
  3. This is a histamine release and will likely go away
  4. The chart should be marked “allergic to all opioids”
  5. Changing the PCA to hydromorphone will likely stop
    the itching
A
  1. Answer: C (2 & 4)
    Explanation:
    Itching from opioids (usually the naturally occurring such
    as codeine and morphine) is usually not an
    antibody/antigen reaction but rather a direct histamine
    release from the mast cells, as well as a central μ receptor
    stimulation. Changing to a synthetic opioid such as
    hydromorphone will usually resolve the problem. In the
    face of a “true allergy”, there is usually no cross reactivity
    across classes.
    Source: Andrea M. Trescot, MD
217
Q
2692. All of the following are symptoms of alcohol
withdrawal :
1. Coarse tremor of hands or tongue
2. Generalized tonic-clonic seizures
3. Tachycardia, sweating, dilated pupils
4. Abducent nerve paresis or paralysis
A
  1. Answer: A (1,2, & 3)
    Explanation:
    Alcohol withdrawal occurs when there is a relative drop in
    blood alcohol levels; therefore, it can develop while still
    drinking. The patients are likely to show a coarse, fastfrequency
    generalized tremor that is made worse by motor
    activity or stress and is easily observed when the hands or
    tongue are extended. Withdrawal is manifested by
    autonomic hyperactivity (increased BP, tachycardia,
    sweating), malaise, vomiting with anxiety, depression,
    irritability, cognitive changes, and possible seizures.
    Source: Psychiatry specialty Board Review By William M.
    Easson, MD and Nicholas L. Rock, MD
218
Q

2693.Those that drink alcohol on a regular basis are
recognized to have increased risk when mixed with
controlled substances, particularly sedatives, opioids and
benzodiazepines. Which is true of those that consume
alcohol, and placed at risk in this population?
1. 120 million Americans use alcohol regularly.
2. 20% of the American population consider themselves
alcohol users.
3. 16.1 million are heavy, or regular drinkers.
4. 1% of Americans have participated in a binge drinking
episode one month prior to the survey.

A
  1. Answer: B (1 & 3)
    Explanation:
    Alcohol is a signifi cant problem, particularly when
    controlled substances are being used, and those that have
    signifi cant risks associated with them, particularly
    pharmacokinetically long acting drugs. An example that
    underscores this issue is that the FDA recently removed
    Palladone?, a pharmacokinetically long acting
    hydromorphone preparation from the market. Palladone?
    has been considered for pain control for a number of
    years,
    recently introduced, and to be found an unacceptably high
    risk when mixing alcohol and this drug. 50% of the
    American population consider themselves alcohol
    consumers, and 22% of those participated in a binge
    drinking episode one month prior to the survey. 16.1
    million are heavy drinkers, and at signifi cant risk.
    Source: Hans C. Hansen, MD
219
Q
  1. What are the characteristics of a drug dependent (addict)
    practitioner?
  2. Starts by taking controlled drug samples
  3. Never asks staff to pick up medications in their names
  4. Calls in scripts in names of family members or fi ctitious
    patients and picks them up himself
  5. Never uses another doctor’s DEA number
A
2694. Answer: B (1 &amp; 3)
Explanation:
Drug Dependent (Addict)
Starts by taking controlled drug samples
Asks staff to pick up medications in their names
Uses another doctor’s DEA number
Calls in scripts in names of family members or fi ctitious
patients and picks them up himself
Source: Laxmaiah Manchikanti, MD
220
Q

2695.Upon abrupt discontinuation of L-alpha-acetylmethadol
(LAAM) in a tolerant patient, one may observe:
1. Decreased respiratory response to increased carbon
dioxide.
2. Withdrawal symptoms, which may not appear for up
to 3 days.
3. Vomiting, diarrhea, hypertension, skeletal muscular
twitching as severe signs.
4. Few symptoms until 72 hours have passed.

A
  1. Answer: E (All)

Source: Stimmel, B

221
Q
  1. Highly tolerant opioid users, maintained on their drug
    in a research setting, will:
  2. Continue to feel a “rush” when their drug of choice is
    administered intravenously.
  3. Continue to use their drug of choice for reasons other
    than fear of experiencing withdrawal.
  4. Continue to experience pleasurable effects from food,
    sex, tobacco and other non-opiate drugs.
  5. Will not use any other drug for medical or recreational
    purposes
A
  1. Answer: A (1,2, & 3)

Source: Stimmel, B

222
Q
  1. A former drug addict is admitted to the psychiatric ward
    with depression secondary to chronic back pain. He has
    been taking tramadol 6 tablets per day. He was started on
    paroxetine and you were consulted for pain management.
    He complains that the tramadol is not working, and he is
    noted to become more agitated. Your assessment is:
  2. He is drug seeking
  3. He is withdrawing from illicit medications
  4. His depression is making his pain worse
  5. He is on the inappropriate antidepressant
A
  1. Answer: D (4 Only)
    Explanation:
    The most likely problem is that the paroxetine (Paxil®) (a
    potent CYP2D6 inhibitor) is preventing the metabolism
    of tramadol to the active M1 metabolite. He is therefore
    not drug seeking or withdrawing from illicit drugs, but
    instead has had his previously working opioid made
    ineffective by the drug interaction.
    Source: Andrea M. Trescot, MD
223
Q
  1. What are the Methadone interactions?
  2. Cimetidine will increase methadone levels
  3. Butalbital will decrease methadone levels
  4. Ciprofl oxin will increase methadone levels
  5. Phenytoin will decrease methadone levels
A
  1. Answer: E (All)
    Explanation:
    Cimetidine, and Ciprofl oxin are CYP3A4 inhibitors, and
    will increase methadone levels. Ciprofl oxin can inhibit
    CYP3A4 by up to 65%. Phenytoin and butalbital will
    induce the CYP3A4 enzyme and decrease blood levels.
    Source: Andrea M. Trescot, MD
224
Q
  1. A patient with esophageal cancer has been taking
    hydrocodone 10mg 2 TID by mouth with good relief.
    However, he is admitted to the hospital with esophagitis
    from radiation and is not able to tolerate any medications
    by mouth. Options for pain management include:
  2. 1 mg/hr morphine IV continuously
  3. 50 mg meperidine IM every 6 hours
  4. 0.5mg hydromorphone IV every 4 hours
  5. 25 mcg/hr Fentanyl transdermally
A
  1. Answer: B (1 & 3)
    Explanation:
    The patient’s total dose of hydrocodone is 60mg per day,
    which is equal to 60mg oral morphine per day. This
    converts to 20mg IV morphine per day (30mg morphine =
    10mg IV), which is divided by 24 to get the hourly dose of
    1mg per hour. This is equivalent to 3mg per day of
    hydromorphone (10mg IV morphine = 1.5mg IV
    hydromorphone so 20mg IV morphine = 3mg IV
    hydromorphone), which, divided into 6 doses (q4hrs) =
    0.5mg per dose. Meperidine is not appropriate, and
    fentanyl is too slow an onset for the initial conversion.
    Source: Andrea M. Trescot, MD
225
Q
  1. In the management of alcohol withdrawal delirium, the
    clinician may wish to use all of the following :
  2. Chlordiazepoxide
  3. Magnesium sulfate
  4. Thiamine
  5. Intravenous glucose
A
  1. Answer: E (All)
    Explanation:
    The prefered medications for the management of alcohol
    withdrawal delirium are the benzodiazepines
    (chlordiazepoxide, diazepam, lorazepam, oxyazepam).
    Multivitamins, particularly thiamine, B12, and folic acid,
    should be used. Thiamine IV or IM should be given prior
    to glucose loading. If seizures develop, using magnesium
    sulfate. Clonidine, propranolol, chloral hydrate,
    benzodiazepines, or barbiturates can be used depending
    on the total clinical picture.
    Source: Psychiatry specialty Board Review By William M.
    Easson, MD and Nicholas L. Rock, MD
226
Q
  1. Which of the following are principles of medical ethics?
  2. Justice
  3. Benefi cence
  4. Autonomy
  5. Non-negligence
A
  1. Answer: A (1, 2, & 3)
227
Q
  1. Which of the following statements about alcoholics are
    correct?
  2. Suicidal behavior is common after personal loss
  3. High incidence of alcohol abuse is patients who commit
    suicide
  4. Alcohol tends to worsen depression
  5. Alcoholics who threaten suicide usually do not kill
    themselves
A
  1. Answer: A (1, 2, & 3)
    Explanation:
    Alcoholism is the third largest health problem after heart
    disease and cancer. In males 25-44 years old, alcohol plays
    a major role in all four leading causes of death: accidents,
    homicides, suicides, and alcoholic cirrhosis. The chronic
    use of alcohol produces psychological, interpersonal, and
    medical problems, which include violence, absence from
    work, loss of job, and legal diffi culties. Alcohol is a factor
    associated with at least 50% of traffi c fatalities, 50% of
    homicides, and 25% of suicides.
    Source: Psychiatry specialty Board Review By William M.
    Easson, MD and Nicholas L. Rock, MD
228
Q

2703.True statements regarding the Controlled Substances Act
(CSA) include which of the following?
1. Methadone is a DEA Schedule II controlled substance
that is indicated for the relief of severe pain and detoxifi
cation or maintenance of narcotic addiction.
2. A prescription may not be issued for the dispensing
of methadone for detoxifi cation treatment or maintenance
treatment
3. To use Schedule II opiates for detoxifi cation from opiate
addiction, a special registration is required.
4. Drug Abuse Treatment Act of 2000 allows physicians to
use Schedule III agents to detoxify chemical dependent
patients in an offi ce setting, provided the physician qualifi es for and obtains a waiver issued through DEA.

A
  1. Answer: E (All)
229
Q

2704.True statements regarding the Controlled Substances Act
in determining the control or removal from schedules of
substances include the following:
1. Drugs actual or relative potential for abuse.
2. Scientific evidence of its pharmacological effect, if
known.
3. Any risks to the public health
4. Guarantee by the manufacturer that it will be provided
at affordable price to public.

A
  1. Answer: A (1, 2, & 3)
    Explanation:
    Factors determinative of control or removal from
    schedules by the attorney general are as follows:
  2. Its actual or relative potential for abuse.
  3. Scientifi c evidence for its pharmacological effect, if
    known.
    The state of current scientifi c knowledge regarding the
    drug or other substance.
  4. Potential and current pattern of abuse
    The scope, duration, and signifi cance of abuse.
    What, if any, risk there is to public health.
    Its physic or physiological dependence liability.
    Whether the substance is an immediate precursor of a
    substance already controlled under this
    title.
  5. Price has no impact.
230
Q

2705.True statements about methadone include the following:
1. It is useful as an analgesic
2. It has greater oral effi cacy than morphine
3. It produces a milder but more protracted withdrawal
syndrome than that associated with morphine
4. Adverse reactions may include constipation, respiratory
depression, and light headedness

A
  1. Answer: E (All)
    Explanation:
  2. Methadone is an opioid receptor agonist. It is used as an
    analgesic and to treat opioid abstinence and heroin users
    (methadone maintenance).
  3. Methadone has greater oral effi cacy than morphine and
    a much longer biologic half-life;
  4. Methadone produces milder but more protracted
    abstinence syndrome associated with methadone.
  5. Adverse reactions may include constipation, respiratory
    depression, and light headedness.
231
Q
2706. Drug testing may be performed by utilizing any of the
following technique(s):
1. Urine drug screening
2. Hair samples
3. Saliva testing
4. Specifi c drug analysis of blood
A
  1. Answer: E (All)
232
Q

2707.The transdermal route of fentanyl administration
has been used in cancer patients because it offers the
following advantages:
1. Convenience of dosing.
2. Rapid absorption through the skin allows quick titration.
3. Highly potent opioid for analgesic effi cacy.
4. Low cost.

A
  1. Answer: B (1 & 3)

Source: Reddy Etal. Pain Practice: Dec 2001, march 2002

233
Q
2708.During cocaine withdrawal, which of the following
symptoms can be anticipated?
1. Cardiac arrhythmias
2. Desire for sleep, often with insomnia
3. Delirium
4. Depression
A
  1. Answer: C (2 & 4)
    Explanation:
    Cocaine withdrawal has no specifi c physiological signs,
    but there are physical problems (“crash”) that peak in two
    to four days. Depression and irritability can persist for
    weeks. These patients show a desire for sleep, often with
    insomnia, with disturbed sleep and increased dreaming,
    general fatigue, and suicidal ideation. Drug-seekingbehavior
    usually occurs after bein drug-free for a few days.
    Source: Psychiatry specialty Board Review By William M.
    Easson, MD and Nicholas L. Rock, MD
234
Q

2709.After chronic amphetamine use, abrupt withdrawal is
likely to cause which of the following symptoms?
1. Seizures
2. Delirium
3. Formication
4. Sleep disturbance

A
  1. Answer: D (4 Only)
    Explanation:
    Amphetamines or “speed” are stimulants with reinforcing
    effects similar to cocaine. Chronic amphetamine use
    causes tachycardia, elevated BP, pupillary dilation,
    agitation, elations, and hypervigilance. Adverse side effects
    include insomnia, fever, headaches, confusion, irritability,
    hostility, and visual hallucinations.
    Source: Psychiatry specialty Board Review By William M.
    Easson, MD and Nicholas L. Rock, MD
235
Q
  1. The following statements are accurate for addiction and
    dependence.
  2. Based on the Controlled Substances Act, the term “addict”
    means any individual who habitually uses any
    narcotic drug so as to endanger the public health and
    safety.
  3. Based on DSM-IV defi nition, addiction means maladaptive
    pattern leading to distress or impairment.
  4. DSM-IV defi nition of substance dependence includes
    tolerance, withdrawal, and continued use despite problems.
  5. Federation of State Medical Board guidelines for the
    treatment of pain recommend use of controlled substances
    in patients with history of substance with no
    additional monitoring, referral, or documentation.
A
  1. Answer: B (1 & 3)
    Explanation:
  2. The term “addict” by CSA means any individual who
    habitually uses any narcotic drug so as to endanger the
    public morals, health, safety, or welfare, or who is so far
    addicted to the use of narcotic drugs as to have lost the
    power of self-control with reference to his or her
    addiction.
  3. There is no defi nition for addiction in DSM-IV.
    DSM-IV defi nes substance abuse with at least 1 in 12
    month period.
  4. Maladaptive pattern leading to distress or impairment.
    Recurrent failure to fi eld role.
    Recurrent physically undesirous behavior.
    Recurrent legal problems.
    Continued use despite social problems.
    Never met dependence criteria
    DSM-IV defi nition for substance dependence is as follows:
    Tolerance
    Withdrawal
    Larger Amounts/Longer periods
    Efforts or desire to cut down
    Large amount of time using/obtaining/recovering
    Activities given up: social/work/recreation
    Continued use despite problems
    Need 3 of above in 12 months
    An alternate defi nition from the American Society of
    Addiction Medicine for addiction is as follows:
    Addiction
    A primary, chronic neurobiologic disease with
    genetic, psychosocial and environmental factors
    effecting its course and presentation
    Characterized by one or more of the following
    Impaired control of drug use
    Compulsive use
    Craving
    Continued use despite harm
  5. The federation recommends several additional steps in
    patients with addiciton or abuse
    Source: Manchikanti L, Board Review 2005
236
Q
  1. Choose the accurate statement(s) below regarding
    the purposes for which a prescription for controlled
    substance can be issued:
  2. A prescription for a controlled substance must be issued
    for a legitimate medical purpose in the usual course of
    a physician’s practice.
  3. A prescription for a controlled substance may be issued
    to a physician so that he or she can dispense the drugs
    to patients as medically necessary.
  4. Prescriptions for Schedule III, IV, and V drugs are permitted
    for purposes of detoxifi cation or maintenance
    treatment if the drug is specifi cally approved by the
    Food and Drug Administration (FDA) for those uses
    and the prescribing physician meets the legal requirements
    for prescribing such drugs for such uses.
  5. Prescriptions for Schedule II drugs for detoxifi cation or
    maintenance treatment are permitted
A
  1. Answer: B (1 & 3)
    Explanation:
    2 and 4 are incorrect. A prescription cannot be issued in
    order for an individual practitioner to obtain controlled
    substances for supplying the individual practitioner for the
    purpose of general dispensing to patients. Schedule II
    drugs cannot be prescribed for narcotic addiction
    treatment.
    Reference: 21 USC 823(g); 21 CFR 1306.04
    Source: Erin Brisbay McMahon, JD, Sep 2005
237
Q

2712.True statements regarding the fi ve schedules of controlled
substances, known as Schedules I, II, III, IV, and V include
all the following:
1. The Schedule I substances have high potential for abuse
and the substance has no currently accepted medical
use in the treatment in the United States.
2. The Schedule I substances may be changed to a lower
schedule if the safety of the drug is demonstrated even
though there is a high potential for abuse and there is
no accepted medical use for medical treatment.
3. The Schedule II drugs have high potential for abuse
and may lead to severe psychological or physical dependence.
4. Schedule V drugs or substances have a high potential
for abuse and may lead to physical or psychological
dependence.

A
  1. Answer: B (1 & 3)
    Explanation:
    1, 2. Schedule I
    The drug or other substance has a high potential for
    abuse.
    The drug or other substances has no currently accepted
    medical use in treatment in the United
    States.
    There is a lack of accepted safety for use of the drug or
    other substance under medical
    supervision.
  2. Schedule II
    The drug or other substance has a high potential for
    abuse.
    The drug or other substances has no currently accepted
    medical use in treatment in the United
    States or a currently accepted medical use with severe
    restrictions.
    Abuse of the drug or other substances may lead to
    severe psychological or physical dependence.
    Other
    Schedule III
    The drug or other substance has a potential for abuse
    less than the drugs or other substances in
    schedules I and II.
    The drug or other substances has no currently accepted
    medical use in treatment in the United
    States.
    Abuse of the drug or other substance may lead to
    moderate or low physical dependence or high
    psychological dependence.
    Schedule IV
    The drug or other substance has a low potential for
    abuse relative to the drugs or other
    substances in schedule III.
    The drug or other substance has a currently accepted
    medical use in treatment in the United
    States.
    Abuse of the drug or other substances may lead to
    limited physical dependence or psychological
    dependence relative to the drugs or other substances in
    schedule III.
  3. Schedule V
    The drug or other substance has a low potential for
    abuse relative to the drugs or other
    substances in schedule IV.
    The drug or other substance has a currently accepted
    medical use in treatment in the United
    States.
    Abuse of the drug or other substances may lead to
    limited physical dependence or psychological
    dependence relative to the drugs or other substances in
    schedule IV.
    Source: Manchikanti L, Board Review 2005
238
Q
  1. Examples of pro-drugs include:
  2. Gabapentin
  3. Morphine
  4. Baclofen
  5. Codeine
A
  1. Answer: D (4 only)
    Explanation:
    Gabapentin, morphine, and baclofen have their primary
    action as the parent drug. Morphine’s metabolite, M6G,
    has analgesic activity, but a pro-drug is a drug whose
    parent compound has no activity. Codeine, which is a prodrug,
    has no activity until it is metabolized into morphine.
    Source: Andrea M. Trescot, MD
239
Q
  1. The following is an accurate statement with regards to
    function of Controlled Substances Act.
  2. It creates a closed system of distribution for those authorized
    to handle controlled substances.
  3. The cornerstone of this system is the licensure of all
    those authorized by the State Medical Licensure Board
    to handle controlled substances.
  4. Only the individuals and practices which dispense directly
    to the patients from their clinics are required to
    maintain a DEA license.
  5. It is required to maintain complete inventory of controlled
    substances, only if the drugs are administered by physician, but not if dispensed to the patient
A
  1. Answer: A (1, 2 & 3)
    Explanation:
  2. The CSA created a closed system of distribution for
    those authorized to handle controlled substances.
  3. The system is the registration of all those authorized by
    the DEA to handle controlled substances.
  4. Only the individuals and practices that dispense directly
    to the patients from their clinics are required to maintain a
    DEA license.
  5. All individuals and fi rms that are registered are
    required to maintain complete and accurate inventories
    and records of all transactions involving controlled
    substances, as well as the security for the storage of
    controlled substances.
    The attorney general may limit revocation or
    suspension of a registration to the particular
    controlled substance. However, the Board of Medical
    Licensure may also limit this indirectly by
    means of requesting the limitation by DEA and
    reaching an agreement with the practitioner.
    Source: Manchikanti L, Board Review 2005
240
Q
2715. If a patient has inadequate relief from an opioid, options
include:
1. increase the dose
2. increase the frequency
3. add a breakthrough medication
4. change medications
A
2715. Answer: E (All)
Explanation:
Other options include changing route and adding adjuvant
meds.
Source: Andrea M. Trescot, MD
241
Q
  1. Mixed opioid agonist-antagonists (nalbuphine,
    pentazocin) have limited use in cancer patients because:
  2. Respiratory depression is a common side effect
  3. Mixed interaction at the opioid receptor can precipitate
    withdrawal symptoms.
  4. Pruritus is a common side effect.
  5. Effectiveness is limited by a dose-related ceiling effect.
A
  1. Answer: C (2 & 4)

Source: Reddy Etal. Pain Practice: Dec 2001, march 2002

242
Q
  1. In accordance with the Federal Controlled Substances
    Act of 1970, which of the following applies towards
    regulating the use of narcotic drugs for opioid
    detoxifi cation or maintenance?
  2. Practitioner is separately registered with the DEA as a
    narcotic treatment program
  3. Practitioner may dispense or prescribe a controlled drug
    in schedules III, IV, or V to a narcotic dependent individual
    for addiction treatment
  4. Practitioner in solo practice will not treat more than
    30 patients at any one time with scheduled drugs for
    detoxifi cation or maintenance
  5. A narcotic drug can not be dispensed for a period in
    excess of 180 days for the purposes of detoxifi cation
A
  1. Answer: E (All)
    Explanation:
    21 U.S.C 823 (g)
    Source: James D. Colson, MS, MD
243
Q

2718.In order to qualify for a waiver under the Drug Abuse
Treatment Act of 2000 to treat opioid addiction with
scheduled drugs, a physician must meet the following
conditions:
1. Hold a current state medical license and valid DEA
number
2. Hold an addiction certifi cation from the American Society
of Addiction Medicine
3. Hold a subspecialty board certifi cation in addiction
psychiatry from the American Board of Medical Specialties
4. Complete at least 8 hours of training in the treatment
and management of opioid-addicted patients

A
2718. Answer: E (All)
Explanation:
http://buprenorphine.sahsa.gov/waiver _qualifi cations
21 U.S.C 823 (g)
Source: James D. Colson, MS, MD
244
Q
  1. Dexmedetomidine and midazolam share the following
    pharmacologic properties:
  2. Provide a continuum of sedation
  3. Preserve respiratory function without potentiating opioid-
    induced respiratory depression
  4. Clearance is decreased in hepatic disease
  5. Selective alpha-2 adrenergic agonist activity
A
  1. Answer: B (1 & 3)
    Explanation:
    Only dexmedetomidine preserves respiratory function
    without potentiating opioid-induced respiratory
    depression.
    Only dexmedetomidine possesses selective alpha-2
    adrenergic agonistic activity, while midazolam works through a GABA2 receptor mechanism.
    Evers AS, Maze M. Anesthetic Pharmacology: Physiologic
    Principles and Clinical Practice. Churchill Livingstone,
    Philadelphia, 2004.
    Morgan GE, Mikhail MS, Murray MJ. Clinical Anesthesia.
    3rd ed. McGraw-Hill, New York, 2002.
    Source: James D. Colson, MS, MD
245
Q
  1. Drug clearance from the plasma at a constant amount
    per unit time and the time for the plasma concentration
    of a drug to decrease by one-half, best represent which of
    the following drug properties?
  2. Pharmacodynamic profi le
  3. Bioequivalence
  4. Drug potency
  5. Metabolism and excretion
A
  1. Answer: D (4 only)
    Explanation:
    Drug clearance and plasma half-life are pharmacokinetic,
    not pharmacodynamic, variables associated with the
    elimination of drug through renal excretion and/or
    hepatic metabolism.
    Bioequivalence compares different formulations of the
    same drug and is not the same as bioavailability or the
    fraction of a drug absorbed following administration.
    Potency is a measure of the amount or dose of drug
    required for an effect.
    Evers AS, Maze M. Anesthetic Pharmacology: Physiologic
    Principles and Clinical Practice. Churchill Livingstone,
    Philadelphia, 2004.
    Hardman JG, Limbird LE. Goodman and Gilman’s
    Pharmacological Basis of Therapeutics. 10th ed. McGraw-
    Hill, New York, 2001.
    Source: James D. Colson, MS, MD
246
Q
  1. The Drug Abuse Treatment Act of 2000 allows for which
    of the following for detoxifi cation treatment?
  2. The use of schedule III drugs to detoxify chemically
    dependent patients in an offi ce setting
  3. The use of opioids in a substance abuser for legitimate
    medical reasons to provide analgesia
  4. The use of opioids for detoxifi cation in a nonabuser,
    who is opioid dependent from legitimate pain therapy
    and desires to discontinue opioid use
  5. Practitioners may administer, dispense, or prescribe a
    controlled drug from any schedule to a narcotic dependent
    individual for addiction treatment
A
  1. Answer: A (1,2, & 3)
    Explanation:
    Practitioner may administer or dispense directly, but not
    prescribe, a narcotic drug in any schedule to a narcotic
    dependent person for the purposes of detoxifi cation or
    maintenance treatment only if the practitioner is
    registered as a narcotic treatment program and compliant
    with DEA regulations
    21 U.S.C 823 (g)
    Source: James D. Colson, MS, MD
247
Q
  1. Which of the following statements about alcohol
    absorption are valid?
  2. Surgical removal of the pylorus allows more rapid absorption
    of alcohol
  3. Most alcohol is absorbed through the gastric mucosa
  4. Secretion of gastric mucus induced by high concentration
    of alcohol delays absorption
  5. The longer the alcohol remains in the blood, the gretaer
    the effect.
A
  1. Answer: B (1 & 3)
    Explanation:
    Alcohol absorption is slowed by food, but increased by
    water, especially if carbonated. Alcohol goes directly into
    the bloodstream from the stomach and it is distributed
    throughout all tissues of the body. If somach alcohol
    concentration becomes too high, mucus is secreted and
    the pyloric valve closes, thereby slowing absorption.
    Source: Psychiatry specialty Board Review By William M.
    Easson, MD and Nicholas L. Rock, MD
248
Q
  1. True statements with regards to properties of NMDA
    antagonists and therapeutic use and misuse are as
    follows:
  2. Phencyclidine or PCP was developed as an intravenous
    anesthetic.
  3. The unique anesthesia produced by phencyclidine was
    associated with rapid emergence.
  4. MK-801 (dizocilpine) was developed as an anticonvulsant
    and subsequently was used as a brain protective
    agent.
  5. Dextromethorphan is a strong NMDA antagonist, even
    when taken in very small amounts.
A
  1. Answer: B (1 & 3)
    Explanation:
    1.Phencyclidine was developed as an intravenous
    anesthetic. However, the unique anesthesia it produced
    was complicated by a prolonged emergence delirium,
    leading to its demise as a clinically useful agent.
    Phencyclidine also caused symptoms of sensory
    deprivation, which is an excellent drug model of
    schizophrenia.
    2.The desirable anesthetic properties of phencyclidine
    were retained in the short-acting arylcyclohexylamine
    derivative ketamine or Ketalar, which produced a much
    briefer emergence delirium. The term “dissociative
    anesthetic” was coined to emphasize the anesthetized
    patient was “disconnected” from his or her environment.
    Ketamine subsequently was discovered by the drug abuse
    community, where it is known as K, Super K, Special K,
    and Kat Valium.
    Phencyclidine has been placed in Schedule I of the Federal
    Controlled Substances Act, and Ketamine in Schedule II.
    3.MK-801 was developed as an anticonvulsant and
    subsequently was used as a brain protective agent.
    However, it was discarded because of its PCP-like effects.
    Clinical trials of MK-801 have been extremely limited,
    and the results are not publicly available.
    4.Dextromethorphan is an antitussive agent. When taken
    in very large amounts, it produces dysphoric mental effects
    that can be related to its weak NMDA antagonistic
    properties.
    5.Amantadine and a related compound, memantidine,
    have been shown to be weak NMDA receptor antagonists.
249
Q
  1. True statements about heroin are as follows:
  2. The chemical name is diacetyl morphine
  3. Heroin is more water soluble but less potent than morphine
  4. Heroin is metabolized in humans by de-acetylation to
    6-mono-acetylmorhpine and morphine
  5. Heroin is classifi ed as Schedule II drug and is widely
    available for therapeutic purposes in the United States.
A
  1. Answer: B (1 & 3)
    Explanation:
  2. Diacetyl morphine or heroine was fi rst synthesized in
  3. It was produced in 1989 by the Bayer Company and
    marketed under the name heroin.
    Heroin is synthesized from morphine by acetylation at
    both the 3 and 6 position.
  4. Heroin is more water soluble and also more potent than
    morphine.
  5. Heroin is metabolized in humans by de-acetylation to
    6-mono-acetylmorphine and then further metabolized to
    morphine
  6. Heroin is classifi ed as Schedule I drug. It is not
    available for any therapeutic use in the United States. It is
    prescribed in a few other countries as a pain medication or
    for use in the management of heroin addiction.
250
Q
  1. Which of the following are true regarding nociceptive
    pain?
  2. Alpha 2 antagonists are useful for management.
  3. Automatic fi ring of damaged nerves is a component.
  4. Deafferentation can produce a fi ring of fi rst order neurons.
  5. Prostaglandin inhibition may be useful for management.
A
  1. Answer: D (4 only)
    Explanation:
    Nociceptive pain is caused by the activation of nociceptors
    in the tissues, and is divided into somatic and visceral
    causes. Deafferentation and automatic fi ring are
    characteristics of neuropathic pain, and respond to alpha 2
    antagonists. NSAIDs, which block prostaglandins, are
    often useful in nociceptive pain.
    Source: Andrea M. Trescot, MD
251
Q
  1. Which of the following is true about state medical
    regulation:
  2. Investigation of physicians is generally the result of complaints received
  3. State Boards may obtain copies of patient records
    without patient consent, upon written request, under
    HIPAA.
  4. State regulations generally prohibit any medical act
    which is an unreasonable danger to the health, safety or
    welfare of patient or public.
  5. A physician has a right to hire a lawyer at any stage of a
    medical board investigation or proceeding.
A
  1. Answer: E (All)

Source: temp

252
Q
  1. A state could, if it chose to do so, do which of the
    following:
  2. Make ordinary negligence in treatment a basis for
    professional discipline, even without any requirement
    of intent.
  3. Require random periodic audits of physician’s practices,
    including patient health care records, to ensure quality
    of care.
  4. Generally outlaw any practice which is not accepted by
    the majority of physicians in the relevant specialty.
  5. Authorize state medical boards to impose very short jail
    terms (less than 30 days).
A
  1. Answer: A (1, 2 & 3)
253
Q
  1. Methadone blood levels are:
  2. increased by cimetidine
  3. decreased by butabutal
  4. increased by Ciprofl oxin
  5. decreased by grapefruit juice
A
  1. Answer: A (1,2, & 3)
    Explanation:
    Grapefruit juice will increase methadone levels.
    Source: Andrea M. Trescot, MD
254
Q

2729.As per the Controlled Substances Act, a denial, revocation,
or suspension of registration may be carried out based on
the following grounds.
1. The physician has failed to inform change of his offi ce
address.
2. Has been convicted of a felony.
3. The physician has reached an agreement with the State
Medical Board of Licensure.
4. Has had his state license or registration suspended, revoked,
or denied.

A
  1. Answer: C (2 & 4)
    Explanation:
    * The attorney general may suspend or revoke a
    registration to manufacture, distribute, or dispense a
    controlled substance upon fi nding that the registrant:
    - Has materially falsifi ed any application fi led.
    - Has been convicted of a felony.
    - Has had his state license or registration suspended,
    revoked, or denied.
    * Has committed such acts as wound render his
    registration inconsistent with the public interest.
    * Has been excluded or directed to be excluded from
    participation in a program pursuant to Section 1128 (A)
    of the Social Security Act.
255
Q
  1. An intervention on a chemically dependent individual
    should include all of the following:
  2. A clear message
  3. Caring for the patient
  4. Planning
  5. Presence of persons important to the patient
A
  1. Answer: E (All)

Source: Stimmel, B

256
Q
  1. If a patient is unable to tolerate oxycodone because of
    nausea, the likely opioids to be tolerated would be:
  2. Fentanyl
  3. Propoxyphene
  4. Methadone
  5. Morphine
A
  1. Answer: A ( 1, 2, & 3)
    Explanation:
    Morphine is in the same class of opioids (phenanthrenes)
    as oxycodone, but morphine has a 6-OH group (associated
    with more nausea). Fentanyl, meperidine, propoxyphene,
    and methadone are completely different classes of opioids
    Source: Trescot AM, Board Review 2004
257
Q

2732.Which of the following are true regarding opioidinduced
constipation in a patient with cancer pain?
1. Impaired defecation refl ex
2. Increased colonic motility
3. Reduced colonic peristaltic activity
4. Bulk laxatives most helpful

A
  1. Answer: A (1,2, & 3)
    Explanation:
    Opioids increase colonic motility but reduce peristaltic
    acitivity; in addition, the defecation refl ex is impaired.
    Bulk laxatives are a poor choice because adequate water
    intake is required and the bulk can make the stool hard
    and precipate impaction.
    Source: Oxford Textbook of Palliative Medicine, 2nd Ed
258
Q
2733. Analgesic agents capable of producing tolerance,
dependence and withdrawal include:
1. Codeine
2. Propoxyphene (Darvon)
3. Buprenorphine
4. Pentazocine (Talwin)
A
  1. Answer: E (All)

Source: Stimmel, B

259
Q
  1. What are true statements about painful crises in sicklecell
    disease:
  2. The pain is rarely severe.
  3. Abdominal crises are frequent.
  4. Joint crises are frequent, accompanied by swelling and
    discoloration of the affected joint.
  5. Analgesics may frequently have to be administered in
    greater than standard doses due to the development of
    tolerance
A
  1. Answer: C (2 & 4)

Source: Stimmel, B

260
Q
  1. The ultimate decisions regarding the specifi c medical
    treatment to be rendered to a patient in a specifi c
    situation are made by:
  2. The physician
  3. The courts
  4. The state legislature
  5. The patient
A
  1. Answer: D (4 only)
261
Q

2736.The CSA requires the following of practitioners who
dispense ONLY manufacturer’s samples of controlled
substances to patients.
1. An initial, and then biennial, inventories.
2. A secure locked box for storage of controlled substances.
3. As separate dispensing log, in addition to any records
kept in the patients’ charts.
4. Complete records of all controlled substances received,
dispensed, or otherwise disposed of.

A
  1. Answer: E (All)

Source: Arthur Thexton

262
Q
  1. The CSA requires the following of practitioners who
    administer controlled substances via any modality to
    patients directly, before or during procedures in the
    offi ce or surgical suite, from physician-owned stock or
    supplies:
  2. An initial, and then biennial, inventories
  3. A secure locked box for storage of controlled substances
  4. As separate dispensing log, in addition to any records
    kept in the patients’ charts.
  5. Complete records of all controlled substances received,
    dispensed, or otherwise disposed of.
A
  1. Answer: E (All)

Source: Arthur Thexton

263
Q
  1. When a state has different rules than the CSA:
  2. Federal constitutional supremacy principles dictate that
    the CSA overrules all state regulations relating to controlled
    substances.
  3. The issue is decided on a case-by-case basis, by the
    courts.
  4. State rules govern, as the states traditionally regulate
    medicine and pharmacy.
  5. Whichever rule is more restrictive must be followed, as
    both have full legal force and effect.
A
  1. Answer: D (4 only)

Source: Arthur Thexton

264
Q

2739.When changing a patient’s controlled substance
medications, a physician may desire a patient to bring
in all unused supplies of discontinued controlled
substances. Which of the following is true?
1. The physician may refuse to prescribe additional controlled
substances until the patient destroys (in a verifi
ed manner) or surrenders all unused meds.
2. Because they are the legal property of the patient, the
physician can do nothing about the patient’s previously
prescribed and dispensed medications.
3. The physician may observe the patient destroy the medications
by fl ushing them down the toilet, unless such
disposal is prohibited by state law.
4. The physician may take possession of unused medications,
inventory them, and send them to the DEA.

A
  1. Answer: B (1 & 3)

Source: Arthur Thexton

265
Q
  1. The following MAY legally be prescribed by a physician
    without a special registration or permission from DEA:
  2. methadone
  3. buprenorphine
  4. injectible C-II morphine
  5. heroin
A
  1. Answer: A (1,2, & 3)
    Explanation:
    There are restrictions on methadone and buprenorphine
    use ONLY when prescribed to treat addiction; when
    prescribed for pain or other conditions, they may be
    prescribed by anyone authorized to prescribe other
    controlled substances in the same schedule.
    Source: Arthur Thexton
266
Q
  1. Mechanisms of drug interactions include:
  2. Drug-drug interactions
  3. Drug allergies
  4. Drug-food interactions
  5. Drug doses
A
  1. Answer: C (2 & 4)
    Explanation:
    Drug interactions include drug-drug, drug-food, and
    drug-condition interactions. Drug allergies infl uence the
    choice of medicines, and drug doses are important in drug
    treatment, but neither are specifi cally related to drug
    metabolism.
    Source: Andrea M. Trescot, MD
267
Q
  1. The choice of opioid medications is infl uenced by:
  2. Frequency of pain
  3. Response to prior opioids
  4. Daily activity
  5. Cost and insurance plan
A
  1. Answer: E (All)

Source: Andrea M. Trescot, MD

268
Q
2743. If a patient is a “slow metabolizer”, possible responses to
medications might include:
1. Increased toxicity
2. Decreased effect
3. Increased effect
4. Decreased excretion
A
  1. Answer: E (All)
    Explanation:
    A slow metabolizer cannot detoxify drugs quickly, leading
    to increased toxicity and possibly increased effect. If the
    drug needs to be metabolized for increased effect (such as
    with a pro-drug), slow metabolizers will have a decreased
    effect. Since metabolism is necessary for most excretion,
    slow metabolizers would have decreased excretion.
    Source: Andrea M. Trescot, MD
269
Q
  1. The following are true statements regarding morphine:
  2. Morphine is primarily renally metabolized
  3. Morphine is primarily renally excreted
  4. Morphine is metabolized by CYP2D6
  5. Morphine is metabolized by glucuronidation
A
2744. Answer: C (2 &amp; 4)
Explanation:
Morphine is primarily hepatically metabolized by
glucuronidation
Source: Andrea M. Trescot, MD
270
Q
  1. Which of the following is a true statement:
  2. Tramadol’s fi rst metabolite has less activity than the
    parent compound
  3. Heroin is metabolized to morphine
  4. Morphine’s metabolites increase in liver failure.
  5. Hydrocodone is metabolized to hydromorphone.
A
2745. Answer: C (2 &amp; 4)
Explanation:
Tramadol’s M1 metabolite has greater activity. Morphine would have decreased metabolism in liver failure, but the
metabolites accumulate in renal failure.
Source: Andrea M. Trescot, MD
271
Q

2746.Medications that should be avoided with grapefruit
include:
1. those with a low oral bioavailablity
2. those metabolized by CYP3A4
3. those with an intestinal transport by p-glycoprotein
4. those metabolized by CYP2D6

A
  1. Answer: A (1,2, & 3)
    Explanation:
    Grapefruit inhibits CYP3A4, not 2D6, and has its action
    by
    altering liver and intestinal 3A4 as well as inhibiting
    intestinal transport by p-GP. Medicines with a low
    bioavailability have the potential for increased absorption.
    Source: Andrea M. Trescot, MD
272
Q

2747.If a patient who is on tramadol is given a CYP2D6
inhibitor,
1. the analgesia of tramadol will decrease
2. the analgesia of tramadol will increased
3. the excretion of tramadol will decrease
4. the excretion of tramadol will increase

A
  1. Answer: B (1 & 3)
    Explanation:
    Tramadol is metabolized by CYP2D6 to an active
    metabolite that has more effect than the parent
    compound, so that CYP2D6 inhibitors cause the effective
    analgesia to decrease. However, CYP2D6 is also the
    enzyme responsible for the excretion of tramadol, so that
    inhibition leads to decreased excretion and the increased
    possibility for seizures.
    Source: Andrea M. Trescot, MD
273
Q

2748.A newly immigrated patient from Viet Nam with
tuberculosis, neuropathy, and acid refl ux disease is being
prescribed methadone for pain following spine surgery
to stabilize a fracture. The metabolism of methadone has
been documented to be affected by:
1. Isoniazid
2. Carbamazepine
3. Cimetidine
4. Ethnicity

A
  1. Answer: A (1,2, & 3)
    Explanation:
    Isoniazid and carbamazepine will increase the metabolism
    of methadone (leading to decreased levels), while
    cimetidine will slow down the metabolism (leading to
    increased levels. While many CYP enzymes show ethnic
    differences, methadone metabolism has not been
    documented to have an ethnic variability.
    Source: Andrea M. Trescot, MD
274
Q
2749. Incipient liver failure due to acetaminophen toxicity will
most affect the metabolism of:
1. Lidocaine
2. Methadone
3. Codeine
4. Meperidine
A
  1. Answer: B (1 & 3)
    Explanation:
    Lidocaine is highly dependant upon liver excretion, while
    methadone is not. Codeine is metabolized to morphine,
    which is highly dependant upon liver excretion (though
    M3G accumulates in renal insuffi ciency), while
    meperidine requires renal excretion.
    Source: Andrea M. Trescot, MD
275
Q
  1. Signifi cant drug-food interactions include:
  2. Coffee and tea are more rapidly metabolized in the presence
    of ciprofl oxin.
  3. Phenobarbital is useful to treat folate defi ciency neuropathy.
  4. Smokers have a greater perceived effect of propoxyphene
    than non- smokers.
  5. NSAIDS cause greater renal damage in patients with
    high dietary fat intake.
A
  1. Answer: D (4 only)
    Explanation:
    Caffeine (in coffee and tea) is metabolized by CYP1A2,
    which is inhibited by ciprofl oxin. Phenobarbital will
    increase the risk of folate defi ciency neuropathy. Smokers
    metabolize propoxyphene faster and therefore have less
    effect.
    Source: Andrea M. Trescot, MD
276
Q
  1. Studies have shown:
  2. Daily pain is present in one third of the population over
    65 years old
  3. Moderate to severe non-cancer pain is present in 9% of
    the US population.
  4. One-thirds of chronic pain patients have been living
    with pain for more than 5 years.
  5. Only 15% of primary care physicians enjoy taking care
    of pain patients.
A
  1. Answer: C (2 & 4)
    Explanation:
    55% of American adults over 65 have pain daily.
    The Arthritis Foundation, “Pain In America: Highlights
    from a Gallup Survey,” 2000
    9% of the US population suffers from moderate to severe
    pain, and 2/3rds have suffered for more than 5 years.
    Roper Starch Worldwide, Inc., “Chronic Pain In America:
    Roadblocks To Relief,” Jan. 1999
    Only 15% of PCPs enjoy pain patients
    Potter M, Schafer S, et al. Opioids for chronic
    nonmalignant pain: attitudes and practices of primary
    care physicians. J Fam Pract 2001;50(2):145-51
    Source: Andrea M. Trescot, MD
277
Q
2752.Epidemiologic risks for work-related low back pain
include:
1. Prior WC claim
2. Perceived over-education
3. Cumulative compressive back forces
4. Peak hand forces
A
  1. Answer: E (All)
    Explanation:
    Norman R, Wells R, Neumann P, et al. A comparison of
    peak vs cumulative physical loading factors for reported
    low back pain in the automobile industry, Clinical
    Biomechanics, 13(8): 561-573, 1998.
    Source: Andrea M. Trescot, MD
278
Q
  1. Regarding cancer pain:
  2. There are 1 million cancer patients world-wide
  3. 10% of cancer patients in active treatment report pain
  4. Half of advanced cancer patients complain of pain
  5. More than one-third of oncology outpatients complain
    of pain that interferes with ADLs
A
  1. Answer: D (4 only)
    Explanation:
    There are 17million cancer patients world-wide. 30 to
    40% of cancer patients in active treatment report pain.70
    to 90% of advanced cancer patients complain of pain. 36%
    of oncology outpatients complain of pain that interferes
    with ADLs
    Grond S, Sech D, et al Assessment of cancer pain: a
    prospective evaluation in 2266 cancer patients referred to a
    pain service. Pain (1996), 64, 107-114.
    Source: Andrea M. Trescot, MD
279
Q

2754.The following statements are true regarding the
paleospinothalamic tract:
1. The paleospinothalamic tract is poor in opioid receptors.
2. The paleospinothalamic tract connects the thalamus to
the cortex.
3. The paleospinothalamic tract passes impulses from the
2nd order neurons to the C-fi bers.
4. The paleospinothalamic tract connects the thalamus
and the reticular activating system.

A
  1. Answer: C (2 & 4)
    Explanation:
    The paleospinothalamic tract is rich in opioid receptors,
    connects the thalamus to the cortex and reticular
    activation system, and passes impulses from the C-fi bers to
    the 2nd order neurons.
    Source: Andrea M. Trescot, MD
280
Q
  1. What are the important aspects of chronic pain?
  2. Management is inexpensive
  3. Multiple regions are involved
  4. Low economic impact
  5. Persistent pain in 30% - 70% of patients after initial
    attack
A
  1. Answer: C (2 & 4)
    Explanation:
    Chronic Pain
    Multiple region involvement
    Costly
    Chronic, persistent pain 30% - 70% after initial attack
    High economic impact
    Patient suffering - dysfunction, immunosuppression,
    depression
    Health care - extensive costly non-productive workups
    and treatments - $145 billion per year just for low back
    Societal - disability, loss of revenues
    Source: Laxmaiah Manchikanti, MD
281
Q

2756.When choosing an opioid, factors to be considered
include:
1. patient compliance
2. dosing schedule of concurrent medications
3. drug interactions
4. opioid side effects

A
  1. Answer: E (All)
    Explanation:
    A patient compliance problem might suggest the choice of
    a once a day medication or patch. If a patient is on other
    BID mediations, a BID pain medication might be the best
    choice. Drug interactions can affect choice of medication,
    such as when a patient is already on Fiorinal, which will
    decrease methadone levels. Side effects such as nausea
    from hydroxylated opioids or constipation that might be
    lessoned by a patch can also infl uence medication choice.
    Source: Andrea M. Trescot, MD
282
Q
2757. Urine screening of patients should be able to detect each
of the following:
1. Cocaine
2. Morphine
3. Alcohol
4. Barbiturates
A
  1. Answer: E (All)

Source: Stimmel, B

283
Q
  1. What are the clinical recommendations in chronic
    opioid therapy?
  2. Daily doses above 180 mg/day of morphine have not
    been validated
  3. Dose escalation beyond the stabilization phase may
    predict a problem
  4. Opioid rotation may be helpful
  5. Drug formulation does not infl uence tolerance
A
2758. Answer: E (All)
Explanation:
Clinical Recommendations
* Limit the dose
- Daily doses above 180 mg/day of morphine have not
been validated
- Dose escalation beyond the stabilization phase may
predict a problem
* Drug formulations
- Formulation does not infl uence tolerance
* Opioid rotation
- Concept of incomplete cross-tolerance
- Rotation may restore effi cacy
Source: Mark V. Boswell, MD, KSIPP 2005
284
Q
  1. Which one of the following results may occur from
    repeated administration of a drug?
  2. Increased metabolism of the drug
  3. Increased metabolism of other drugs
  4. Induction of cytochrome P-450 or glucuronyl transferase
  5. Increased metabolism of endogenous compounds
A
  1. Answer: E (All)
    Explanation:
    All of the choices are possible consequences of drug
    administration. Many enzymes involved in drug
    biotransformation also catalyze the metabolism of
    endogenous compounds such as steroids.
285
Q
2760.Symptoms of opioid toxicity include which of the
following?
1. Leg muscle twitching
2. Pulmonary edema
3. Seizures
4. Hypothermia
A
  1. Answer: C (2 & 4)
    Explanation:
    Opioid toxicity or overdose should be suspected in any
    undiagnosed coma patient or patients with respiratory
    depression (pulmonary edema), shock (hypothermia),
    pupillary construction, and needle marks. Grand mal
    seizures can occur with meperidine overdose.
    Source: Psychiatry specialty Board Review By William M.
    Easson, MD and Nicholas L. Rock, MD
286
Q
  1. Choose the correct statements about cocaine testing.
  2. A patient’s urine may test positive for the cocaine metabolite
    benzoylecgonine after a procedure with cocaine
    as a topical anesthetic for up to 3 to 4 weeks.
  3. Cocaine, a topical anesthetic, is clinically used in certain
    trauma, dental, ophthalmoscopic, and otolaryngologic
    procedures.
  4. There is structural similarity between other topical anesthetics
    that end in “caine” (eg, Novocaine, lidocaine)
    and cocaine or benzoylecgonine.
  5. A positive UDT result for the cocaine metabolite, in the
    absence of a medical explanation, should be interpreted
    as due to deliberate use.
A
  1. Answer: C (2 & 4)
    Explanation:
    Urine Drug Test Methods: Cocaine
    Cocaine, a topical anesthetic, is clinically used in certain
    trauma, dental, ophthalmoscopic, and otolaryngologic
    procedures.
    A patient’s urine may test positive for the cocaine
    metabolite benzoylecgonine after such a
    procedure for up to 2 to 3 days.
    There is no structural similarity between other topical
    anesthetics that end in “caine” (eg,
    Novocaine, lidocaine) and cocaine or benzoylecgonine;
    therefore, cross-reaction does not occur.
    A positive UDT result for the cocaine metabolite, in the
    absence of a medical explanation, should
    be interpreted as due to deliberate use.
    Source: Laxmaiah Manchikanti, MD
287
Q
  1. Which of the following are true about lorazepam:
  2. Has a serum half-life of approximately 12 hours
  3. Exhibits linear kinetics
  4. Is almost completely converted to benzoylecgonine
  5. Is frequently used an anti-anxiety agent
A
  1. Answer: C (2 & 4)
    Explanation:
    Half life is 12 hours, as compared to diazepam which is
    21-37 hours. Is indicated for short term use of anxiety. The
    kinetics are linear.
    It is converted to lorazepam glucuronide (75%) abd NOT
    to benzoylecgonine, a major metabolite of cocaine.
    Reference: Disposition of Toxic Drugs and Chemicals in
    Man. Fifth Edition. Randall C. Baselt. 2000 page 483
    Source: Art Jordan, MD, Sep 2005
288
Q

2763.What are the correct statements of amphetamine in
urine drug testing?
1. Tests for amphetamine/methamphetamine are highly
cross-reactive.
2. Very predictive for amphetamine/methamphetamine
use.
3. UDT will detect other sympathomimetic amines such as
ephedrine and pseudoephedrine
4. Further testing is NOT required.

A
2763. Answer: B (1 &amp; 3)
Explanation:
Urine Drug Test Methods
Amphetamines: Low Specifi city
Tests for amphetamine/methamphetamine are highly
cross-reactive.
They will detect other sympathomimetic amines such as
ephedrine and pseudoephedrine
Not very predictive for amphetamine/methamphetamine
use.
Further testing is required.
Source: Laxmaiah Manchikanti, MD
289
Q
  1. What are the pitfalls of opioid urine drug testing?
  2. Tests for opiates are very responsive for morphine and
    codeine.
  3. Urine drug tests do not distinguish between morphine
    and codeine.
  4. UDT’s show a low sensitivity for semisynthetic/synthetic
    opioids such as oxycodone.
  5. A negative response excludes oxycodone and methadone
    use.
A
2764. Answer: A (1,2, &amp; 3)
Explanation:
Urine Drug Test Methods
Opioids: Pitfalls
Tests for opiates are very responsive for morphine and
codeine
Do not distinguish which is present.
Show a low sensitivity for semisynthetic/synthetic opioids
such as oxycodone.
A negative response does not exclude oxycodone, or
methadone use.
Source: Laxmaiah Manchikanti, MD
290
Q
  1. Defi nitions of addiction include the following:
  2. A primary, chronic neurobiologic disease with genetic,
    psychosocial and environmental factors effecting its
    course and presentation
  3. Characterized by either impaired control of drug use or
    other symptoms
  4. Addiction involves loss of control, craving, compulsive
    use, and continued use despite consequences
  5. DSM-IV defi nition defi nes addiction to involve impaired
    control of drug use
A
2765. Answer: A (1,2, &amp; 3)
Explanation:
Addiction: Defi nitions
DSM-IV - None
A primary, chronic neurobiologic disease with
genetic,psychosocial and environmental factors
effecting its course and presentation
Characterized by one or more of the following
Impaired control of drug use
Compulsive use
Craving
Continued use despite harm
4-Cs
Loss of Control
Craving
Compulsive Use
Continued use despite consequences
Source: Laxmaiah Manchikanti, MD
291
Q

2766.Choose the correct statements about codeine and
morphine
1. Prescribed morphine cannot account for the presence
of codeine.
2. Codeine metabolizes to morphine.
3. Codeine alone is possible due to a small proportion of
patients who lack the cytochrome P450 2D6 enzyme
necessary to convert codeine to morphine.
4. Morphine metabolizes to codeine

A
  1. Answer: A (1,2, & 3)
    Explanation:
    Urine Drug Testing
    Opioid Results: Codeine and Morphine
    Codeine is metabolized to morphine, so both
    substances may occur in urine following codeine use:
    A prescription for codeine may explain the presence
    of both drugs in the urine.
    A prescription for codeine does not normally explain
    the presence of only morphine.
    This is most consistent with: Use of morphine or
    heroin.
    Prescribed morphine cannot account for the presence of
    codeine.
    Codeine metabolizes to morphine, but the reverse
    does not occur.
    Codeine alone is possible due to a small proportion of
    patients who lack the cytochrome P450 2D6 enzyme
    necessary to convert codeine to morphine.
    Source: Laxmaiah Manchikanti, MD
292
Q
  1. Withdrawal convulsions are likely to occur in patients
    who have used chronically which of the following drugs?
  2. Secobarbital
  3. Desipramine
  4. Lorazepam
  5. Phencyclidine
A
  1. Answer: B (1 & 3)
    Explanation:
    Withdrawal convulsions can occur with alcohol, certain
    benzodiazepines, and barbiturates.
    Source: Psychiatry specialty Board Review By William M.
    Easson, MD and Nicholas L. Rock, MD
293
Q

2768.Which of the following statements about
biotransformation reactions are true?
1. They may introduce an active center for further conjugations
2. They almost always yield water-soluble metabolites
3. They are often important in activating pro-drugs
4. They are all inducible upon repeated drug administration

A
  1. Answer: A (1,2, & 3)
    Explanation:
    Phase II reactions typically yield water-soluble
    metabolites. Active centers are introduced during phase I,
    not phase II, biotransformation. Glucuronyl transferase is
    the only phase II enzyme inducible by drug
    administration.
    Prodrugs are often activated by phase I, not phase II,
    reactions; phase II reactions generally terminate drug
    action.
294
Q

2769.What are the guidelines of the Federation of State
Medical Boards (FSMB), adapted by multiple State
Medical Boards?
1. Opioids are used for a legitimate medical purpose
2. Opioids are used for documented abnormalities on
MRI
3. Opioids are used in the course of professional practice
4. Opioids are used if another physician has previously
written prescriptions for controlled substances for the
patient

A
  1. Answer: B (1 & 3)
    Explanation:
    A documented abnormality/pathology on MRI may be an
    incidental fi nding and not causing pain. You should make
    your decision based on examination, previous medical
    records, imaging studies, and careful evaluation.
    Just because another physician has been dispensing
    narcotics to a patient, does not necessarily mean it was
    appropriate or legal. The dispensing/prescribing must be
    based on a legitimate medical purpose as determined by
    the
    current prescribing physician, after careful evaluation and
    examination.
    Source: Art Jordan, MD, Sep 2005
295
Q
  1. What are the requirements according to the FSMB Policy
    Guidelines, to prescribe controlled substances within the
    course of professional practice?
  2. A physician-patient relationship
  3. Diagnosis and documentation of unrelieved pain
  4. Compliance with applicable state or federal law
  5. Urine drug testing (UDT) at least every six months
  6. The accurate statements about prescription drug
    abuse for non-medical purposes include the following:
  7. Prescription drug abuse, particularly of opioid pain
    killers, has increased at an alarming rate over the last
    ten years.
  8. Non-medical use of narcotic pain relievers, tranquilizers,
    stimulants, and sedatives ranks second (behind
    marijuana) as a category of illicit drug abuse among
    adults and youth.
  9. In 2003, 6.3 million Americans were current abusers of
    prescription drugs, with 4.7 million using pain relievers.
  10. E.R. visits for Benzodiazepine abuse in 2002 were
    100,000 and for 0pioid abuse in 2002 were 119,000
A
  1. Answer: A (1,2, & 3)
    Explanation:
    Urine drug testing may be helpful, however is not required
    by law nor in the usual course of professional practice.
    Source: Art Jordan, MD, Sep 2005
296
Q
  1. What are the true statements of drug abuse in the United
    States?
  2. In lifetime, 46% of persons aged 12 and older used
    drugs
  3. In past month, 8.3% of persons aged 12 and older used
    drugs
  4. In past year, 14.9% of persons aged 12 and older used
    drugs
  5. Prescription opioids are not abused
A
  1. Answer: A (1,2, & 3)
    Explanation:
    NEED SLIDE 13
    Source: Laxmaiah Manchikanti, MD
297
Q
2773.Which of the following effects is associated with
benzodiazepines?
1. Paradoxical excitement
2. Ataxia
3. Sedation
4. Amnesia
A
  1. Answer: E (All)
    Explanation:
    Short-acting benzodiazepines are used as preanesthetic
    medications because of their anxiolytic, sedative, and
    amnestic effects. Daytime drowsiness and ataxia are
    commonly produced by benzodiazepines and may impair
    judgment and interfere with motor skills. Paradoxical
    excitement is a rare adverse effect of these drugs.
298
Q
  1. Morphine is used therapeutically
  2. To suppress the withdrawal syndrome associated with
    the chronic use of alcohol
  3. To induce miosis
  4. To treat severe constipation
  5. To relieve pain associated with a heart attack
A
  1. Answer: D (4 Only)
    Explanation:
    Morphine is used to relieve the pain associated with
    myocardial infarction. It can suppress the opioid
    withdrawal syndrome but not the withdrawal syndrome
    associated with other classes of central nervous system
    depressants. Morphine and other opioids induce
    constipation and can be used to treat diarrhea. Miosis is
    an adverse effect of morphine.
299
Q
  1. Medical Records should include which of the following:
  2. Treatment objectives
  3. Instructions and agreements
  4. Periodic reviews
  5. Financial contracts
A
  1. Answer: A (1,2, & 3)
    Explanation:
    Financial contracts are not included.
    In addition, the complete listing includes:
    1: Medical history and physical examination
    2: Diagnostic, therapeutic and laboratory results
    3: Evaluations and consultations
    4: Discussion of risks and benefi ts
    5: Informed consent
    6: Treatments
    7: Medications (including date, type, dosage and quantity
    prescribed)
    Source: Art Jordan, MD, Sep 2005
300
Q

2776.For purposes of ordering a Schedule II emergency
oral prescription, an emergency situation exists if the
prescribing physician determines . . .:
1. That immediate administration of the controlled substance
is necessary for proper treatment of the intended
ultimate user.
2. That no appropriate alternative treatment is available,
including administration of a drug which is not a controlled
substance under schedule II.
3. That it is not reasonably possible for the prescribing
physician to provide a written prescription to be presented
to the person dispensing the substance, prior to
the dispensing.
4. That the patient complains of extreme and unremitting
pain.

A
  1. Answer: A (1,2, & 3)
    Explanation:
    Answer (4) is wrong as the physician does not need to
    determine that the patient is complaining of extreme and
    unremitting pain before fi nding that an emergency
    situation exists.
    Reference: 21 CFR 290.10, 1306.11(d).
    Source: Erin Brisbay McMahon, JD, Sep 2005
301
Q

2777.Which of the following statements concerning
barbiturate is true?
1. Barbiturates can increase bleeding time when administered
to patient taking anticoagulants
2. Patients tolerant to the therapeutic actions of barbiturates
are also tolerant to the analgesic effect of
morphine
3. Barbiturates are used to prevent withdrawal symptoms
associated with heroin dependence
4. Barbiturates are contraindicated in patients with acute
intermittent porphyria

A
  1. Answer: D (4 Only)
    Explanation:
    Barbiturates induce liver microsomal enzymes that
    increase porphyrin synthesis and increase the metabolism
    and inactivation of certain anticoagulants. Barbiturates
    show cross-dependence with other sedative-hypnotic
    drugs but not with opioids
302
Q
  1. Which one of the following conditions may be produced
    by frequent administration of high-dose chlorpromazine
    (Thorazine®) ?
  2. Lens opacities
  3. Skin pigmentation
  4. Obstructive jaundice
  5. Tardive dyskinesia
A
  1. Answer: E (All)
    Explanation:
    Chlorpromazine is associated with all of these adverse
    effects. Because of its numerous adverse effects, some
    authorities consider it to be an obsolete antipsychotic
    agent.
303
Q

2779.Which of the following effects is produced by morphine?
1. Relief of dyspnea accompanying pulmonary edema
2. Decreased sensitivity of the respiratory center to carbon
dioxide
3. Miosis that can be blocked by atropine
4. Vasodilation of cerebral blood vessels

A
  1. Answer: E (All)
    Explanation:
    Morphine produced all of the above effects. It decreases
    the sensitivity of the respiratory center to carbon dioxide
    and directly inhibits the respiratory center, leading to
    respiratory depression. This effect on respiration may be
    responsible for morphine’s benefi cial effect when used to
    treat dyspnea and for cerebral vasodilation. Morphine
    produced miosis by stimulation or the Edinger-Westphal
    nucleus of the oculomotor nerve; this is mediated by
    acetylcholine and can be blocked by atropine.
304
Q
  1. What precautions must a physician take in interpretation
    of urine drug testing?
  2. Consult with laboratory regarding ANY unexpected
    results.
  3. Never use results to strengthen physician-patient relationship
    and support positive behavior change.
  4. Schedule an appointment to discuss abnormal/
    unexpected results with the patient; discuss in a
    positive, supportive fashion to enhance readiness to
    change/motivational enhancement therapy (MET) opportunities.
  5. It is not necessary to document results and interpretation
A
  1. Answer: B (1 & 3)
    Explanation:
    UDT results:
    Consult with laboratory regarding ANY unexpected
    results.
    Schedule an appointment to discuss abnormal/unexpected
    results with the patient; discuss in a positive, supportive
    fashion to enhance readiness to change/motivational
    enhancement therapy (MET) opportunities.
    Use results to strengthen physician-patient relationship
    and support positive behavior change.
    Chart results and interpretation.
    Source: Laxmaiah Manchikanti, MD
305
Q
  1. Which of the following effects if produced by tricyclic
    antidepressant drugs?
  2. Increase in the antihypertensive effect of guanethidine
  3. Hypertensive crisis
  4. Increased absorption of an oral dose of levodopa
  5. Precipitation of narrow-angle glaucoma
  6. What are the correct statements about UDT of
    cocaine?
  7. Tests for cocaine react principally with cocaine and its
    primary metabolite, benzoylecgonine.
  8. Tests for cocaine are non-specifi c in predicting cocaine
    use.
  9. Tests for cocaine have low cross-reactivity with other
    substances
  10. Cold medicines may test false-positive for cocaine
A
  1. Answer: D (4 Only)
    Explanation:
    Tricyclic antidepressants can precipitate narrow-angle
    glaucoma through their muscarinic-cholinoreceptor
    antagonist activity. They may cause hypotension and may
    block neuronal uptake of guanethidine and thus decrease
    its antihypertensive action. They may also decrease the
    gastrointestinal absorption of levodopa.
  2. Answer: B (1 & 3)
    Explanation:
    Urine Drug Test Methods
    Cocaine: Very specifi c
    Tests for cocaine react principally with cocaine and its
    primary metabolite, benzoylecgonine.
    These tests have low cross-reactivity with other substances
    Very specifi c in predicting cocaine use.
    Source: Laxmaiah Manchikanti, MD
306
Q
  1. According to the guidelines of the F.S.M.B.:If the patient
    is at high risk for medication abuse or has a history of
    substance abuse, the physician should consider the use
    of a written agreement between physician and outlining
    patient responsibilities. This agreement should include
    which of the following:
  2. Reasons for which drug therapy may be discontinued
    (e.g., violation of agreement)
  3. Requirement for medical interview with members of
    immediate family
  4. Urine/serum medication levels screening when requested
  5. Periodic reports from a local law enforcement agency
A
  1. Answer: B (1 & 3)
    Explanation:
    Interviews with family members may be helpful under
    certain circumstances if clinically indicated and if within
    legal bounds of privacy, however, this is not listed by the
    Board.
    Periodic reports from law enforcement, without patient
    consent, would be inappropriate and possibly a violation
    of
    privacy laws.
    Source: Art Jordan, MD, Sep 2005
307
Q
  1. A triangle of delivery exists between the following:
  2. Pharmacists
  3. The Physician
  4. Drug Enforcement Administration
  5. Federation of State Medical Boards
A
  1. Answer: A (1,2, & 3)
    Explanation:
    The physician initiates a prescription based on legitimate
    medical need. Once legitimate medical need of a
    controlled substance is established, the patient moves the
    prescription to a pharmacist, who is charged with
    dispensing. The third side of the triangle is the Drug
    Enforcement Administration, which monitors availability.
    The DEA’s responsibility is not to defi ne legitimate
    medical need, or to establish appropriate usage guidelines,
    but to ensure availability of drug.
    Source: Hans C. Hansen, MD
308
Q
  1. Which of the following statements about the symptoms
    of barbiturate withdrawal are correct?
  2. Develops more quickly with secobarbital then phenobarbital
    abusers
  3. Cardiovascular collapse may be fatal
  4. Abdominal discomfort, nausea, and vomiting
  5. Seizures generally precede delirium
A
  1. Answer: E (All)
    Explanation:
    Barbiturate withdrawal (especially short acting) usually
    results in weakness, insomnia, anxiety, tremulousness,
    abdominal discomfort, nausea and vomiting. With
    preexisting cardiovascular problems, there may be fatal
    reactions. Seizures generally precede delirium. Symptoms
    are more marked with secobarbital an dleast with
    phenobarbital withdrawal (due to its long half-life).
    Source: Psychiatry specialty Board Review By William M.
    Easson, MD and Nicholas L. Rock, MD
309
Q
  1. When a person taking a medication or abusing a drug
    develops tolerance, which of the following statements are
    valid?
  2. The same dosage of the drug has reduced effect
  3. Tolerance develops uniformly to all effects of the drug
  4. Physical dependence tends to develop in parallel with
    tolerance
  5. Withdrawal symptoms are less likely after tolerance has
    developed
A
  1. Answer: B (1 & 3)
    Explanation:
    Tolerance occurs when the same dosage of drug has a
    reduced effect and increased amounts of the drug are
    needed to achieve the desired effect. Physical dependence,
    the need to take the drug to prevent withdrawal, tends to
    develop in parallel with tolerance.
    Source: Psychiatry specialty Board Review By William M.
    Easson, MD and Nicholas L. Rock, MD
310
Q

2787.Which of the following statements are correct about
heroin abuse?
1. The peak incidence is age 25 to 35
2. It affects men three times as often as women
3. The majority of heroin abusers are involved in maintenance
programs
4. Most heroin abusers eventually stop on their own

A
  1. Answer: C (2 & 4)
    Explanation:
    Heroin abusers tend to start in late teens and early 20s
    (most common 18-25 years old), with the majority in the
    mid-30s. There is a 3:1 male to female ratio. Suicide in
    abusers is three times greater than in the general
    population. They also have a 20 times greater death rate,
    as well as higher rates of hepatitis B and HIV III viral
    infections.
    Source: Psychiatry specialty Board Review By William M.
    Easson, MD and Nicholas L. Rock, MD
311
Q
2788.Symptoms of heroin withdrawal include all of the
following :
1. Twitching movements in the legs
2. Dilated pupils
3. Increased blood pressure
4. Diarrhea
A
  1. Answer: E (All)
    Explanation:
    Heroin withdrawal symptoms are similar to a infl uenzalike
    syndrome along with anxiety and dysphoria. Physical
    symptoms include yawning, sweating, rhinorrhea,
    lacrimation, pupillary dilation, piloerection, hypertension,
    waves of goosefl esh, twitching movements, deep muscle
    and joint pains, nausea, diarrhea, vomiting, abdominal
    pains, fever, and hot and cold fl ashes.
    Source: Psychiatry specialty Board Review By William M.
    Easson, MD and Nicholas L. Rock, MD
312
Q

2789.In the management of detoxifi ed substance-abusing
patients in a therapeutic community, poor prognosis is
more likely with:
1. Coexisting severe psychopathology
2. Dropout before three months
3. Continued alcohol use
4. Adjunctive use of antidepressants

A
  1. Answer: A (1, 2, & 3)
    Explanation:
    Therapeutic communities for substance abusers have as
    their goals a complete change of lifestyle and abstinence
    from drugs. If the patient’s stay is more than 90 days, there
    is a long-term decrease in illicit drugs use, antisocial
    behavior, and arrests, and increased employment. With a
    12-month stay, subjects fare even better at fi ve years postprogram
    follow-up.
    Source: Psychiatry specialty Board Review By William M.
    Easson, MD and Nicholas L. Rock, MD
313
Q

2790.DSM-IV defi nition of substance dependence includes
some of the following items as part of at least 3 in 12
months.
1. Larger amounts/longer periods
2. Patient spends large amount of time using/obtaining
controlled substances
3. Activities given up: social/work/recreation
4. Continued use despite problems

A
2790. Answer: E (All)
Explanation:
SUBSTANCE DEPENDENCE: DSM-IV DEFINITION
Need 3 in 12 months
Tolerance
Withdrawal
Larger Amounts/Longer periods
Efforts or desire to cut down
Large Amount of time using/obtaining/recovering
Activities given up: social/work/recreation
Continued use despite problems
Source: Laxmaiah Manchikanti, MD
314
Q

2791.Physicians should not fear regulatory action from
the Board for ordering, prescribing, dispensing or
administering controlled substances, including opioid
analgesicsChoose correct statements described in
model policy for the use of controlled substances for the
treatment of pain by Federation of State Medical Boards:
1. For a legitimate medical purpose
2. For documented abnormalities/pathology on M.R.I.
Scan
3. In the course of professional practice
4. If another physician has previously written prescriptions
for controlled substances for the patient

A
  1. Answer: B (1 & 3)
    Explanation:
    A documented abnormality/pathology on MRI may be an
    incidental fi nding and not causing pain . . . treat the
    patient, not the MRI.
    Just because another physician has been dispensing
    narcotics to a patient, does not necessarily mean it was
    appropriate or legal. The dispensing/prescribing must be
    based on a legitimate medical purpose as determined by
    the current prescribing physician, after careful evaluation
    and examination.
    Source: Art Jordan, MD, Sep 2005
315
Q
  1. As a pain physician, you are evaluating a new patient
    who recently moved to your area. Consistent with your
    usual offi ce policy, all new patients get a urine drug test.
    The patient’s urine is positive for morphine, however
    his medication list shows no listing for any controlled
    substances.Which of the following are possible sources
    for this fi nding?
  2. Morphine use
  3. Heroin use
  4. Poppy seeds
  5. Codeine
A
  1. Answer: E (All)
    Explanation:
    The opiate immunoassay screens were designed to detect
    heroin abuse, not adherence to a therapeutic opioid
    regimen. These assays detect morphine and
    codeine—heroin is rapidly metabolized to 6-
    monoacetylmorphine (6-MAM) and then to morphine.
    Performing opiate immunoassays at the federally
    mandated level of 2000 ng/ml should eliminate nearly all
    positive results due to morphine from foodstuffs. Only
    specifi c detection of 6-MAM by GC/MS is proof of heroin
    intake. Street heroin may be contaminated with codeine.
    Remember that codeine may be metabolized to morphine.
    Hydrocodone can also be produced as a minor metabolite
    of codeine. J Anal Toxicol 2000;24:530-535
    Source: Art Jordan, MD, Sep 2005
316
Q
  1. A signifi cant minority of persons with substance abuse
    disorder have and atypical course and will eventually
    either stop using or be able to return to controlled use.
    Which of the following factors are associated with being
    able to stop or control use?
  2. Stable premorbid personality
  3. Developing medical complications of substance abuse
  4. Age greater than 40
  5. Arrest and incarceration
A
  1. Answer: B (1 & 3)
    Explanation:
    Those who either stop or control drug use after a period of
    addiction are more likely to be older than age 40, have a
    normal premorbid personality, never be arrested for
    substance abuse, and to undergo substance abuse
    treatment / rehabilitation. Developing medical
    complications from abuse rarely leads to abstinence.
    Source: Roger Cicala, MD, Sep 2005
317
Q

2794.A 29-year old patient whom you had been treating
for postlaminectomy syndrome with only time release
morphine, 120 mg per day, had a urine drug screen
positive for cocaine and benzodiazepines. When this
was discussed with the patient, he admitted getting
medications from a number of physicians and to
injecting cocaine intravenously, but refused evaluation
by an addictionologist and was dismissed from your
practice. 3 days later an emergency room physician calls
because the patient has been admitted following his fi rst
ever grand mal seizure. Which of the following are likely
causes of the seizure?
1. Cocaine overdose
2. Opiate withdrawal
3. Benzodiazepine withdrawal
4. Cocaine withdrawal

A
  1. Answer: B (1 & 3)
    Explanation:
    Seizure can occur with stimulant overdose or
    benzodiazepine withdrawal. It is not a reported part of the
    syndrome of opiate withdrawal or of cocaine withdrawal.
    Given the positive drug screen for benzodiazepines and cocaine, either scenario is possible.
    Source: Roger Cicala, MD, Sep 2005
318
Q
  1. An otherwise healthy 45 year old patient taking 100 mg
    of morphine per day for pain associated with rheumatoid
    arthritis has decided to stop his pain medication 3 days
    prior to seeing you in the offi ce. Which of the following
    symptoms indicate severe withdrawal that probably
    requires treatment:
  2. Fever
  3. Headache
  4. Hypertension
  5. Rhinorrhea
A
  1. Answer: B (1 & 3)
    Explanation:
    All of the above are symptoms of opiate withdrawal along
    with numerous others. Fever, Hypertension, and
    Tachycardia are considered the most consistent symptoms
    indicated severe withdrawal reaction.
    Source: Roger Cicala, MD, Sep 2005
319
Q

2796.In obtaining a history during the evaluation of a new
patient, which of the following would be considered risk
factors for possible substance abuse?
1. History of substance abuse in a brother and father.
2. History of psychiatric problems in the mother
3. History of bipolar illness in the patient
4. Age over 40 years old

A
  1. Answer: A (1,2, & 3)
    Explanation:
    Personal and family history of substance abuse, personal
    and family history of psychiatric illness, male gender, age
    under 40, and nonmarried status are all associated with
    increased prevalence of substance abuse.
    Source: Roger Cicala, MD, Sep 2005
320
Q
  1. According to the ASAM, APS, AAPM consensus statement
    on the use of opioids for the treatment of chronic pain,
    which of the following characterize addiction?
  2. Impaired control over drug use
  3. Continued use despite harm
  4. Craving
  5. Compulsive drug use
A
2797. Answer: E (All)
Explanation:
The above are the 4 listed characteristics in the Consensus
Statement.
Source: Roger Cicala, MD, Sep 2005
321
Q
  1. What is the defi nition of addiction as per the Controlled
    Substances Act?
  2. The term “addict” means any individual who habitually
    uses any narcotic drug so as to endanger the public
    morals, health, safety, or welfare
  3. “Addict” is a patient who is taking as per prescription
    very high opioids in cancer pain
  4. Any individual who is so far addicted to the use of narcotic
    drugs as to have lost the power of self-control with
    reference to his addiction.
  5. “Addict” is a patient taking controlled prescription
    drugs
A
  1. Answer: B (1 & 3)

Source: Laxmaiah Manchikanti, MD

322
Q
2799. If urine tampering is suspected, the following should be
included in urine drug testing (UDT)
1. Temperature
2. Creatinine
3. PH
4. Color
A
  1. Answer: E (All)
    Explanation:
    The temperature should be 90-100 degrees F. within 4
    minutes of voiding. The pH should remain with the range
    of 4.5 to 8.0. The urinary creatinine should be greater than
    20 mg/dl; less than 20 mg/dl is considered dilute and less
    than 5 mg/dl is not consistent with human urine. Color
    may be a result of substances from food pigments,
    medications, or disease states.
    Ideally, the collection room should not contain a basin
    with
    running water, to reduce potential for specimen dilution,
    and blue pigment should be added to the toilet water.
    Reference: Cook, Caplan et al; The characterization of
    human urine for specimen validity determination in
    workplace drug testing: a review. J Anal Toxicol.
    2000;24:579-588.
    Urine Drug Testing in Clinical Practice: Pearls & Pitfalls:
    Purdue Pharma, 2005 (available free on request)
    Source: Art Jordan, MD, Sep 2005
323
Q
  1. Identify the “federal fi ve” drugs or drug classes that must
    be tested for in federal employees and federally regulated
    industries?
  2. Marijuana
  3. Cocaine
    3.Amphetamine/Methamphetamine
  4. Methadone
A
2800. Answer: A (1,2, &amp; 3)
Explanation:
Methadone, is not included in the federal fi ve.
The correct federal fi ve are:
1: Marijuana (delta-9-THC acid)
2: Cocaine (benzoylecgonine)
3: Opiates
4: Phencyclidine
5: Amphetamine/methamphetamine
Source: Art Jordan, MD, Sep 2005
324
Q
  1. “Joe the Doper” is one of your patients. You are
    prescribing oxycodone for a legitimate medical purpose,
    and his pain is well controlled. Joe had a routine urine
    drug test as part of a federal job interview and the screen
    reported opiates as “none detected”Joe “found religion”
    during his last incarceration in jail for cutting the heads
    off parking meters and swears that he has been taking the
    meds and not selling them on the street. Correct options
    at this time include:
  2. Increase his dose of oxycodone and recheck his urine
  3. Make a note in the chart that you really believe him and
    continue the oxycodone
  4. Change his pain medication to a fentanyl patch
  5. Order GC/MS specifi cally for oxycodone
A
  1. Answer: D (4 Only)
    Explanation:
    Standard urine opiate immunoassay is designed to detect
    only morphine and codeine, and will not detect
    oxycodone.
    GC/MS (Gas chromatography/mass spectrometry)will
    specifi cally detect oxycodone, or other specifi c substances
    as you indicate to the lab.
    You might also want to order a serum oxycodone level to
    get an idea of what the doses are achieving.
    Reference: UDT in Clinical Practice: Purdue Pharma, 2005
    Source: Art Jordan, MD, Sep 2005
325
Q
  1. What are the identifi ed problems of screening tools of
    drug abuse?
  2. Developed in psychiatric fi eld
  3. Rely on subjective reporting of the patients
  4. Not designed to detect prescription opioid abuse
  5. Designed to detect cocaine use
A
  1. Answer: A (1,2, & 3)

Source: Laxmaiah Manchikanti, MD

326
Q
  1. What is the explanation of a routine urine drug test
    in a patient receiving codeine with acetaminophen 240
    mg/day testing positive for codeine and negative for
    morphine?
  2. The laboratory made a mistake
  3. The morphine was metabolized faster than the codeine
    and was therefore excreted earlier
  4. The morphine was “neutralized” by the specifi c carrier
    agent in the Codeine with acetaminophen
  5. The patient is one of a small number who lack cytochrome
    P450 2D6 and cannot convert codeine to
    morphine
A
  1. Answer: D (4 Only)
    Explanation:
    A small number of patients lack cytochrone P450 2D6 and
    cannot convert part of the codeine to morphine, a normal
    metabolite of codeine
    Source: Art Jordan, MD, Sep 2005
327
Q
2804. What is the prevalence of co-existing diseases in drug
dependence?
1. Alcohol dependence (63%)
2. Any psychiatric disorder (74%)
3. Antisocial personality disorder (44%)
4. Major depression (25%)
A
2804. Answer: E (All)
Explanation:
Reference: Compton, et al. Am J Psychiatry 2001; 160:
890-895.
Co-existing Diseases and Drug Dependence
Alcohol dependence (63%)
Any psychiatric disorder (74%)
Antisocial personality disorder (44%)
Phobic disorder (41%)
Major depression (25%)
Source: Mark V. Boswell, MD, KSIPP 2005
328
Q

2805.Identify the components of the four cornerstones of
good clinical practice in chronic opioid prescribing.
1. A specifi c diagnosis that is opioid responsive
2. Verify no evidence of drug abuse
3. Document improved function
4. Manage side effects appropriately

A
  1. Answer: E (All)

Source: Mark V. Boswell, MD, KSIPP 2005

329
Q
  1. What are the characteristics of simple chronic pain?
  2. Pain lasting longer than 6 months.
  3. Tend to have no more distress or psychopathology than
    what is expected in the general population.
  4. Tend to continue working.
  5. Tend to maintain meaningful relationships
A
  1. Answer: E (All)
    Explanation:
    Simple Chronic Pain
    Pain lasting longer than 6 months.
    Tend to have no more distress or psychopathology than
    what is expected in the general population.
    Tend to continue working.
    Tend not to become overly reliant on medications, i.e.,
    have various ways to self-manage pain.
    Tend to maintain meaningful relationships
    Tend to maintain a sense of meaning and direction to their lives.
    Source: Murray McAllister, PsyD, LP - Spring 2004
330
Q
  1. What are the true statements about marijuana urine
    drug testing (UDT)?
  2. UDTs provide reasonable reliability
  3. Marinol tests positive
  4. Protonix may test false-positive
  5. Marijuana may be positive 2 years after use.
A
  1. Answer: A (1,2, & 3)
331
Q
  1. Somatoform disorders include the following:
  2. Physical symptoms suggestive but not fully explained by
    a general medical disorder
  3. Includes: somatization disorder
  4. Includes: conversion disorder
  5. Includes: major depression
A
  1. Answer: A (1,2, & 3)

Source: Renee R. Lamm, MD, Sep 2005

332
Q
  1. DSM-IV defi nition of substance abuse includes at least
    one of the following in 12 months:
  2. Maladaptive pattern leading to distress or impairment
  3. Recurrent failure to fi ll role
  4. Recurrent physically hazardous behavior
  5. Recurrent legal problems
A
2809. Answer: E (All)
Explanation:
SUBSTANCE ABUSE: DSM-IV DEFINITION
At least one in 12 months:
Maladaptive pattern leading to distress or impairment
Recurrent failure to fi ll role
Recurrent physically hazardous behavior
Recurrent legal problems
Continued use despite social problems
Never met dependence criteria
Source: Laxmaiah Manchikanti, MD
333
Q

2810.Identify accurate statements of open label studies of
opioids
1. Less then 50% of patients were continued on opioids at
2 yeras
2. Most of patients experiences at least one adverse event
3. No fi rm conclusions were made about tolerance and
addiction
4. Results were made applicable to general public

A
2810. Answer: A (1,2, &amp; 3)
Explanation:
Open Label Studies
44% on opioids at 2 years
80% of patients experienced at least one adverse event
No fi rm conclusions about tolerance and addiction
Patients in trials were highly selected
Results not generally applicable
5% of 1000 screened patients included
Source: Mark V. Boswell, MD, KSIPP 2005
334
Q
2811. What are common side effects leading to discontinuation
of opioids?
1. Constipation
2. Nausea
3. Somnolence
4. Hyperactivity
A
2811. Answer: A (1,2, & 3)
Explanation:
Side Effects are Common - Opioids are often discontinued
because of adverse event:
Constipation
Nausea
Somnolence
Vomiting
Source: Mark V. Boswell, MD, KSIPP 2005
335
Q

2812.Based on systematic reviews, what are the correct
statements about opioids?
1. Morphine and oxycodone are not equal
2. Morphine 20 mg/day and oxycodone 30 mg/day are
equally ineffective .
3. Improved functional status was conclusive
4. Opioids did not improve depression

A
  1. Answer: C (2 & 4)
    Explanation:
    Reference: Kalso E, Edwards JE, Moore A, McQuay JH.
    Opioids in chronic non-cancer pain: systematic review of
    effi cacy and safety. Pain 2004; 112;372-380.
    Morphine and oxycodone equally effective
    Morphine 20mg/day and oxycodone 30 mg/day not
    effective
    All patients in studies were on opioids previously
    Opioids did not improve depression
    Improved functional status unclear
    Source: Mark V. Boswell, MD, KSIPP 2005
336
Q

2813.Identify the suggested requirements to be included in an
agreement, if the patient is at high risk for medication
abuse or has a history of substance abuse:
1. Reasons for which drug therapy may be discontinued
(e.g., violation of agreement)
2. Requirement for medical interview with members of
immediate family
3. Urine/serum medication levels screening when requested
4. Periodic reports from a local law enforcement agency

A
  1. Answer: B (1 & 3)
    Explanation:
    Interviews with family members may be helpful under
    certain circumstances if clinically indicated and if within
    legal bounds of privacy, however, this is not listed by the
    Board.
    Periodic reports from law enforcement, without patient
    consent, would be inappropriate and possibly a violation
    of
    privacy laws.
    Source: Art Jordan, MD, Sep 2005
337
Q
  1. What are substance induced disorders?
  2. Delirium-intoxication or withdrawal
  3. Dementia
  4. Amnestic disorders
  5. Psychotic disorders
A
  1. Answer: E (All)

Source: Renee R. Lamm, MD, Sep 2005

338
Q
  1. A 42 year old physician who suffers from ankylosing
    spondylosis is referred for pain management. After 2
    months of treatment with long-acting opioids he feels
    so much better that he plans to return to work as an
    emergency room physician. Which of the following
    would be appropriate actions:
  2. Recommend he have neuropsychologic testing to document
    good mental function while taking opioids.
  3. Write a statement that he is capable of returning to the
    practice of emergency medicine at this time.
  4. Insist the physician inform his employer and the hospital
    of the medications he takes as a requirement to
    continue treatment.
  5. Inform him that legally he cannot return to practicing
    medicine while taking opioids.
A
  1. Answer: B (1 & 3)
    Explanation:
  2. Neuropsychologic testing can document the
    physician is mentally acute while on chronic opioid
    therapy.
  3. Without adequate documentation of the physician
    patient’s mental ability and the requirements of the
    hospital, the pain treatment physician could place him or
    herself at risk by making such a statement.
  4. The pain treatment physician cannot directly inform
    the interested parties of the patient’s medications, but can
    cease treating the patient if he or she feels the situation is
    not acceptable.
  5. There is no law in most states that prevents practicing
    medicine while taking opioids.
    Source: Roger Cicala, MD, Sep 2005
339
Q

2816.Factitious disorders consist of the following:
1. Physical or psychiatric symptoms that are intentionally
produced to assume a sick role
2. External incentives are present
3. Most severe form of Münchhausen Syndrome
4. Intentional symptoms with obvious goal

A
  1. Answer: B (1 & 3)

Source: Renee R. Lamm, MD, Sep 2005

340
Q
  1. The “Whizzinator” is which of the following:
  2. An electromechanical device for stirring alcoholic
    drinks
  3. A urologic testing device to measure the force of urine
    fl ow
  4. A suction device for maintaining an erection
  5. A commercially available device to thwart urine drug
    testing
A
  1. Answer: D (4 Only)
    Explanation:
    It is important that the physician know that there are
    many available devices to avoid detection of improper or
    illegal substances in the urine, including the
    “Whizzinator”; an artifi cial penis which contains urine
    “guaranteed” to be drug free.
    Source: Art Jordan, MD, Sep 2005
341
Q
  1. What are D.E.A considered “certain recurring
    concomitance of condemned behavior” in physician
    conviction?
  2. The physician involved used street slang rather than
    medical terminology for the drugs prescribed
  3. The physician warned the patient to fi ll prescriptions at
    different drug stores
  4. There was no logical relationship between the drugs
    prescribed and treatment of the condition allegedly
    existing
  5. The physician issued prescriptions to a patient known to
    be delivering the drugs to others
A
  1. Answer: E (All)
    Explanation:
    Explanation:
    All of the above answers are listed in the interim policy.
    In addition, the following are also listed as “certain
    recurring concomitance of condemned behavior”:
    1:An inordinately large quantity of controlled substances
    was prescribed
    2: Large numbers of prescriptions were written
    3: No physical examination was given
    4: The physician prescribed controlled drugs at intervals
    inconsistent with legitimate medical treatment
    5: The physician wrote more than one prescription on
    occasions in order to spread them out
    **You will frequently see some of these behaviors listed as
    “Red Flags”
    Source: Art Jordan, MD, Sep 2005
342
Q
  1. A prominent pain physician in North Carolina is treating
    Ms. Lavonia Gotrocks from Indiana. Traveling to see this
    physician greatly interferes with her social calendar, and
    she does not wish to make the trips more than every
    three months. She is currently taking sustained release
    oxycodone and hydromorphone.The physician writes her
    prescriptions on September 1, and then writes additional
    prescriptions for the same medications and indicates that
    they should be fi lled on October1, and November1. Her
    next appointment in the offi ce is December 1.Which of
    the following responses is/are correct?
  2. This is correctly called “alternate dating” and not covered
    by the “No Refi ll” rule
  3. Is a recurring tactic among physicians who seek to avoid
    detection when dispensing controlled substances for
    unlawful (nonmedical) purposes
  4. Is legal if done for no more than 3 months total (3 prescriptions)
  5. Is tantamount to writing a prescription authorizing
    refi lls of a schedule II controlled substance
A
  1. Answer: C (2 & 4)
    Explanation:
    “1” was commonly referred to as alternate dating prior to
    the F.A.Q.’s and was generally accepted by the D.E.A. as a
    legal but not recommended activity. The F.A.Q.’s actually
    stated that this action was acceptable, however this was
    specifi cally addressed as illegal in the “Interim Policy”
    statement in November of 2004.
    Source: Art Jordan, MD, Sep 2005
343
Q
  1. What is the impact of psychological factors in treatment
    of pain with a comorbid substance use disorder?
  2. Impedes diagnosis and complicates interventions
  3. Pain can mask addiction - switch to legal drugs
  4. Promotes regression and may induce hyperalgesia and
    extreme tolerance
  5. Positive impact as psychotherapeutic drugs provide
    excellent analgesia
A
  1. Answer: A (1,2, & 3)
    Explanation:
    Reference: Collins and Streltzer 2003, Am J. Addict 12:2,
    Covington an dKoltz, 2003 Prin of Addict Med
    Source: Renee R. Lamm, MD, Sep 2005
344
Q
  1. If a physician is aware that a patient is a drug addict
    and/or has resold prescription narcotics, which of the
    following is correct according to the “Interim Policy”
  2. It is merely recommended that the physician engage in
    additional monitoring of the patient’s use of narcotics
  3. The physician, as a D.E.A. registrant, has a responsibility
    to exercise a much greater degree of oversight to
    prevent diversion
  4. Should prescribe controlled substances for intervals of
    1-2 weeks only with frequent urine drug screens
  5. May not dispense controlled substances with the
    knowledge they will be used to support addiction or
    be resold
A
  1. Answer: C (2 & 4)
    Explanation:
    Reference: “Interim Policy Statement” November 2004
    Source: Art Jordan, MD, Sep 2005
345
Q
  1. Urine Drug Testing (UDT) may be useful in which of the
    following situations?
  2. To determine if a patient is taking the controlled substances
    prescribed
  3. To determine the patient’s state of hydration in an effort
    to regulate the dosage of medication
  4. To determine if the patient is taking medications and
    substances which are not prescribed by the physician
    administering the test
  5. To determine the half-life of the drugs prescribed
A
  1. Answer: B (1 & 3)
    Explanation:
    The state of hydration on an isolated sample would be of
    no use in determining dosage. Likewise, the half life of the
    medication could not be determined from an isolated
    urine sample, while only checking for drugs present
    It is most important that the ordering physician know
    exactly which drugs are included in the specifi c test, and
    which drugs may not be detected. Examples include
    oxycodone, fentanyl, and methadone which may not be
    detected in many basic urine drug screens on the market.
    Source: Art Jordan, MD, Sep 2005
346
Q
  1. A hypertensive crisis is most likely to result from the
    action of drugs from which one of the following drug
    classes:
  2. Tricyclic antidepressants
  3. Barbiturates
  4. Opioids
  5. Monoamine oxidase (MAO) inhibitors
A
  1. Answer: D (4 Only)
    Explanation:
    Monoamine oxidase (MAO) inhibitors such as
    tranylcypromine may precipitate a hypertensive crisis
    when used in the presence of certain foods that contain
    tyramine, or in the presence of certain sympathomimetic
    agents. Note also that opioids, particularly meperidine,
    may also (although rarely) precipitate a hypertensive crisis
    when used with MAO inhibitors.
347
Q
  1. Which of the following statements about U.S. enlisted
    men who became addicted to opioids in Vietnam are
    correct?
  2. Nearly 90% did not become addicted again within three
    years of return to the United States
  3. Relapse more common in older white soldiers
  4. Higher relapse rate in sons of alcoholic parents
  5. About 75% of soldiers who used heroin fi ve or more
    times became drug dependent
A
  1. Answer: E (All)
    Explanation:
    U.S. enlisted soldiers addicted to opioids in RVN did not
    follow the pattern of addicted civilians: on return to U.S.;
    the great majority no longer used the drug (heroin).
    Source: Psychiatry specialty Board Review By William M.
    Easson, MD and Nicholas L. Rock, MD
348
Q

2825.What are the true statements about early history of
Opium?
1. Arabia - (600-900 A. D.) used medicinally. When the
Koran forbade alcohol, Opium and Hashish became the
primary social drugs.
2. Galen - 100 A. D. - ‘great cure-all’
3. Greece - mixed with wine 100 B. C.
4. Eber’s Papyrus, 1500 B. C. - pain relief

A
  1. Answer: E (All)
    Explanation:
    History of Opium
    * Eber’s Papyrus, 1500 B. C. - pain relief
    * Greece - mixed with wine 100 B. C.
    * Galen - 100 A. D. - ‘great cure-all’
    * Arabia - (600-900 A. D.) used medicinally. When the
    Koran forbade alcohol, Opium and Hashish became the
    primary social drugs.
    Source: Roger Cicala, MD, Sep 2005
349
Q
  1. Strategies to reduce aberrant drug behaviors include:
  2. Random urine drug screens
  3. Narcotic contracts
  4. No early refi lls
  5. Opioid rotation
A
  1. Answer: A (1,2, & 3)
    Explanation:
    1, 2, & 3. Random drug screens, narcotic contracts, and
    aggressive refi ll policies (no early refi lls) have been felt to
    help control aberrant drug behaviors.
  2. Opioid rotation tries to address the issue of drug
    tolerance.
    Source: Trescot A, Board Review 2003
350
Q
  1. The characteristics of marijuana include:
  2. It may lower intraocular pressure
  3. A sign of acute intoxication is reddening of conjunctiva
  4. It has antiemetic properties
  5. Heavy chronic use can lower serum testosterone levels
    in men
A
  1. Answer: E (All)
    Explanation:
    The active ingredient in marijuana is Delta-9-
    tetrahydrocannabinol.
    In general, marijuana is a CNS stimulant causing
    tachycardia, giddiness, and , at high doses, visual
    hallucinations.
  2. Potential therapeutic uses include antiemesis in cancer
    chemotherapy and reduction of intraocular pressure in
    glaucoma.
  3. Acute intoxication is characterized by reddening of the
    conjunctiva (bloodshot eyes) owing to local vasodilation.
  4. Potential therapeutic uses include antiemesis in cancer chemotherapy and reduction of intraocular pressure in
    glaucoma.
  5. Chronic use has been associated with an amotivational
    syndrome and with a reduction in serum testosterone and
    sperm count
351
Q
  1. Sequelae of an acute cocaine overdose include
  2. Myocardial ischemia and high output cardiac failure
  3. Seizure activity
  4. Tremulousness and hyperthermia
  5. Blockade of inhibition of epinephrine
A
  1. Answer: A (1,2, & 3)
    Explanation:
    (Stoelting, Anesthesia and Co-Existing Disease, 3/e, pp
    528-529.)
    Acute cocaine overdose will increase in central
    catecholamine levels are increased. Cocaine inhibits the
    reuptake of norepinephrine. Increased circulating
    norepinephrine levels have numerous effects on the
    cardiac system, including coronary artery vasospasm, an
    increase in myocardial oxygen consumption, and an
    increase in systemic vascular resistance. These effects can
    cause high-output cardiac failure and cardiac ischemia.
    Source: Curry S.
352
Q
2829.Symptoms of withdrawal from opioids include the
following symptoms and signs:
1. Sweating
2. Restlessness
3. Irritability
4. Hot/cold fl ashes
A
  1. Answer: E (All)
353
Q
  1. Goals of pharmacotherapy in opioid addiction include
    the following:
  2. Prevention or reduction of withdrawal symptoms
  3. Prevention or reduction of drug craving
  4. Restoration to or toward normalcy of any physiologic
    function disrupted by chronic drug use.
  5. To provide addictive drugs to prevent relapse
A
  1. Answer: A (1,2, & 3)
    Explanation:
  2. Goals of pharmacotherapy include prevention or
    reduction of withdrawal symptoms
  3. Goals of pharmacotherapy include prevention or
    reduction of drug craving
  4. Goals of pharmacotherapy include restoration to or
    toward normalcy of any physiologic function disrupted by
    chronic drug use.
  5. Goals of pharmacotherapy also include prevention of
    relapse to use of addictive drugs.
354
Q
  1. What are the correct statements of Harrison Narcotics
    Tax Act of 1914?
  2. Required physicians to register and keep records of prescribed
    medications.
  3. Created Bureau of Narcotics of Treasury Department
    (and Federally run Heroin Clinics for addicts).
  4. 3,000 physician arrests during 1920s.
  5. By 1930 “addict pattern” was male, minority, criminal.
    But much smaller (?20,000).
A
  1. Answer: E (All)
    Explanation:
    Harrison Narcotics Tax Act of 1914:
    * Tax on all opium and coca.
    * Required physicians to register and keep records of
    prescribed medications.
    * Outlaws sale and distribution except prescribed by
    physician.
    * Created Bureau of Narcotics of Treasury Department
    (and Federally run Heroin Clinics for addicts).
    * The Bureau of Narcotics became the Prohibition Unit
    after passage of the Volstead Act. After Prohibition ended
    it became the Federal Narcotics Bureau and later the D. E.
    A.
    Harrison Act Case Law:
    * Three physician cases: Webb (1919), Moy (1920), and
    Behrman (1921).
    - Behrman: Physicians could not prescribe to ‘habitual
    users.
    - Webb and Moy: Must be “in the usual course of
    practice” to a “legitimate patient”.
    - 3,000 physician arrests during 1920s.
    * By 1930 “addict pattern” was male, minority, criminal.
    But much smaller (?20,000).
    * 1925 Linder case allowed physicians to prescribe long
    term or in addicted persons - but few would.
    Source: Roger Cicala, MD, Sep 2005
355
Q
2832. What are the requirements of continued controlled
substance prescribing?
1. Reduction in pain
2. Improvement in functional status
3. Lack of evidence of drug abuse
4. Lack of unmanageable side effects
A
2832. Answer: E (All)
Explanation:
Continued controlled substance prescribing requires:
* Diagnosis
* Reduction in pain
* Improvement in functional status
* Lack of evidence of drug abuse
* Documented informed consent
Source: Mark V. Boswell, MD, KSIPP 2005
356
Q
  1. Which of the following statements about alcohol
    withdrawal delirium are correct except ?
  2. May be precipitated by surgery
  3. Withdrawal seizures are most common 24 hours after
    withdrawal
  4. Delirium tremens has peak incidence four days after
    withdrawal
  5. Does not occur while still drinking
A
  1. Answer: D (4 Only)
    Explanation:
    Alcohol withdrawal delirium (delirium tremens, DT’s) is
    characterized by confusion, disorientation, fl uctuating or
    clouded consciousness, perceptual disturbances, delusions,
    vivid hallucinations, agitation, insomnia, mild fever, and
    marked autonomic arousal. Problems may apepar
    suddently or two or three days after cessation or
    redduction of heavy drinking, with a peak at the fourth or
    fi fth day. Symptoms may last four to fi ve weeks, but in the
    majority of patients, problems subside after three days.
    About one-third who develop alcohol withdrawal seizures
    (“rum fi ts”) go into delirium tremens. The best treatment
    is to prevent withdrawal by the use of benzodiazepines and
    a hig-calorie, high carbohydrate diet with supplemental
    vitamins.
    Source: Psychiatry specialty Board Review By William M.
    Easson, MD and Nicholas L. Rock, MD
357
Q
  1. What are the true statements describing history of
    spread of opium?
  2. Arabic traders spread use and cultivation to China by
    900 A. D.
  3. 1644 China outlawed tobacco, opium smoking became
    endemic
  4. 1700 British East India company smuggled opium from
    India to China for tea (China refused legitimate trade).
  5. 1839 - Because of “rampant addiction” Chinese emperor
    destroyed 45,000 pounds of British opium in Canton,
    beginning the Opium war
A
  1. Answer: E (All)
    Explanation:
    History of Opium
    * Arabic traders spread use and cultivation to China by
    900
    A. D.
    * 1644 China outlawed tobacco, opium smoking became
    endemic * 1700 British East India company smuggled opium from
    India to China for tea (China refused legitimate trade).
    * 1839 - Because of “rampant addiction” Chinese emperor
    destroyed 45,000 pounds of British opium in Canton,
    beginning the Opium war.
    * Britain won the ensuing war, receiving Hong Kong and
    forcing China to accept opium in trade.
    * Opium fi nally outlawed in China by 1913.
    Source: Roger Cicala, MD, Sep 2005
358
Q

2835.Chose the correct statements about use of opium in
Europe and America:
1. Paracelsus 1500 called laudanum (opium, cloves, and
alcohol) the “stone of immortality”.
2. Thomas Sydenham brought to England about 1700.
3. 1831 Sertuener given Nobel Prize for isolating individual
opiates.
4. 1853 - hypodermic syringe invented. The American
Civil War and Prussian Wars of 1860-1870 led to widespread
morphine injection.

A
  1. Answer: E (All)
    Explanation:
    Use in Europe and America
    * Opium knowledge was lost during Dark Ages,
    reintroduced after Crusades.
    * Paracelsus 1500 called laudanum (opium, cloves, and
    alcohol) the “stone of immortality”.
    * Thomas Sydenham brought to England about 1700.
    * 1831 Sertuener given Nobel Prize for isolating
    individual opiates.
    * 1853 - hypodermic syringe invented. The American Civil
    War and Prussian Wars of 1860-1870 led to widespread
    morphine injection.
    Source: Roger Cicala, MD, Sep 2005
359
Q
  1. What are the historical aspects of opiate abuse?
  2. “Soldier’s Disease” - morphine addiction - reported
    during American Civil War and Prussian Wars of 1860-
    1870.
  3. 10% of U. S. population used laudanum nonmedicinally.
  4. Widespread laudanum and opiate abuse in U. S. in late
    1800s. Sears carried syringe kits in catalogs. Estimated
    500,000 - 1,000,000 addicted in U. S.
  5. Heroin introduced in 1898 as “non-addicting” morphine.
A
  1. Answer: B (1 & 3)
    Explanation:
    Opiate Abuse
    * “Soldier’s Disease” - morphine addiction - reported
    during American Civil War and Prussian Wars of 1860-
    1870.
    * Widespread laudanum and opiate abuse in U. S. in late
    1800s. Sears carried syringe kits in catalogs. Estimated
    500,000 - 1,000,000 addicted in U. S.
    * 4% of U. S. population used laudanum nonmedicinally.
    * Laudanum addiction was considered fashionable.
    Typically described as ‘middle-aged, upper-class white
    female’. Samuel Taylor Coleridge, Elizabeth Barrett
    Browning, Thomas De Quincy all quite open about their
    addiction.
    * Heroin introduced in 1898 as “non-addicting”
    morphine. Used as cure for “Morphinism” similar to
    methadone today.
    Source: Roger Cicala, MD, Sep 2005
360
Q

2837.Choose the correct statement of U.S. opiate abuse
patterns:
1. Change from morphine to heroin in 50s-60s. Dramatic
increase in numbers 60s-70s.
2. Reduction in opiate abuse during 80s and early 90s
(?cocaine effect?)
3. Prescription opiate abuse increases in late 1990s (?or
increased awareness?)
4. Heroin resurgence begins late 1880’s.

A
  1. Answer: A (1,2, & 3)
    Explanation:
    U. S. Opiate Abuse Patterns
    * Change from morphine to heroin in 50s-60s. Dramatic
    increase in numbers 60s-70s.
    * Reduction in opiate abuse during 80s and early 90s
    (?cocaine effect?)
    * Prescription opiate abuse increases in late 1990s (?or
    increased awareness?)
    * Heroin resurgence begins late 1990s.
    Source: Roger Cicala, MD, Sep 2005
361
Q
  1. What are true statements about non-opiate drugs of
    abuse?
  2. Coca products began widespread use in 1880s in patent
    medicines and ‘soft drinks’. Endorsed by the Surgeon
    General in 1886.
  3. Amphetamines 1920s, used by military, physicians,
    widespread public use in 30s.
  4. Barbiturates and amphetamines began widespread use
    in 1940s, originally as OTC medications and in patent
    medications.
  5. Hallucinogens popularized in 1960s. Declined by late
    1970s.
A
  1. Answer: E (All)
    Explanation:
    Other Drugs of Abuse
    * Coca products began widespread use in 1880s in patent
    medicines and ‘soft drinks’. Endorsed by the Surgeon
    General in 1886.
    * Amphetamines 1920s, used by military, physicians,
    widespread public use in 30s.
    * Barbiturates and amphetamines began widespread use
    in 1940s, originally as OTC medications and in patent
    medications.
    * Marijuana use reported in 1880s. Became popular
    during prohibition and through the great depression.
    * Hallucinogens popularized in 1960s. Declined by late
    1970s.
    Source: Roger Cicala, MD, Sep 2005
362
Q
  1. Prescriptions for pain relief are receiving special
    attention by the Drug Enforcement Agency and the Offi ce
    of Inspector General due to . . .
  2. The signifi cant increase in the types of pain prescriptions
    available today.
  3. The signifi cant increase in the number of pain medications
    prescribed today.
  4. Evidence of doctor shopping by persons who obtain
    pain prescriptions from doctors either for their own
    abusive use or for illegal resale to others.
  5. A lessening of illegal drug traffi cking.
A
  1. Answer: A (1,2, & 3)
    Explanation:
    Prescriptions for pain relief are receiving special attention
    by the DEA and even the OIG because of the signifi cant
    increase in the types of pain prescriptions available today,
    the signifi cant increase in the number of prescriptions for
    these medications, and the evidence of doctor shopping by
    persons who obtain pain prescriptions from multiple
    doctors for their own abusive use or for illegal resale to
    others.
    Source: U.S. Drug Enforcement Administration News
    Release, October 23, 2001
    (http://www.usdoj.gov/dea/pubs/pressrel/pr102301.html).
    Source: Erin Brisbay McMahon, JD, Sep 2005
363
Q
  1. Choose the statements reflecting prescription drug use
    for non-medical purposes.
  2. Overall, it is believed that 10% of prescription drugs are
    used for non-medial purposes.
  3. Controlled prescription drugs for non-medical purposes
    have been reported to be used over 6 million people
    over the age of 12 years in the United States.
  4. Prescription drug abuse and illicit drug abuse may be
    signifi cantly higher in chronic pain patients than in
    normal population.
  5. Marijuana use is only second to cocaine.
A
  1. Answer: A (1,2, & 3)
    Explanation:
  2. 10% of prescription drugs are used for non-medical
    purposes.
  3. Over 6 million people over the age of 12 years in the
    United States have been reported to use controlled prescription drugs for non-medical purposes.
  4. Opioid abuse is seen in as high as 18% to 24% of the
    patients in chronic pain. Illicit drug use is seen in 14% to
    32% of chronic pain patients.
  5. Marijuana is the most commonly used illicit drug,
    followed by cocaine.
    Source: Laxmaiah Manchikanti, MD
364
Q
  1. Delirium is an acute confusional state that results from
    diffuse organic brain dysfunction. In the cancer patient,
    the causes include:
  2. Opioid toxicity
  3. Dehydration
  4. Hypoxia
  5. Brain metastases
A
  1. Answer: E (All)

Source: Reddy Etal. Pain Practice: Dec 2001, march 2002

365
Q
  1. Which of the following is NOT true regarding penalties
    for an unlawful intentional or knowing distribution or
    dispensation of controlled substances?
  2. As little as a one-year sentence may be imposed.
  3. A life sentence may be imposed.
  4. Supervised release will be imposed and will last from
    1-5 years.
  5. The court has the power to suspend a sentence or grant
    probation or parole, no matter what the facts of the
    case are
A
  1. Answer: D (4 Only)
    Explanation:
    Answer (a) is wrong because the court cannot grant
    probation, parole, or a suspended sentence if death or
    serious bodily injury results with respect to a S I or II drug.
    Reference: 21 USC 841.
    Source: Erin Brisbay McMahon, JD, Sep 2005
366
Q
  1. What is the suggested protocol of Ballantyne and Mao
    published in New England Journal of Medicine?
  2. Ensure benefi t will out weigh risk
  3. Evaluate possible addiction and problems with poor
    functioning
  4. Watch deterioration in function related to lack of motivation
    to improve
  5. Once opioids are started no further monitoring is required
A
2843. Answer: A (1,2, &amp; 3)
Explanation:
Ballantyne J, Mao J. Opioids for Chronic Pain. NEJM.
2003; 349: 1943-1953
Source: Mark V. Boswell, MD, KSIPP 2005
367
Q
  1. Partial fi lling of a prescription for a controlled substance
    listed in Schedule III, IV, or V is permissible, if:
  2. Each partial fi lling is recorded in the same manner as
    a fi lling.
  3. The total quantity dispensed in all partial fi llings does
    not exceed the total quantity prescribed.
  4. No dispensing occurs after six months after the date on
    which the prescription was issued.
  5. The prescribing practitioner authorizes the partial fi lling
    in writing.
A
  1. Answer: A (1,2, & 3)
    Explanation:
    Answer (1) is wrong; that is not a requirement for a partial
    fi lling of a prescription for a Schedule III, IV, or V
    substance.
    Reference: 21 CFR 1306.23.
    Source: Erin Brisbay McMahon, JD, Sep 2005
368
Q
  1. Which of the following is true with respect to a partial
    fi lling of a Schedule II prescription?
  2. If the remaining portion of the prescription is not or
    cannot be fi lled within 72 hours of the partial fi lling, the
    pharmacist must notify the prescribing physician.
  3. A partial fi lling is allowed for a terminally ill patient, and
    the prescribing physician has the sole responsibility to
    make sure the controlled substance is for a terminally
    ill patient.
  4. A partial fi lling is allowed for a terminally ill patient, and
    the prescribing physician and the pharmacist both have
    the responsibility to make sure the controlled substance
    is for a terminally ill patient.
  5. A partial fi lling is allowed for a terminally ill patient, and
    the pharmacist has the sole responsibility to make sure
    the controlled substance is for a terminally ill patient.
A
  1. Answer: B (1 & 3)
    Explanation:
    Answers (2) and (4) are wrong because both the
    prescribing physician and the pharmacist have the
    responsibility to make sure the controlled substance is for
    a terminally ill patient when the partial fi lling is for a
    terminally ill patient.
    Reference: 21 CFR 1306.13.
    Source: Erin Brisbay McMahon, JD, Sep 2005
369
Q
  1. A fax will serve as the original prescription for a Schedule
    II narcotic substance . . .:
  2. That is to be compounded for direct administration to
    the patient by parenteral, IV, intramuscular injection,
    subcutaneous or intraspinal infusion.
  3. For a resident of a long-term care facility.
  4. For a patient enrolled in a hospice program licensed by
    the state or certifi ed and/or paid for by Medicare, if the
    prescription notes that the patient is a hospice patient.
  5. For any terminally ill patient.
A
  1. Answer: A (1,2, & 3)
    Explanation:
    Answer (4) is wrong because a fax will not serve as the
    original prescription for a Schedule II controlled substance
    for any terminally ill patient.
    Reference: 21 CFR 1306.11.
    Source: Erin Brisbay McMahon, JD, Sep 2005
370
Q
  1. Pain physicians must consider the following in chronic
    long-term opioid therapy:
  2. Prolonged, high dose therapy may have adverse consequences
  3. The opioid formulation does not reduce development
    of tolerance
  4. Abuse potential of long acting and short acting formulations
    are the same
  5. Long term opioids produce adverse physiologic changes
    (immune, hormonal, pain, etc)
A
  1. Answer: E (All)
    Explanation:
    * Prolonged, high dose therapy may have adverse
    consequences
    * The opioid formulation does not reduce development of
    tolerance
    * Abuse potential of long acting and short acting
    formulations are the same
    * Long term opioids produce adverse physiologic changes
    (immune, hormonal, pain, etc)
    * Opioid rotation may help reduce the need for dose
    escalation and improve effi cacy
    Source: Mark V. Boswell, MD, KSIPP 2005
371
Q
  1. Substance dependence is best characterized by which of
    the following statements?
  2. Substance use is discontinued once there is insight about
    the physical or psychological harm that is likely to have
    been caused or exacerbated by the substance use
  3. Important social, occupational or recreational activities
    are decreased because of the use of substances.
  4. Symptoms may be due to another general medical
    condition.
  5. Tolerance and withdrawal are associated with it.
A
  1. Answer: C (2 & 4)

Source: Cole EB, Board Review 2003

372
Q

2849.No Schedule II prescription drug may be dispensed
without a written prescription, unless:
1. A physician calls in a refi ll.
2. It is dispensed directly by the physician to the ultimate
user.
3. The drug is dispensed to another physician.
4. There is an emergency situation which is defi ned by
regulation; in that case, an oral prescription may be
allowed.

A
  1. Answer: C (2 & 4)
    Explanation:
    Answer (1) is wrong because no Schedule II prescription
    can be refi lled. Answer (3) is wrong because there is no
    exception for dispensing drugs to another physician.
    Reference: 21 USC 829(a).
    Source: Erin Brisbay McMahon, JD, Sep 2005
373
Q
  1. A patient presents to you with injury of ankle strain.
    The ankle is swollen and extremely painful. However,
    there was no fracture. The patient is also on opioid
    maintenance treatment with methadone of 120 mg daily.
    True statements with regards to his pain management
    including the following:
  2. Opioid maintenance patients develop full tolerance to
    the analgesic effects of the maintenance dose of methadone.
  3. During opioid maintenance treatment, a cross-tolerance
    develops to all opioid agonist drugs.
  4. The usual maintenance dose of opioid maintenance
    does not provide any analgesia, and adequate analgesia
    will require higher doses of opioid agonists given more
    frequently than in the non-tolerant patient.
  5. The usual maintenance dose provides signifi cant analgesia,
    thus, no opioid agonists are required to provide
    analgesia for the acute pain.
A
  1. Answer: A (1,2, & 3)
    Explanation:
    Patients being maintained with methadone require special
    consideration for acute pain management in surgical or
    trauma situations.
  2. Maintenance patients develop full tolerance to the
    analgesic effects of the maintenance dose of methadone.
    2.During opioid maintenance treatment, a cross-tolerance
    develops to all opioid agonists drugs, accounting for the
    “blockade effect. Early research has demonstrated that
    stable opioid maintenance treatment patients could not
    distinguish 20 mg of intravenous morphine from
    intravenous saline.
  3. The usual maintenance dose does not provide any
    analgesia, and adequate analgesia will require higher doses
    of opioid agonists given more frequently than in the nontolerant
    patient.
    Methadone has a half-life of 24 to 36 hours, but its
    analgesic effects range from 4 to 6 hours,
    which is similar to morphine in both potency and
    duration.
    Morphine, Dilaudid, codeine, and other agonist drugs
    are appropriate for opioid maintenance
    treatment patients. Mixed agonist-antagonists (pentazocine, butorphanol,
    nalbuphine) and partial agonists
    (buprenorphine) must not be used, as they will
    precipitate an opioid withdrawal syndrome.
    Meperidine and propoxyphene should be avoided
    because of the risk of seizures at the higher
    doses required to produce analgesia in these patients.
  4. Maintenance doses of opioids do not provide adequate
    analgesia in acute pain.
374
Q
2851.Examples of the phenanthrene class of opioid include all
except:
1. Morphine
2. Fentanyl
3. Codeine
4. Meperidine
A
2851. Answer: B (1 & 3)
Explanation:
Morphine and codeine are phenanthrenes. Fentanyl and
meperidine are phenylpiperidines.
Source: Trescot A, Board Review 2003
375
Q
  1. 30 mg of Morphine Sulphate orally is equivalent to:
  2. 10mg MSO4 IV
  3. 20mg of oral oxycodone
  4. 1.5mg hydromorphone IV
  5. 20mg methadone
A
  1. Answer: E (All)
    Explanation:
    Although equipotent charts may vary, in general, 30mg of
    oral MSO4 is equivalent to 10mg MSO4 IV, 20mg of oral
    oxycodone, 1.5mg of IV hydromorphone, or 20mg of
    methadone.
    Source: Trescot A, Board Review 2003
376
Q
  1. Chronic alcoholism is associated with:
  2. Retrobulbar optic neuropathy
  3. Caudate calcifi cation
  4. Cerebellar anterior lobe degeneration
  5. Acoustic neuroma
A
  1. Answer: B (1 & 3)
    Explanation:
    Chronic alcoholism is associated with retrobulbar optic
    neuropathy, cerebellar anterior lobe degeneration,
    encephalopathy (Wernicke’s), subdural hematoma,
    amnestic disorder (Korsakoff ’s syndrome), dementia,
    peripheral neuropathy, pancreatitis, esophageal varices,
    duodenal ulcer, cardiomyopathy, pulmonary infections
    (especially tuberculosis), cirrhosis, and fetal alcohol
    syndrome.
    Source: Psychiatry specialty Board Review By William M.
    Easson, MD and Nicholas L. Rock, MD
377
Q
2854. In treatment opioid overdose, which of the following is
effective?
1. Methadone
2. L-alpha-acetylmethadol (LAAM)
3. Buprenorphine
4. Naloxone
A
  1. Answer: D (4 Only)
    Explanation:
    In Acute opioid overdose, the drug of choice is naloxone
    HCI (Narcan), 0.4-2.0 mg, preferably IV, every 2 to 3
    minutes, to a maximum dose of 10 mg. Nalocone is an
    opioid antagonist that blocks opioid receptors. Other
    opioid antagonists are nalorphine and levellorphane. In
    an opioid withdrawal procedure, naltrexone HCI
    (Trexan), clonidine, and methadone may be used as they
    have longer acting effects.
    Source: Psychiatry specialty Board Review By William M.
    Easson, MD and Nicholas L. Rock, MD
378
Q

2855.When a patient has been taking heavy doses of
barbiturates for an extended period, early symptoms of
withdrawal are likely to include:
1. Weakness
2. Insomnia
3. Anxiety
4. Tremulousness

A
  1. Answer: E (All)
    Explanation:
    Barbiturate withdrawal (especially short acting) usually
    results in weakness, insomnia, anxiety, tremulousness,
    abdominal discomfort, nausea and vomiting. With
    preexisting cardiovascular problems, there may be fatal
    reactions. Seizures generally precede delirium. Symptoms
    are more marked with secobarbital an dleast with
    phenobarbital withdrawal (due to its long half-life).
    Source: Psychiatry specialty Board Review By William M.
    Easson, MD and Nicholas L. Rock, MD
379
Q
  1. Physical dependence is a term defi ned as:
  2. The presence of withdrawal symptoms with abrupt discontinuation
    of drug.
  3. Is synonymous with addiction
  4. Reveals abrupt withdrawal symptoms when antagonist
    is administered
  5. Does not respond to a tapering dose, and requires detoxifi cation
A
  1. Answer: B (1 & 3)
    Explanation:
    Physical dependence is an occurrence that follows opioid,
    benzodiazepine, or other controlled substance use, and
    sometimes non-controlled substances, such as nicotine
    and alcohol, particularly revealed when abruptly
    discontinued. Titration is acceptable, and done slowly,
    with the caveat that physicians that do not have a special
    attachment to their DEA certifi cate do not use methadone
    as a taper. Opioid antagonist drugs can abruptly throw an
    individual into withdrawal.
    Source: Hans C. Hansen, MD
380
Q
  1. All states that have guidelines, laws, and/or regulations
    governing the use of controlled substances to treat pain
    set, as a minimum standard, that the physician should ask
    the patient whether he/she has
  2. Ever used prescribed controlled drugs before.
  3. Ever been to a pain doctor before.
  4. A history of chemical/substance abuse, including alcohol,
    illicit, and licit drugs.
  5. Had any tests related to his/her pain condition.
A
  1. Answer: E (All)
    Explanation:
    Reference: The Federation of State Medical Boards’ Model
    Policy for the Use of Controlled Substances for the
    Treatment of Pain (May 2004); www.fsmb.org.
    Explanation:
    The correct answer is E, all of the above, because each of
    these items plays into the minimum standards related to
    the taking of a patient history and the performance of a
    physical examination prior to prescribing controlled
    substances for the treatment of pain. A provider should
    not omit any of these questions from his/her interaction
    with the patient. Many of these questions can be set forth
    in a general history form. Providers may also develop
    special forms on substance abuse issues to use with
    patients prior to prescribing them controlled substances.
    In all cases, providers should cover these areas and more
    with their patients. Consult your state materials on the use
    of controlled substances for the treatment of pain.
    Source: Jennifer Bolen, JD, Sep 2005
381
Q
  1. What are the correct statements of urine drug testing?
  2. Thin-layer chromatography (TLC) is a relatively old
    technique, testing the migration of a drug on a plate or
    fi lm, which is compared to a known control
  3. Gas chromatography (CGMS) is most sensitive and specifi
    c test, most reliable, and labor intensive/costly
  4. Enzyme immunoassay is easy to perform/highly sensitive,
    more sensitive than TLC, and less expensive than
    GC/MS
  5. Rapid drug screens are not similar to other enzyme immunoassay
    testsand may be more expensive
A
  1. Answer: A (1,2, & 3)
    Explanation:
    Thin-layer chromatography (TLC)
    Relatively old technique, testing the migration of a drug
    on a plate or fi lm, which is compared to a known control
    Gas chromatography: liquid and mass spectometry
    (CGMS)
    Most sensitive and specifi c tests
    Most reliable
    Labor intensive/costly
    Several days to know results
    Used to confi rm results of other tests
    Enzyme immunoassay
    Easy to perform/highly sensitive
    More sensitive than TLC Less expensive than GC/MS
    Common tests
    EMIT (enzyme multiplied immunoassay test)
    FPIA (fl uorescent polarization immunoassay)
    RIA (radioimmunoassay)
    Screen only one drug at a time
    Rapid drug screens
    Similar to other enzyme immunoassay tests
    May be more expensive
    Source: Laxmaiah Manchikanti, MD
382
Q
2859. Which of the following may cause constipation in the
cancer patient?
1. Chronic opioid use for pain.
2. Iron supplementation for anemia.
3. Antacids containing Ca and Al.
4. Drugs with anticholinergic effects.
A
  1. Answer: E (All)

Source: Reddy Etal. Pain Practice: Dec 2001, march 2002

383
Q
  1. Evidence based medicine was developed in response to
    public need. What are the accurate statements?
  2. Patients and payers call for more accountability
  3. Changing patient-physician relationship with fi nancial
    focus is positive
  4. Concern about increasing costs
  5. Clinicians (and patients) are good at decision making
A
  1. Answer: B (1 & 3)
    Explanation:
    What is the public need?
    Patients and payers call for more accountability
    - Changing patient-physician relationship
    - Concern about increasing costs
    Clinicians (and patients) coping with information
    overload need tools for better decision making
    - Synthesis of alternative diagnostic and treatment options
    - Quantifi cation of outcomes
    Source: Laxmaiah Manchikanti, MD
384
Q

2861.Mixed opioid agonist-antagonists (nalbuphine,
pentazocine) have limited use in cancer patients because:
1. Respiratory depression is a common side effect
2. Interaction at the opioid receptor can precipitate withdrawal
symptoms.
3. Pruritus is a common side effect.
4. Effectiveness is limited by a dose-related ceiling effect.

A
  1. Answer: C (2 & 4)

Source: Reddy Etal. Pain Practice: Dec 2001, march 2002

385
Q
2862.Which of the following sedative medications have
analgesic properties:
1. Midazolam®
2. Ketamine®
3. Propofol®
4. Dexmedetomidine®
A
  1. Answer: C (2 & 4)
    Explanation:
    Ref: Frogen and Avram. Chapter 15. Nonopioid
    Intravenous Anesthetics. In: Clinical Anesthesia, 2nd
    Edition. Barash, Cullen, Stolling; Lippencott, 1992, pg 388
    Source: Day MR, Board Review 2003
386
Q
  1. What are the steps to avoiding trouble with misuse of
    controlled substances?
  2. Know your state laws and regulations
  3. Know your Medical Licensure Board’s guidelines on
    prescribing controlled substances
  4. DEA statement about your state on its website
  5. Follow the rules of advocacy groups and organization
    supporting unrestricted use of controlled substances
A
  1. Answer: A (1,2, & 3)
    Explanation:
    AVOIDING TROUBLE
    Know your state
    -what laws and regulations are in place?
    -does your Medical Licensure Board have guidelines on
    prescribing controlled substances?
    -what does DEA say about your state on its website?
    http://www.dea.gov/pubs/state_factsheets.html
    Source: Erin Brisbay McMahon, JD, Sep 2005
387
Q
  1. What are some of the communication issues faced by
    health care providers in terminal patients?
  2. Diagnosis and prognosis
  3. Advanced directives and do-not-resuscitate(DNR)
    orders
  4. Spiritual needs
  5. Symptom Management
A
  1. Answer: E (All)

Source: Reddy Etal. Pain Practice: Dec 2001, march 2002

388
Q
  1. Patient complains of low back pain, headaches, and
    depression, She is taking Lortab (hydrocodone) 10/500
    six per day, Fioricet (butalbital) six per day, and Paxil
    (paroxetine) 20mg per day. She complains of inadequate
    pain relief. Appropriate medication management would
    include:
  2. Counsel patient on the amount of acetaminophen, and
    change her to lower acetaminophen products
  3. Start methadone 10mg six per day
  4. Change her antidepressant to a super-selective serotonin
    reuptake inhibitor
  5. Change the butalbital to a triptan
A
  1. Answer: B (1 & 3)
    Explanation:
    Patient is taking 4950mg acetaminophen per day (Lortab
    has 500mg per tablet = 3000mg, Fioricet has 325mg per
    tablet = 1950mg). Toxic acetaminophen levels range from
    3 to 4 gm per day. The equivalent dose of methadone
    would be 10mg TID (hydrocodone 60mg per day =
    morphine 60mg; decrease dose by 1/2 =30mg, divide by 3
    and dose q 8hr would be 10mg q 8hrs). Paroxetine is a
    potent CYP2D6 inhibitor and will inhibit the metabolism
    of hydrocodone to hydromorphone, leading to decreased
    analgesia. Neither escitalopram (Lexapro) or citalopram
    (Celexa), which are considered “super-selective SSRIs”,
    will inhibit CYP2D6. Butalbital and triptons can cause
    rebound headaches, and should be changed; prophylactic
    medications may be needed. Methadone gives good relief
    of headaches, but butalbital will decrease methadone
    levels.
    Source: Andrea M. Trescot, MD
389
Q
  1. Drug testing may be performed by any of the following
  2. Hair samples
  3. Saliva testing
  4. Serum drug testing
  5. Urine drug screening
A
2866. Answer: E (All)
Explanation:
Drug Testing may be performed by any of the following:
Urine Drug Screening
Specifi c drug analysis (blood)
Hair Samples
Saliva Testing
Serum Levels
Source: Laxmaiah Manchikanti, MD
390
Q
  1. Regarding the metabolism of opioids:
  2. Some opioids are metabolized by glucuronidation
  3. Some opioids are metabolized by the P450 enzyme
    system
  4. M6G is an analgesic metabolite of morphine
  5. M3G is an analgesic metabolite of morphine
A
  1. Answer: A (1,2, & 3)
    Explanation:
    Some opioids, such as morphine, are metabolized by
    glucuronidation, while other opioids, such as
    propoxyphene are metabolized by the P450 system. M6G
    is analgesic while M3G is probably produces hyperalgesia.
    Reference: Sjogren P, Jensen NH, Jensen TS. Disappearance
    of morphine-induced hyperalgesia after discontinuing or
    substituting morphine with other opioid agonists. Pain
    1994 Nov;59(2):313-6.
    Source: Andrea M. Trescot, MD
391
Q
  1. The following statements are true:
  2. All opiates are opioids
  3. All opioids are opiates
  4. All opioids are narcotics
  5. All narcotics are opioids
A
  1. Answer: B (1 & 3)
    Explanation:
    Opiates are naturally occurring alkaloids such as
    morphine. Opioids are natural or synthetic compounds
    that work at the opioid receptor. All opiates are opioids,
    and all opioids are narcotics. However, not all opioids are
    opiates. All narcotics are opioids; examples of nonopioid
    narcotics include marijuana and LSD.
    Source: Andrea M. Trescot, MD
392
Q
  1. What are the correct statements about urine drug testing
    (UDT)?
  2. A UDT would be positive if the patient took the drug
    (true positive) and negative if the drug was not taken
    (true negative).
  3. Sensitivity of a test is the ability to identify a particular
    drug.
  4. False-positive or False-negative results can occur, so it is
    imperative to interpret the UDT results carefully.
  5. Specificity is the ability to detect a class of drugs.
A
  1. Answer: B (1 & 3)

Source: Laxmaiah Manchikanti, MD

393
Q
  1. Which of the following statements about
    biotransformation are true ?
  2. Biotransformation often produces metabolites with less
    affi nity for receptors than the parent drug
  3. Biotransformation often produces metabolites with a
    higher renal clearance than the parent drug
  4. Biotransformation often entails multiple enzyme-catalyzed
    reactions
  5. Biotransformation reactions often occur in the liver
A
  1. Answer: E (All)
    Explanation:
    Biotransformation generally produces metabolites that are
    more water soluble than the parent drug. The metabolites
    are less lipophilic and, hence, are poorly reabsorbed in the
    kidney, thus facilitating elimination.
394
Q
2871. Which of the following adverse effects if associated with
the use of Neuroleptic agents ?
1. Acute dystonia
2. Gynecomastia
3. Sedation
4. Loss of libido
A
  1. Answer: E (All)
    Explanation:
    Acute dystonia, gynecomastia, sedation and loss of libido
    may all be seen in patients being treated with neuroleptic
    agents.
395
Q
  1. Which of the following statements best describes a
    characteristic of the antimigraine agent ergotamine?
  2. It promotes vasodilatation
  3. It is useful in reducing premature contractions of the
    uterus
  4. It acts as a serotonin (5HT)-receptor antagonist
  5. It acts as an alpha-adrenoceptor agonist
A
  1. Answer: D (4 Only)
    Explanation:
    Ergotamine actions are mediated by agonist actions at
    both
    serotonin (5HT)-receptors and ?-adrenoceptor.
    Ergotamine causes vasoconstriction and is contraindicated
    during pregnancy.
396
Q
  1. Effects on the respiratory system by opioids include:
  2. Equipotent doses of opioids result in equal amounts of
    respiratory depression
  3. Depression of cough is a different mechanism than respiratory
    depression
  4. There is a direct respiratory depression effect on the
    medulla
  5. Respiratory rate decreases decrease fi rst and then CO2
    and hypoxia response decreases.
A
  1. Answer: A (1,2, & 3)
    Explanation:
    Equipotent doses of opioids are equipotent on the
    respiratory system, and there is a direst effect on the
    medulla. The cough suppression is a different mechanism
    than the respiratory response. However, respiratory rate
    will only drop after the CO2 and hypoxia responses have
    decreased.
    Source: Andrea M. Trescot, MD
397
Q
  1. Choose the correct statement about state board rules
    physicians do not know:
  2. Cannot Rx Schedule II or III for family members
  3. Can provide samples of unscheduled drugs for family,
    but MUST document in a medical record
  4. Cannot Rx to anyone (including friends) if you have
    not documented their H&P and have a current chart
    on fi le.
  5. Can Rx for yourself
A
  1. Answer: A (1,2, & 3)
    Explanation:
    State Board Rules You Might Not Know
    Cannot Rx Schedule II or III for family members
    Can provide samples of unscheduled drugs for family, but
    MUST document in a medical record
    Cannot Rx for anyone in sexual relationship, EVER.
    Cannot Rx for yourself, EVER.
    Cannot Rx to anyone (including friends) if you have not
    documented their H&P and have a current chart on fi le.
    Source: Laxmaiah Manchikanti, MD
398
Q
  1. Which of the following statements are true with regards
    to the Controlled Substances Act of the Comprehensive
    Drug Abuse Prevention and Control Act of 1970 ?
  2. It is the legal foundation of the government’s fi ght
    against the abuse of drugs and other substances.
  3. It is a consolidation of numerous laws regulating the
    manufacture and distribution of narcotics, stimulants,
    depressants, hallucinogens, anabolic steroids and
    chemicals used in the illicit production of controlled
    substances.
  4. All the substances that are regulated under existing federal
    law are placed into I of V schedules.
  5. Schedule I is reserved for the least dangerous drugs that
    have the highest recognized medical use.
A
  1. Answer: A (1,2, & 3)
    Explanation:
    The Controlled Substances Act (CSA), title 2 of the
    Comprehensive Drug Abuse Prevention and Control Act
    of 1970 is the legal foundation of the government’s fi ght
    against the abuse of drugs and other substances. This law
    is a consolidation of numerous laws regulating the
    manufacture and distribution of narcotics, stimulants,
    depressants, hallucinogens, anabolic steroids, and
    chemicals used in the illicit production of controlled
    substances.
    All the substances that are regulated under existing federal
    law are placed into I of V schedules. This placement is
    based upon the substances’ medicinal value, harmfulness,
    and potential for abuse or addiction.
    Schedule I is reserved for the most dangerous drugs that
    have no recognized medical use.
    Schedule V is the classifi cation used for the least
    dangerous drugs.
    The Act also provides a mechanism for substances to be
    controlled, added to a schedule, decontrolled, removed
    from control, rescheduled, or transferred from one
    schedule to another.
    Source: Manchikanti L, Board Review 2005
399
Q
  1. What are correct statements of Food and Drug Act
    Amendments?
  2. Durham-Humphrey 1951 - make OTCs require prescriptions.
  3. Boggs Act (1951)
  4. Narcotic Control Act (1956)
  5. Drug Abuse Control Acts of 1956 and 1958 labeled ‘potential
    drugs of abuse’ and gave power to DEA precursors
    to regulate.
A
  1. Answer: A (1,2, & 3)
    Explanation:
    Food and Drug Act Amendments
    * Durham-Humphrey 1951 - make OTCs require
    prescriptions.
    * Boggs Act (1951)
    * Narcotic Control Act (1956)
    * Drug Abuse Control Acts of 1965 _and 1968 labelled
    ‘potential drugs of abuse’ and gave power to DEA
    precursors to regulate.
    Source: Roger Cicala, MD, Sep 2005
400
Q
2877.What is the level of care - necessary to achieve and
maintain abstinence from opioids?
1. Medically managed inpatient treatment
2. Intensive outpatient program
3. Residential treatment program
4. Medically managed phone consultations
A
2877. Answer: A (1,2, &amp; 3)
Explanation:
Level of Care - Necessary to Achieve &amp; Maintain
Abstinence
* Medically managed inpatient treatment
- Medical/surgical hospital
- Psychiatric hospital
* Medically supervised inpatient treatment
* Partial hospitalization
* Intensive outpatient program
* Residential treatment program
Source: Kennison Roy, MD
401
Q

2878.What are the perceived barriers to non-opioid
management?
1. The opioid model words well from a business standpoint
2. Easy to assemble a multidisciplinary team
3. “Rebound pain” phenomenon after detoxifi cation
4. Multidisciplinary model works well from a business
standpoint

A
2878. Answer: B (1 &amp; 3)
Explanation:
Barriers to Non-Opioid Management
* Payment issues
- The opioid model works well from a business
standpoint
* System diffi culties
- Hard to assemble a multidisciplinary team
* Issues of detoxifi cation
- “Rebound pain” phenomenon
Source: Kennison Roy, MD
402
Q

2879.The Federation of State Medical Boards (FSMB)
encourage practitioners to include the following in the
medical record :
1. Medical history
2. Pain history
3. Working diagnosis
4. History of allergies to opioids

A
  1. Answer: A (1,2, & 3)
    Explanation:
    The Federation of State Medical Boards requires specifi c
    documentation in the medical record to defi ne legitimate
    medical need when controlled substances are used. This
    includes: medical history, substance or chemical abuse,
    pain history, appropriate studies, working diagnosis,
    treatment plan, rationale for treatment selected, patient education, and documentation that the patient and
    physician understand the treatment goals. The standard
    medical history course will cover allergies to medications,
    but not necessarily relevant to the concept of legitimate
    medical need. For appropriate prescription habitry to be
    realized, diagnosis, historical features and exam must meet
    the need for the medication, clearly outlined and
    understood in the medical record.
    Source: Hans C. Hansen, MD
403
Q
  1. The action of Tramadol is considered:
  2. A centrally acting synthetic opioid analgesic.
  3. Associated with mu-opioid receptor activity.
  4. Inhibition of reuptake of norepinephrine and serotonin.
  5. Tramadol induced analgesia is totally reversed by naloxone.
A
  1. Answer: A (1,2, & 3)

Source: Hansen HC, Board Review 2004

404
Q

2881.A COX 2 selective agent may be preferred in the
following:
1. History of GI bleed or complicated ulcer
2. Anticoagulant use
3. Oral corticosteroid use
4. Age < 60

A
  1. Answer: A (1,2, & 3)

Source: Jackson KC. Board Review 2003

405
Q

2882.All of the following statements are correct about
detoxifi cation EXCEPT:
1. In one year post completion outcome data all patients
still had some pain - most had much less
2. In one year post completion outcome data 70% had
achieve continuous sobriety
3. In one year post completion outcome data pain was not
ever worse without narcotics than it was while taking
narcotics
4. In one year post completion outcome data only 20% had
continued sobriety

A
2882. Answer: D (4 Only)
Explanation:
* Outcome studies are becoming available
- One year post completion outcome data:
All patients still had some pain - most had much less
70% had achieved continuous sobriety
Pain was not ever worse without narcotics than it was
while taking narcotics.
Source: Kennison Roy, MD
406
Q

2883.Controlled substance abuse in work place may be
identifi ed by the following signs?
1. Work performance alternating between periods of high
and low productivity and mistakes made due to inattention,
poor judgment and bad decisions.
2. Confusion, memory loss, and diffi culty concentrating
or recalling details and instructions. Ordinary tasks
require greater effort and consume more time
3. Interpersonal relations with colleagues, staff and patients
suffer.
4. Promptly admits errors or accepts blame for errors or
oversights

A
  1. Answer: B (1 & 3)
    Explanation:
    Watch for signs:
    Work performance alternating between periods of high
    and low productivity and mistakes made due to
    inattention, poor judgment and bad decisions;
    Confusion, memory loss, and diffi culty concentrating or
    recalling details and instructions. Ordinary tasks require
    greater effort and consume more time;
    Interpersonal relations with colleagues, staff and patients
    suffer. Rarely admits errors or accepts blame for errors or
    oversights;
    Heavy “wastage” of drugs;
    Sloppy recordkeeping, suspect ledger entries and drug
    shortages;
    Inappropriate prescriptions for large narcotic doses;
    Insistence on personal administration of injected narcotics
    to patients;
    Source: Erin Brisbay McMahon, JD, Sep 2005
407
Q
  1. Controlled substance is considered a national epidemic
    in U.S. What are correct statements showing grim
    national statistics?
  2. Opioid abuse increased 85% from 1994-2000
  3. Oxycodone abuse increased 166% since 1994
  4. Hydrocodone abuse increased 116% since 1994
  5. Methadone abuse increased 140% since 1994
A
  1. Answer: E (All)
    Explanation:
    Source: Manchikanti et al., National All Schedules
    Prescription Electronic Reporting Act (NASPER):
    Balancing Substance Abuse and Medical Necessity, Pain
    Physician 2002
    GRIM NATIONAL STATISTICS
    Opioid abuse increased 85% from 1994-2000
    Oxycodone abuse increased 166% since 1994
    Hydrocodone abuse increased 116% since 1994
    OxyContin suspected in 282 overdose deaths during a 19-
    month period
    Source: Erin Brisbay McMahon, JD, Sep 2005
408
Q

2885.Choose the correct statements describing opioids in
patients with substance abuse?
1. Federal Guidelines allow for use of opioids for analgesia
in persons with substance abuse disorder for “legitimate
medical reasons”.
2. No clear documentation of the pain problem is needed
to demonstrate the physician without proper credentials
is not attempting to detoxify an opiate abuser.
3. State regulations in certain states do no allow for prescription
of opioids in patients with substance abuse
and consider prescribing opioids in known substance
abusers malprescribing.
4. Federal Guidelines do not allow for use of opioids for
analgesia in persons with substance abuse disorder for
“legitimate medical reasons”.

A
  1. Answer: B (1 & 3)
    Explanation:
    Federal Guidelines allow for use of opioids for analgesia in
    persons with substance abuse disorder for “legitimate
    medical reasons”.
    Clear documentation of the pain problem is needed to
    demonstrate the physician without proper credentials is
    not attempting to detoxify an opiate abuser.
    State regulations in certain states do no allow for this, and
    consider prescribing opioids in known substance abusers
    malprescribing.
    Source: Laxmaiah Manchikanti, MD
409
Q

2886.What are the correct statements about controlled
substance abuse?
1. Almost half a ton of prescription narcotics reached six
counties in Eastern Kentucky from 1998-2001, equating
to .75 pound for every adult in those counties.
2. On a per capita basis, Eastern Kentucky drugstores,
hospitals, and legal outlets receive more prescription
painkillers than anywhere else in the United States.
3. Nationally, emergency room visits for hydrocodone
overdoses increased 500 percent from 1990-2000
4. OxyContin sells on the street for about $10/pill; Lortab
sell for $2/pill and Lorcet for $1/pill

A
  1. Answer: A (1,2, & 3)
    Explanation:
    Source: Linda Johnson, Eastern Kentucky: Painkiller
    Capital, Lexington Herald-Leader, Jan.19, 2003 and Linda
    Johnson, Lesser-Known Favorites Cheap, Abundant,
    Lexington Herald-Leader, Jan.19, 2003.
    Case Study: Eastern Kentucky
    * Almost half a ton of prescription narcotics reached six
    counties in Eastern Kentucky from 1998-2001, equating to
    .75 pound for every adult in those counties.
    * On a per capita basis, Eastern Kentucky drugstores,
    hospitals, and legal outlets receive more prescription
    painkillers than anywhere else in the United States.
    The Escalating Problem: Hydrocodone
    * Nationally, emergency room visits for hydrocodone
    overdoses increased 500 percent from 1990-2000
    * Three Eastern Kentucky counties had enough Lortab,
    Lorcet, and Vicodin pills in 2001 to provide every adult in
    those counties with 156 pills
    * Oxycontin sells on the street for about $40/pill; Lortabs
    sell for $20/pill and Lorcets for $9/pill
    Source: Erin Brisbay McMahon, JD, Sep 2005
410
Q
  1. What are Federation of State Medical Boards Guidelines
    for the Treatment of Pain?
  2. Use of controlled substances, including opiates may be
    essential in the treatment of pain
  3. Effective pain management is a part of quality medical
    practice
  4. Patients with a history of substance abuse may require
    monitoring, consultation, referral and extra documentation
  5. MD’s should not fear disciplinary action for legitimate
    medical purposes
A
  1. Answer: E (All)
    Explanation:
    Federation of State Medical Boards Guidelines for the
    Treatment of Pain
    Use of controlled substances, including opiates may be
    essential in the treatment of pain
    Effective pain management is a part of quality medical
    practice
    Patients with a history of substance abuse may require
    monitoring, consultation, referral and extra
    documentation
    MD’s should not fear disciplinary action for legitimate
    medical purposes
    Source: Laxmaiah Manchikanti, MD
411
Q
  1. What are the reasons for drug testing in patients in your
    practice?
  2. To assess if the patient is taking the medications prescribed
  3. To assess if the patient is taking substances/drugs NOT
    prescribed
  4. To assess if the patient is taking licit and illicit drugs
  5. To assess if the prescribed drugs caused diabetes
A
  1. Answer: A (1,2, & 3)

Source: Laxmaiah Manchikanti, MD

412
Q
2889. An opioid treatment program must provide which of the
following?
1. Drug abuse testing services
2. Vocational services
3. Educational services
4. Medical services
A
  1. Answer: E (All)
    Explanation:
    All of the above are required services for an OTP.
    Refernece: 42 CFR 8.12.
    Source: Erin Brisbay McMahon, JD, Sep 2005
413
Q
  1. The pain management physician is challenged to find
    non-traditional approaches for pain management.
    Effective treatments to be considered in a multimodality
    approach include
  2. Myofascial relief and musculoskeletal treatment strategies.
  3. Psychological drug therapy.
  4. Chiropractic care.
  5. Naturopathic medicine.
A
  1. Answer: E (All)
    Explanation:
    It is advisable that an allopathic physician stay true to
    scientifi c and well-validated approaches when treating
    pain. Many times the pain management physician is a
    referral of desperation, and expectations of the patient
    may be unrealistically high. Alternative therapies such as
    naturopathic medicine may be trialed, in conjunction with
    well-established treatment to enhance positive outcome.
    The patient should understand from the beginning the
    treatment style, policy and procedures of the clinic, and
    expectations, particularly if controlled substances are
    being used.
    Source: Hans C. Hansen, MD